You are on page 1of 294

‫‪SMLE‬‬

‫‪Collections‬‬
‫‪V2‬‬

‫ــــــــــــــــــــــــــــــــــــــــــــــــــــــــــــــــــــــــــــــــــــــــــــــــــــــــــــــــــ‬
‫‪Updated version with more than 1900 New questions‬‬
‫‪Collected and designed by Sami Althobiti‬‬
‫من منطلق التسهيل في تجميع األسئلة الجديدة وكما الحظ الجميع مدى انتشار تجارب زمالئنا األطباء‬
‫والطبيبات في مختلف وسائل التواصل وكون تلك األسئلة والتجارب ال تأتي مجتمعة فكان الهدف من هذه‬
‫المذكرة البسيطة تجميع األسئلة الجديدة والتجارب في قالب واحد كما كتبها األطباء والطبيبات كل ذلك‬
‫ألجل التسهيل على الجميع للوصول بأسرع وقت ولإلطالع على مالحظات من اختبروا االختبار الجديد‬

‫هناك عدة مالحظات قبل البدء فاألسئلة منقولة كما كتبها الزمالء والزميالت ويُرجى مراجعة األجوبة‬
‫وحلها بأنفسكم نظرا ً لعدم دقة األجوبة فالغرض الرئيسي معرفة األسئلة وطبيعتها ثم حلها فيما بعد‬

‫التجارب والنصائح تم فصلها عن األسئلة وجعلها في آخر المذكرة لالطالع عليها‪،‬‬


‫هناك عدد كبير من األسئلة كُتبت باللغة العربية معظمها المطلوب منه واضح والقليل مبهمة إلى حد ما‬
‫نشكر جميع من كتب ونقل ولو سؤاالً واحدا ً من هذه األسئلة والرسول صلى هللا عليه وسلم يقول "وهللا‬
‫في عون العبد ما كان العبد في عون أخيه" وجزاهم هللا خير الجزاء‪.‬‬

‫تم تجميع هذه األسئلة من قروبات الواتس بشكل عام وقنوات التليقرام‬
‫وسيتم تحديث هذه المذكرة تباعا ً ان شاء هللا في حال نُشرت أسئلة جديدة‬
‫تم تحديث هذه المذكرة بتاريخ ‪ 4102 / 01 / 42‬وإضافة مايقارب ‪ 1500‬سؤال جديد من تاريخ‬
‫النسخة األولى الموافق ‪ 4102 / 9 / 01‬ليُصبح مجموع األسئلة الجديدة في هذه المذكرة حتى هذا‬
‫التاريخ ما يزيد على ‪ 1900‬سؤال‪.‬‬

‫قام بتجميع وتنسيق هذه األسئلة والتجارب أخوكم سامي الثبيتي‬


‫شكر خاص لمن قام بإرسال ومشاركة األسئلة الجديدة‬
‫إلضافة أسئلة جديدة على اإليميل أو الواتس اب‬
‫‪SMLE-Collections@hotmail.com‬‬
‫‪0505650113‬‬
‫أو إضافتها بالرابط العام لجمع األسئلة هنا‬

‫‪October 24, 2017‬‬ ‫‪2‬‬


INDEX

New Questions from June to


4 1
11, September, 2017
New Questions from 11,
50 September to 22 October 2
2017

286 Experiences and Advices 3

294 Resources Suggested 4

October 24, 2017 3


Q/ man with aortic stenosis develop syncope what is the cause :
systemic hypotension

Q/ women at 40 week gestation suddenly developed dyspnea with


hypoxia and chest pain :
my answer is amniotic fluid embolism
other options are pulmonary embolism and mi

Q/ patient with auditory hallucination and delusion for one month and
the doctor diagnose him schizophrenia what is the wrong in dx :
my answer : duration should be at least 6 months

Q/ first line depression ttt :


ssri

Q/ women found unconscious and empty bottle of medication near to


her and she was diagnosed with depression previously on exam
dilated un responsive pupil what is the drug:
serotonin syndrome my answer
acyticlcoline
dopamine

Q/ about 20Q about incidence and prevalence and relative risk .

Q/ patient on warfarin come with melena and pt high what ttt:


iv vitamin k my answer
fresh frozen plasma
protamine sulphat

Q/ patient cannot extend his leg at knee joint which muscle


responsible ?
Sartorius muscle
quadriceps
biceps femoris

Q/ patient was in outpatient clinic in waiting room sudden loss


conscious and nurse found him not breath what you will do :
chest compression 30/2

October 24, 2017 4


check carotid pulse
give tow breath

Q/ painless ulcer )) syphilis

Q/ end stage liver diseases with fungal infection ttt : capsufungin

Q/ apoptosis :
p53
...
Q/ student with 10 of his classmate developed dry cough mild s.o.b
and bilateral consolidation :
leogenialla my answer
mycoplasma
staph
strept.

Q/ nephrotic syndrome ttt:


predinslone my answer
azathioprine

Q/ patient with burn 3 degree anterior trunk and lower lift limb and
upper left limb 70 kg how many liters in first 8 hours :
6.5 my answer
8.5
10.5
12.5

Q/ vitamin k dependent factors .

Q/ best ttt for traveler diarrhea ciprofloxacin

Q/ Child , iemplical swelling , yellow discharge + pic

Q/ ‫جتك دعوه تتكلم عن دواء بحفل او شيء زي كذا وفيه شركه اخترعت دوا جديد قالت لك ابغى‬
‫ارعى الحملة ايش تسوي ؟‬

October 24, 2017 5


Q/ von Walberg most important test ?
Pt
Ptt
Bt

Q/ child 13 month witch of the fallowing consider milestone ?


Crol
Walk
Sitting without support
Walking with holding furniture

Q/ child with parking cough what u expects to find in osculation ?

Q/ child with skin rash honey cluster Dx

Q/ infant parking cough what is the organism ?

Q/ mRNA to rRNA type of band ( something like that ^)

Q/ girl with morning periorbital edema

Q/ 8- 34 wk + open cervix + plugging membrane + fetal HR .. (forget


no.) what is your 1st management ?

Q/ where surgeon well do vagus nerve crush ?

Q/ posterior ... N. Injury

Q/ pt treated for peptic ulcer with triple therapy now she is free of sx
what u'll do ?
-nothing
-H pylori antigen test
-endoscopy

Q/ Sx of epiglottitis diagnostic test ?


-Nasopharyngeal soap
-...etc.

Q/ heart duct emperionic origination ? Specific name not Mesoderm etc.

October 24, 2017 6


Q/ Case of girl 21 y.o with syncope and sign starting from 5
Pic of ECG

Q/ Blood supply to AV and AS node in the heart ?

Q/ How Aspirin cause hyperthermia ?


-Increase metabolism
-The metabolisation of aspirin in the body
- etc.

Q/ Many cases about Anemia!


Mode of action
3 ECG

Q/ Women hade insect bite then develop erythema and LN


enlargement ( pic of forearm with liner erythema )
Lab value = increase WBC
What your Dx ?

Q/ Man just married got pigmentation on his hand he said that his
cuisine had similar condition What the chances his doter become a
carrier ?
-1:2
-2:4
-1:3

Q/ Pt had accident developed multi organ failure , now he has


increase serum phosphate caused by which organ failure ?
Liver
Kidney
Hear

Q/ Child + Coca Cola urine color

Q/ Steroid (or aspirin not sure )syndrome effect which age ?


1 _ 12 m
12 _ 24m
24 _ 96 m

October 24, 2017 7


Q/ Old male present with mid clavicle mass ( smoker for 20y and i
think he drinks )
What is your initial management
-Core biopsy
-Fine needle biopsy
-bronchoscopy

16/8
around 100 question in the link below was screenshotted from
WhatsApp conversation

https://www.dropbox.com/sh/dd5nbp79sainqz5/AAAe4-
SKFZ_wT6fEdilZ0cmAa?dl=0

23/8

Q/ Mid diastolic murmur in left sternum ??


- Mitral stenosis

Q/ question about Right bundle branch block

Q/ milestone of baby can hold his head and when he looks at his
flying hand he laughs ??
- 4 months

Q/ question describe hernia that is irreversible, painful abdomen ??


- Strangulated hernia

Q/ he gave a scenario and asked about sensitivity & specificity ( in


statistics ) you should understand the definition to answer according
to scenario

Q/ scenario of patient used a medication for depression and


developed rash in all of his body ??
- Toxic Epidermal Necrolysis ( TEN )

October 24, 2017 8


Q/ long scenario he wants the equation of relative risk
The answer is a ÷ (a+b) ÷ c ÷ ( c+d )

Q/ he asked a question about statistics I can’t remember it but the


answer was ( 66 :1 )

Q/ scenario about child went to camp and developed fever and


vomiting after coming home ??
- the answer was Brucellosis ( Not Sure )

Q/ about female with foul smelling greenish discharge and asked


about organisms ??
- Trichomoniasis

Q/ scenario of child complaining of ear pain when you examined him


you found bulge in tympanic membrane ??
- Otitis media

Q/ of embryology I can’t remember ??


- Transposition of great vessels

Q/ about child was born pre term and have shortness of breath ??
- Surfactant

Q/ about watery diarrhea without blood and asked abbot organism ??


- I chose C. Defficile ( Not sure )

Q/ what organism can cause paralysis ??


- botolinium

Q/ bacterial sexual like behavior ??


- conjunction

Q/ scenario about female sexually active and came with symptom I


forgot , in lab results he mentioned gram negative diplococci and
asked about the diagnosis ??
- N. Gonorrhea

October 24, 2017 9


Q/ scenario of man came from desert and got cutaneous leshmania
what is the treatment ??
- I forgot the drug name but it was the first choice I think

Q/ how does protein enter the peroxisomes ??


- Folded with the help of C- terminal

Q/ Which of the following made in nucleolus ??


- I think rRNA ( not sure maybe mRNA )

Q/ Protein after transfer sequence what is the last destination ??


- endoplasmic reticulum

Q/ long senario I don’t remember about lipid and cholesterol and


asked about gene ??
- the answer is LDL receptor

Q/ long scenario the summary is child was normal when he was on


breast feeding and now his mother feeds him juice and he developed
symptoms I only remember jaundice ,, what should be avoided ??
- Fructose

Q/ type of bond between tRNA and mRNA ??


- hydrogen bond

Q/ patient have polycystic kidney disease what kind of cancer he will


get ??
Ovarian
Lung
Kidney
Breast

Q/ danlos ehler what is the type of transmission ??


- Autosomal dominant

Q/ I can’t remember but the answer was gene ( 16 & 18 )

Q/ scenario of patient with diabetes ,, and asked about how does


diabetes affect on wound healing ??

October 24, 2017 10


‫‪- limit phagocytosis‬‬

‫‪Q/ I can’t remember the scenario but the answer was Rota Virus‬‬

‫‪Q/ they asked about one or two questions from the pre test questions‬‬
‫‪in the website‬‬

‫‪25/8‬‬

‫‪ /Q‬كيس عن اهلير دانلوس بس ما قاله بالصريح يسال عن المود اوف انهيرتنس‬

‫‪ /Q‬سؤال ايش الميوتيشن اللي يصير في سيكل سيل‬

‫‪4 /Q‬اسئلة بالنص من ملزمة الكوميونيتي‬

‫‪ /Q‬كيس فيها يونيالترال ابدومنال ماس هيماتوريا ابسينت ايريس ? ويلمز تيومر‬

‫‪ /Q‬كيس بايالترال ماس وهيماتوريا ? بوليسيستك كيدني‬

‫‪ /Q‬كيس جاية ب كحة وستول ما يحصله فلش وهيا صغيرة كان عندها ريكرنت هيستوري اوف ابر‬
‫ريسبايراتوري تراكت انفكسن ? سيستك فايبروزيز‬

‫‪ /Q‬فايرال قاستروانترايتس? روتافايرس‬

‫‪2 /Q‬كيس عن مننجيوكوكال انفكشن‬


‫‪ ca125 /Q‬حق ايش? اوفارين كانسر‬

‫‪ /Q‬كيس عن ولد امه هيباتيتس بوزيتف وعمره ‪ 6‬سنين وما اتطعم اي شي غير ‪ bcg‬ايش‬
‫نسويله?نديه تطعيمات من اول وجديد‬

‫‪ 2 /Q‬كيس عن ا نتيجينات الهيباتاتيس واحد يسال عن االنتيجن اللي في الويندو والتاني يسال ايش‬
‫حيكون موجود في حالة كرونك انفكشن‬

‫‪October 24, 2017‬‬ ‫‪11‬‬


‫‪ /Q‬سؤال موست سبيسيفك تيست فور سيلفلس‪? FTA‬‬

‫‪ /Q‬سؤالين عن ستيج اوف كانسر واحد كان كاتب ستيج ‪ B2‬كولون كانسر عبارة عن ايش‪ ..‬والتاني‬

‫‪ /Q‬سؤال كل كم شهر نقيس جولوكوز للديابيتبك? كل ‪3‬‬

‫‪ /Q‬سؤال هايبرتنسف وجاي في تشينج في اوبتك ديسك ايش النيكست ستيب في المانجمنت? ريفيرال‬
‫لالوفثا وال نعالجها‬

‫‪ /Q‬واحد جاي عنده ديابيتك السر نون هيلنق عليها سؤالين واحد يسال كيف الديابيتز يضعف االميونيتي‬
‫والتاني ايش المانجمنت‬

‫‪ /Q‬سؤال وحدة حامل صارلها بروم بعدين صارلها بين وفيفر (كانه بيقول كوريوامنيوناتس) ايش‬
‫نيكست ستيب المانجمنت? نولدها قيصري وال اي في انتي بيوتك‬

‫‪ /Q‬كيس عنده هايبركوليستريميا وفي التالتين ايش الجين ‪? LDL‬وال ‪Apo100‬‬

‫‪ /Q‬كيس جايبين صورة راش في البتوك ايش الدياقنوزيز? هينقشالين‬

‫‪ /Q‬سؤال كواركتيشن اوف اورطا يجي مع ايش? تيرنر‬

‫‪ /Q‬سؤال هايبرتنسف ابر اكستريميتي وابسنت بلس لور اكستريمتي دياقنوزير? كواركتيشة اوف اورتا‬

‫‪ /Q‬كيس بيبي كان طبيعي لمن بدأ يشرب فورموال صار عنده جونديس الزم نبعده عن ايش? جاالكتوز‬
‫او الكتوز مني فاكرة ايش الخيار‬

‫‪/Q‬ايش االيميديت كوز اوف ديث في البيرن? انهليشن انجوري‬

‫‪ /Q‬كيس واحد عنده تيبيكال بانكرياتاتيس سيمبتوز والالب ريزلت طالع كمان انه في ‪GGT / ALT‬‬
‫عاليين ايش سبب البانكرياتيتس? الكحولول وال ستون? انا اخترت ستون عشان الالب‬

‫‪ /Q‬سؤال عن ال ‪ ADHD‬وسؤال عن ال ‪Autism‬‬

‫‪ /Q‬سؤال عن ديسمونريا جاية ب بريست تندرنس ومود سيمبتومز عمرها ‪ 09‬ايش اديها?‬
‫بروجيسترون ميني بيلز وال ?‪ SSRI‬انا اخترت ‪ SSRI‬عشان البريست والمود‬

‫‪/Q‬سؤال عن اسم الفياقرا العلمي المغروض ما ياخد اية دوا? نايتريت‬

‫‪October 24, 2017‬‬ ‫‪12‬‬


‫‪ /Q‬سؤال كان تيبيكال برزنتيشن للسكيزةفرنيا بس انه االعراض بدات من شهرين ف ايش الدياقنوزير‬
‫اللي بدالها? سكيزوفرينيفورم‬

‫‪ 4 /Q‬كيسات نيفروتيك تتراوح من دياقنوزيز لمانجمنت النفستقيشن‬

‫‪/Q‬صورة اي سي جي (حقيقة انا شفتها تايب ‪ 4‬سكند ديقري هارت بلوك) وكان يسال عن الدياقنوزيز‬

‫‪ /Q‬سؤال اي سي جي ديب ‪ s‬وبعدين لونق ‪( R‬مني فاكرة الليدز) وحاطط خيارات اخترت ليفت بندل‬
‫برانش بلوك‬

‫‪ /Q‬سؤال اي سي جي حاطط ليدز وهوا جاي ب ‪ MI‬ويسال عن مكانها‬

‫‪ /Q‬سؤال صورة جفن سفلي طالع ع برة ايش هوا? اكتروبيون‬

‫‪ /Q‬سؤالين عن ليكن بالنس واحد فين اكتر مكان يجي والتاني ايش البروقنوزيز حقه هل يختفي وال‬
‫يتحول ماليقننسي وال يروح ويرجع‬

‫‪ /Q‬سؤال بيبي جاي ب مايلد ديهايدريشن وفومتنق ودياريا ايش نسويله ‪? ORS‬بس وال ندي معاه‬
‫انتي ايمتك( انا اخترت ندي معاه انتب ايمتك النه بيستفرغ)‬
‫‪ /Q‬سؤال واحد عنده ايرون ديفشنسي انيميا وكرونيك قايترايتس كيف نهالج االنيميا? اخترت ‪IM‬‬
‫ايرون‬

‫‪24/8‬‬

‫‪Q/ Orthopedic case of 13 yrs old boy felt down and got traumatized‬‬
‫‪came e forearm swelling and pain X ray pic provided Something‬‬
‫‪special on x ray that is the growth plates are widely separated‬‬

‫✔‪Answer was Salter Harris fracture‬‬


‫‪Q/ which of the following is at higher risk of colorectal carcinoma‬‬
‫‪Multiple choices ( one of them was male e high fat diet and "Gardner‬‬
‫‪syndrome" ---> polyposis coli. I think it was the closest answer. other‬‬
‫‪choices include high salt diet and smoker ..‬‬

‫‪October 24, 2017‬‬ ‫‪13‬‬


Q/ what is true about Pap smear
12 till 65 yrs repeated every year
12 till 65 yrs repeated every 5 yrs
12 till 35 repeated every 3 years
03 till 65 repeated every 5 yrs ✔

Q/ A pt with Lab findings of hypothyroidism, he has hoarseness And


a large neck swelling OE lt lobe is swollen and is larger than the right
one Best Mx is
2/FNA
1/Thyroid lobectomy ,, I chose this as the pt has pressure sx : on recurrent
laryngeal n ==> hoarseness of voice
0/ Radio isotope scan

Q/ 23 yrs old pt e a hx of painful knee swelling plus dysuria


Urine culture was negative Other details I could not recall
RA
Reiter’s syndrome
Others
I chose reiters syndrome 😅(can't see, can't pee, can't climb a
tree)=(uveitis, urethritis and arthritis)

Q/ female has genital warts over the past years, genital warts are
associated with what ?
====> hyperkeratosis ✔

Q/ greenish foul smelling vaginal discharge, oE/ cervix e dots and


erythema(the q provided the description of strawberry cx)
===> trichomoniasis ✔

Q/ a child e episodes of cyanosis in lips and peripheries + precordial


murmur
What is best to reach a diagnosis
CXR
ECG

October 24, 2017 14


echocardiography ✔ (TOF)

Q/ One of the following is seen in bacterial vaginosis


No itching or burning
Yellow discharge on Vaginal PH paper
Greenish discharge

Q/ young female with fishy odor vaginal discharge


And positive wiff test
B vaginosis✔
Trichomoniasis
Candidiasis
Chlamydia infection

Q/ pediatrics cases ( rubella infection "fever plus rash started on the


face and spread into the trunk and LL)

Q/ ( HSP plus a picture of macular rash in buttocks and LL , the


question asked about the drug given in HSP)

Q/ (TTP case " pt with fever, altered LOC for 5 days, his condition
deteriorated and developed body rash, and became jaundiced, lab
tests showed low Hb, low PLT)

Q/ infant with abdominal colicks, vomiting and bloody stool, other


data provided, diagnosis is intussusception, the question asked
about the 1st step in Mx "
2/iv fluids and bowel rest✔
1/ CT abdomen
0/surgery
4/barium enema

Q/ milestone of a child at hospital laughing and cooing, when the


doctor approaches him, he felt afraid and was looking for his mother
===> 6 months✔

October 24, 2017 15


Q/ 16 yrs old drug abuser, what invx should be done to him
2/ test for HIV, I chose this
1/test of HBV

Q/ absent red eye reflex is seen in ===> retinoblastoma✔

Q/ 4 to five cases about bacterial and viral conjunctivitis

Q/ pediatric e purulent eye discharge


Culture showed gram negative diplococci
How to treat
2/iv cephalosporin✔
1/steroid
0/topical Abx

Q/ a female came to gynae clinic with post coital bleeding,


Source of bleeding is
2/cervix✔
1/uterus
0/vulva
4/vagina

Q/ food poisoning case, 4 family members ate from a restaurant, they


developed diarrhea and vomiting and remit after 24 hrs
Culture showed gram positive bacilli
2/salmonella
1/shigella
0/ SA
4/Bacillus ceres ✔

Q/ couples are said to be infertile after marriage and should seek for
fertility After
0 /2 M
6 /1 M
9 /0 M

October 24, 2017 16


21 /4 M ✔

Q/ young couples trying to conceive for 6 months, both are healthy


=====> try some more.✔

Q/ 3 cases about septic Arthritis;


( one about diagnosis "they gave typical presentation with positive
culture more than 50000 WBC and more than 75 % PMN)
( case 2 septic arthritis with a culture of gram positive cocci in
clusters, pt was given cloxacillin and no improvement after 5 days,
What to do ? ======> start vancomycin✔

Q/ a child diagnosed as serous OM, best to relieve his symptoms is


===> Myringotomy✔

Q/ 38 wks, multiparous, with a trial of ECV on week 36, her amniotic


fluid index is 12, now her fetus is in lateral position, her membranes
ruptured with clear fluids, What would be the indication for CS?
2/ repeated ECV
1/ amniotic Fluid index
0/ ROM
4/ fetal position✔

Q/ A woman in labor, everything is fine, she asked for Epidural


analgesia, what test should be requested,
2/Hb
1/PLT count ✔

Q/ CSF circulates in which space?


2/epidural
1/subdural
0/Subarachnoid✔

October 24, 2017 17


Q/ drug given with analgesia to decrease its side effect
===> metoclopromide✔

Q/ a child with cutaneous hemangioma, those hemangiomas could


be found in which organ?
2/liver
1/spleen
0/kidneys

Q/ typical scenario of myasthenia Gravis( fatigability and weakness


worsen at the end of the day, What is the pathophysiology;
2/ viral
1/bacteria
0/drug induced
4/ Autoimmune ✔

Q/ another scenario about Myasthenia graves,


Part affected;
( ====> Acetylcholine receptors✔

Q/ pt heard about incretin on TV and wishes to know more about its


action,

===> increases insulin secretion✔

Q/ infection associated with dog bites =====> polymicrobial ✔

Q/ A pt with depression on medications found comatose with empty


pill bottle beside her On invx; ABG showed Metabolic acidosis, what
drug overdosed?
Aspirin ✔
SSRI

Q/ description of Herpes simplex genital lesions

====> multiple painful fluid filled vesicles✔

October 24, 2017 18


Q/ pt on sulfa drugs developed rashes, best description is,
====> skin blisters✔

Q/ clear scenario about depression( low mood lack of sleep,,,, what


is deficient in this patient?

=====> serotonin ✔

Q/ a pt with multiple fractures, OE sky blue sclera, X ray showed


multiple healing fractures with callus formation

===> Osteogenesis imperfecta✔

Q/ pt with vitiligo on medications for 3 yrs no response, she


requested a surgical TX What would it be,

====> Melanocytes transfer✔

Q/ a pt with heart failure, on ACEI furosemide spironolactone and


other drugs, Now stable and clinically free, what drug modification is
appropriate ?!
====> stop spironolactone✔

Q/ 13 yrs old pt with salmonella infection Resistant to


chloramphenicol, appropriate Tx is
2/ continue chloramphenicol
1/ add cipro
0/give cipro alone
4/ IM ceftriaxone

Q/ 3 yrs. old baby with fever, neck rigidity, culture revealed


diplocci/Gm-ve
(They provided a picture of diffuse meningiococcemia skin rash
And asked about what to give to his Family

======> Oral rifampicin✔

October 24, 2017 19


Q/ Necrotizing fasciitis best Abx regime is
===> pipracillin and tazobactam✔

Q/ best way to diagnose bacterial vaginosis

====> gram stain ✔

Q/ 2Q were so clear about Syphilis👍


( painless Genital ulcer plus inguinal lymphadenopathy, )
( a pt. with painless Genital ulcer, how to diagnose?

===> dark field microscopy✔

Q/ a child brought to ER with barking cough, red epiglottis, thumb


sign on x ray, Best initial management?
2/ examination of epiglottis
1/ Endotracheal intubation.
0/Emergent tracheostomy(I chose this)
4/ nasopharyngeal Tube

Q/ a pt. came for ophthalmology check up, has optic disc cupping,
Tonometry showed high IOP What would u tell this pt.
2/ tonometry is sufficient
1/do check up for blood related members
0/interventions may reverse these changes

Q/ a child with diarrhea, abdominal bloating, diagnosed with


giardiasis, What is the most sensitive test to diagnose his case, ?
0 /2 consecutive stool analysis
0 /1 separate stool analysis
4/ stool immunoassay ( I chose this)

Q/ a child from Ghana with a neck mass,


Biopsy showed starry sky appearance, KI 67 was positive,
Which genetic change is true?
===> over expression of C myc gene ✔

October 24, 2017 20


Q/ a child developed fever headache and malaise, after traveling to a
southern area on a trip with his friends,
His mother thought that it is normal after this tired trip, his condition
worsen and he developed skin rash, sore throat, lymphadenopathy,
and hepatosplenomegaly on examination, what test to order and
reach a diagnosis?
====> monospot test ✔(infectious Mononucleosus)

Q/ diabetic pt with uncontrolled Blood sugar, presented with an ulcer


on sole of foot surrounded by erythema, what is the best action?
2/oral Abx and discharge the pt
1/ reassurance
3/
4/ admission for debridement and deep tissue culture✔

Q/ what determines the prognosis in CLL;


2/stage of the disease.
1/age at the time of diagnosis
0/Bone marrow aspiration

Q/ most specific sign for perforated duodenal ulcer,


====> sudden epigastric pain✔

Q/ 56 yrs old woman with uncontrolled urination upon coughing,


sneezing or laughing, Best Mx measure for her is
===> Kegel Exercise✔

Q/ 23 yrs old female, with burning upon voiding urine, she goes
frequently to bathroom but with little amount of urine voided, she
does not have suprapubic tenderness OE ( Im not sure if urine
analysis was done or not !? Anyway the question asked for
What is suitable?
2/urine culture
2/
3/
4/ testing for Gonorrhea/chlamydia

October 24, 2017 21


Q/ a pediatrics question ,what is most common of the following (
congenital or something else, I can't really remember)
2/ micropenis
1/ hypospadias
0/cryptorchidisim.

Q/ a condition in which circumcision is C/I


2/ phimosis
1/ hypospadias
... read about the C/I

Q/ best treatment plan for Infective endocarditis,


2/Multiple Abx high dose for a short time
1/Multiple Abx extended over a long period of time
... other choices,
Please read about it

Q/ a pt with muscle weakness, apart from hypotension and


bradycardia, his examination was normal,
His lab tests ( high K+, low NA, low Chloride, high urea)
What is the etiology behind his presentation?
2/ hyponatremia
1/hyperkalemia
0/low chloride
4/uremia

Q/ a child who has episodes of vomiting after feeds, although his


appetite is good and wants to eat despite vomiting,
His labs, hypokalemia and low chloride,
Most likely diagnosis is
====> Hypertrophic pyloric stenosis✔

Q/ Which tendon passes through lesser sciatic foramen?

====> Obturator Internus tendon✔

October 24, 2017 22


Q/ squash player, e elbow pain?
=====> lateral Epicondylitis✔

Q/ a child with different types of infections including asprigelluos,


Tb, staph ... etc
Immune defect in =====> NADPH oxidase ✔(chronic granulamatous
disease)

Q/ old female e back pain, relieved e leaning forward and walking


uphill, Peripheral pulses are intact,

===> lumbar spinal stenosis ✔

Q/ pt after a surgery in the posterior triangle of neck, developed loss


of sensation on the skin of lower mandible and Lower part of the ear,
Nerve injured is
===> Great Auricular nerve.

Q/ clear case of Carpal tunnel syndrome, the pt. is a typist working


on computer, nerve affected

===> Median nerve✔

Q/ ophthalmology ex for DM2 patient every


6/2 months
21 /1 months✔
14/0 months
06/4 Months

Q/ most common type of fibroid


2/ submucous
1/ intramural( I chose this)
0/subserous

October 24, 2017 23


Q/ During a surgery in the diaphragm, Phrenic nerve is dissected at
which level?
===> anterior to sclaneous anterior muscle✔

Q/ mechanism of action of Oxybutin( a drug prescribed for urge


incontinance)
====> blocks muscarinic cholinergic receptors.✔

Q/ insulin resistance in type 2 DM, how does it occur


(Pathophysiology)

Q/ gene responsible for apoptosis? ==>p53 ✔

Q/ a pt with PCOS is at risk of which cancer ?


2/cervical
1/endometrial ✔
0/ ovarian
4/breast

Q/ non athlete forcefully planter flexed his foot and heard a click
sound on his leg , What tendon affected?

===>calceneal tendon ✔

Q/ difference bt Delirium and dementia


==> state of consciousness✔

Q/ a 23 yrs old pt has hallucinations, delusions, for 1 month, the


doctor diagnosed him as a case of schizophrenia,
Which of the following is against the diagnosis of schizophrenia?
2/ age of the pt.
1/hallucinations
0/ delusions
4/ duration of symptoms ✔

October 24, 2017 24


Q/ a pt. presented to psychiatry department, the pt. says that some
people know what he is thinking about and his thoughts are known
by others as if distributed,
This type of thought is?
====> thought broadcasting✔

Q/ Lower lip lesion. Lymphatic drainage

Q/ Stab wound at post triangle of neck. Nerve damage? Unable to


raise arm.

Q/ surgery of lower esophageal junction post vagus trunk damage


what Organ effected. Rectum, bladder ,
colon, oesophagus.in options.

Q/ blood supply of post compartment of leg.

Q/ nerve damage effecting lowe law and post auricle which nerve.?

Q/ A patient known diabetic ; whole scenario about blood sugar


control but in the last patient taken out one tablet from pocket and
asking mode of action of that drug. tab Acretin.

Q/ case of Gout given mechanism of action of different drugs and


asked to choose.

Q/ polycystic kidney disease pathology. Inheritance.

Q/ young patient with BP 160/110 high creatinine, ++protein urea.


Drug of choice for HTN.

October 24, 2017 25


Q/ picture of ring cell stage of malaria. Asked the stage and malaria
type.

Q/ picture of histopath of tumor with stary sky pattern. EBV👈

Q/ sickle cell patient with body pain . Option will u admit or refer to
tertiary care. (Only UQUquestion. )

Q/ case of Wegener’s but asked about the basic pathology of


disease.

Q/ patient in labor 6 cm dilatation All fine .fetal CTG accelerations and


variable? Next step?

Q/ middle age women trying to conceive last born 1 year all normal
what you check. FSH👈

Q/ first trimester .with H/O GDM in last pregnancy 3 yrs back.


When you do GTT. 16 wk.👈

Q/ 4p year old with heavy bleeding everything excluded. Even D&R


normal. Cause of bleeding? Anoulatory cycles.👈

Q/ mushroom poisoning case . Pathophysiology of reaction.


Polymerase some thing.SMLE 13 question.

Q/ two x-ray fractures upper and lower limb. Management.?

Q/ case of chest pain old man with transient loss of


consciousness. 3- 4 episodes. Very long bla bla.and X ray

October 24, 2017 26


chest given. Asked about management.??? Want above my head

Q/ Saudi ministry of health postpone hajj for some people in 2015.d/ t


MERS ::: Hep B. ::Elderly DM.:: lactating mother.?

Q/ man with erectile dysfunction with normal morning erections.


Where to refer. Pshyaiatry👈 cardio, neurology :

Q/ dizygotic twins asked about type of chorion?

Q/ bipolar disorder develop symptoms of hypothyroidism. Which


medicine responsible.

Q/ knee pain old age with NSAID 3 weeks. Come with bloody
vomiting. Reason?

Q/ bone tumour speeding in pelvic bone??? From SLE 13.

Q/ Very poor Apgar score with cyanosis. Your action?

Q/ 2 question from statistics. dont remember.

28/8

Q/ a pic of face, with extremely alert looks wide open eyes and
symptoms looked like graves .scenario saying he has palpitations
sweats tachypnea
a..graves’ disease my answer
b..some syndrome
c De georges syndrome??
d hypothyroidism

October 24, 2017 27


Q/ other question was there was a patent age 60 he has
hyperthyroidism he wants to know what long term complications he ll
have....
ct of brain my answer
liver ct
bone scan I selected CT of brain check about it and see if its correct

Q/ a diabetic patient come to you and you have to check survival of


something i don't remember this question completely
0.65
0.89
0.11
0.95 my answer
values were approx. same like this

Q/ patient comes to you with drooling saliva constricted pupil same


scenario almost as in notes what to do
IV atropine
lavage and other options but correct is atropine

Q/ same case which we discussed so many times about chronic renal


failure that came

for optha I don't remember the question it was weird but options were
physiological myopia
pathological myopia
curved cornea like that

Q/ patient was treated for renal stone before ....they attached picture
of X-ray which I did not get he had to go for some surgery so what
will you do
-send him to vascular surgeon for follow up
clear him for surgery
sorry these new questions are difficult for me to remember

Q/ 20 years old student came to you lethargic change of sense of


time blood test normal and waiting for urine test ...what is the cause
caffeine
cannabis my answer

October 24, 2017 28


cocaine
opioid

Q/ a lady taking phenytoin she at age 29 she got seizures now she is
35 and better but she has more hair on her face as side effect of
phenytoin so what will you do
-continue phenytoin
-change to other anticonvulsants
-she can now stop it my answer

Q/ 2 questions about varicella vaccine as we did in um al qura 2 doses


6 weeks apart ..... 2 doses 4 weeks apart these

Q/ case of acoustic neuroma

Q/ first line treatment of depression


ssri my answer
tca
mao

Q/ patient after trauma have chest pain and difficulty breathing next
step
chest tube
thoracotomy my answer
needle aspiration
central line

Q/ patient has symptoms of mania and I forgot the options :(

Q/ patient his family member died from 3 weeks he has hallucinations


and acting lost.
schizophrenia
brief psychotic disorder my answer
schizoaffective disorder

Q/ patient has dysuria frequency and little pain rest all normal what is
next step
ciplofloxacillin for and check after 2 days my answer
amoxicillin or penicillin for 2 weeks

October 24, 2017 29


Q/ a drug which stops sweating ....no option as hydrosamine
cimtidine my answer
odensteron
and two more which were ending with chloride

Q/ nulliparous came to you age 53 he mother got breast cancer when


she was 38 what is the most important thing to check for her
abnormal vaginal bleeding my answer
high cholesterol lever
some mutation in endometrium

Q/ typical case of appendicitis


Q/ typical case of pancreatitis

Q/ man taking diuretics have muscle weakness diarrhea irritability


hypokalemia
hyperkalemia my answer
hyponatermia
hypocalcemia

Q/ how to calculate titration of pain medicine when giving to patient


age, sex , gender
body weight
get maximal effect with less dosage
4th I forgot just read about titration of pain medicine

Q/ child has edematous nasal what to do?


corticosteroids

Q/ after head injury child came to you has swelling in nose what to do
incision drainage
decongestants and nasal drops my answer

Q/ child born from a mother who has gram negative streptococcus


positive at time of delivery ....after birth child has retraction cyanosis
difficulty breathing something that next action
X-ray of chest
blood culture and look for infection
2 antibiotics names were given I selected X-ray

October 24, 2017 30


Q/ child has spot on teeth what supplement to add...
fluoride

Q/ 3 cases about Allergic rhinitis,


All were clear , 2 of them asking for the diagnosis
And one asking for the treatment,( oE, pale oedematous mass)====>
treat e corticosteroids✔

Q/ 18 yrs old female, e hx of amenorrhea for 6months, low BMI, but


she thinks that she is fat and has to lose weight. OE ( increased hair
distribution in her body plus other findings)
What is the diagnosis?
====>Anorexia Nervosa✔

Q/ a teenager came e abdominal pain after a basketball game, on


Examination , there's periumbilical tenderness.
Best action. ?
1/ CT abdomen
2/abdominal US
3/erect chest x ray
4/ re examine after 24 hours.✔

Q/ clear scenario about Crohn's disease (mentioned that she has


post meal periumbilical pain plus other symptoms

==> Crohn's disease ✔

Q/ which of the following breaks the thrombus in MI


1/aspirin
2/warfarin
3/heparin
4/streptokinase✔

Q/ a pt has an upper abdominal pain,


( gastrin and pancreatic enzymes released, and asking about the
diagnosis)

October 24, 2017 31


===> a scenario of zollinger elson syndrome✔

Q/ a child, known case of thalassemia, Had fever after blood


transfusion, what is the best measure to take for prevention of fever,
1/ antipyretic
2/ pretransfusion medications
3/ leukocyte depleted (prior storage)
4/ leukocyte filters at transfusion session.

Q/ clear case of Hypochondriasis

Q/ a pouch was found, containing gastric and pancreatic tissues,


was diagnosed as mickles diverticulum, which part of intestine would
it be at
1/ duodenum
2/jejunum
3/ ileum✔
4/ cecum

Q/ 3 to 4 scenarios about TB (all were direct and clear)


Q/ PPD positive x ray negative, next===> isoniazid for 6 months✔
Q/ A pt on anti TB drugs for 4 weeks developed needle
sensations====> isoniazid ✔

Q/ A pt on anti TB drugs developed eye pain

====> Ethambutol✔

Q/ The effect of anti TB drugs on the eye is one of the following, (


bacterial conjunctivitis, viral conjunctivitis , glucoma , uveitis

====> correct answer is uveitis ✔

Q/ pt on chemotherapy developed iv line sepsis,


( bacterial sepsis) most likely organism

October 24, 2017 32


Closest option was pseudomonas ✔

Q/ a pt end stage liver disease on Central venous catheter developed


sepsis Culture showed budding yeast
Suitable Rx is
1/fluconazole
2/ antiviral
3/Abx
4/ cuspofungin ✔

Q/ clear scenario about DIC

===> pt started to bleed from iv line and other orifices ✔

Q/ clear case of pericardial tamponade( Hypotension, distended neck


veins and muffled heart sounds)

Q/ RTA victim e closed head injury and LOC


What is the best thing to do ?
1/ intubation and ventilation
2/ check the pulses
3/check the pupils
4/ check the airway✔

Q/ a pt was brought to ER comatose, given naloxone, regain his


consciousness, what substance was abused?
Multiple choices, answer is ====>opiates✔

Q/ a pt e solid thyroid nodule 2 cm size, euthyroid, next ?


====> FNA ✔

Q/ an athlete came with xanthametaous lesion on his Achilles'


tendon, otherwise normal, Genetic defect on which of the following
====>LDL receptors ✔

October 24, 2017 33


Q/ Multiparous woman everything was fine, exept the FHR dropped
from 140 to 80, also mentioned that her platelet count is low, what
type of anesthesia to give? ===>GA ✔

Q/ a pregnant lady was provided a pudendal nerve block as an


analgesia, Which structure would be fully sensitive and not blocked
by the analgesia,
===> Rectum ✔

Q/ a physician wants to decrease the recurrence of UTI,


Best measure is ? Multiple choices, and the correct answer is

===> (decrease PH , increase urea and increase urine osmolarity)✔

Q/ an RTA victim was brought to hospital with multiple injuries,


transferred to ICU. There, he developed multi organ failure, The
phosphate level in his blood is raised , what is the cause of his multi
organ failure?
1/heart
2/lungs
3/kidneys ( I chose this)
4/liver

Q/ female, K/c of epilepsy on phenytoin, she has been using it for a


long time, she came e a complaint of hairy growth in her body, and
stated that she is symptom free for about 6 months. What is the
correct action ?
1/stop phenytoin
2/lower the dose
3/shift to another antiepileptic ( I chose this)
4/continue the drug.

Q/ the transmission of maternal antibodies to the fetus in pregnancy


is a way of
1/active artificial Immunity
2/passive artificial immunity
3/passive natural immunity✔

October 24, 2017 34


4/active natural immunity

Q/ multiparous woman term, came with Progressive contractions and


cervical dilatation, mentioned that her membranes ruptured and fluid
was clear
( details of examination plus " uterine tenderness") correct action ?
Multiple choices and one of them was
===> give intrapartum abX ✔
Because this is a case of chorioamnionitis with possible GBS infection so
treated e intrapartum antibiotics👍

Q/ clear case of atopic pt e allergic rhinitis, symptoms and signs


were given, then asked about the cells responsible for his allergy
1/monocytes
2/macrophages
3/mast cells✔ (histamine)
4/Neutrophils

Q/ RTA victim, hypotensive, tachycardia, tachypnea, activation of


baroreceptors will result in which of the following ?
(multiple choices. One of them was HR ✔

Q/ 35 or 40 yrs old multiparous e hx of CS


Came e vaginal bleeding, which was painless, she is 38 weeks, US
examination showed low lying placenta , which of the following would
make the doctor decide CS with hysterectomy?
1/age of pt
2/ Previous CS
3/ placenta accreta ✔
4/ multiparty

Q/ lymphatic drainage of testes


===> para aortic LN ✔

October 24, 2017 35


Q/ a woman e Ca cervix, e mets beyond uterus and cervix,
1st group of LN receiving the lymphatic drainage is
1/ uterine
2/ external iliac
3/ common iliac
4/ para aortic

Q/ flouroquinolones mode of action,


===> DNA gyrase inhibitors ✔

Q/ cellulitis in the face of infants between 6 and 24 months with


purple discoloration is caused by:
1/ GAS
2/SA
3/hib✔
4/strep pneumonia

Q/ which cardiac lesion is least associated with infective


endocarditis
1/ TOF
2/VSD
3/ Truncus arteriosus
4/ASD ✔

Q/ post cholecystectomy pt, developed unilateral parotid swelling,


saliva was cloudy, culture was negative, What does he have?
1/sarcoid granuloma.
2/ bacterial sialadenitis✔
3/sarcoma
4/sojgrens syndrome

Q/ a case about sarcoidosis, asking for the diagnosis.

October 24, 2017 36


Q/ how to differentiate a huge ovarian mass from an ascites?

===> anterior dullness and lateral Tympani ✔

Q/ 2 cases about iron toxicity. ( best way to decontaminate the


stomach of a pt e iron poisoning, there were these choices, " gastric
lavage, syrup epicac , something called tacren I'm not sure about it
and last option was induced vomiting, I chose Gastric lavage, there was
no activated charcoal in the choices.

Case2 iron poisoning, a child swolloed a bottle filled iron pills,


presented to ER e abdominal pain, nausea and confusion,
Best way to manage is ?
Multiple choices and the best answer was IV deferoxamine✔

Q/ An athlete with itchy pink rash between his thighs What to


prescribe?
1/ topical Abx
2/topical steroid
3/topical antifungal, selenium sulfide ✔( tinea cruris)

Q/ Which muscle would be completely paralyzed by obturator nerve


injury
1/gluteus Maximus
2/adductor Magnus
3/adductor longus ✔

Q/ a woman used IUD for contraception, now has vaginal pain and
discharge, What organism responsible?
===> IUD causing PID by actinomyces israelii✔

Q/ Typical scenario of Acute appendicitis, (periumbilical pain


radiating to RIF, pt has anorexia, nausea and fever, what is the
pathophysiology ? ===>Appendiceal obstruction✔

October 24, 2017 37


Q/ A pt drank an ethylene glycol containing product, what renal
damage is suspected?
===>Acute tubular necrosis (ATN) ✔

Q/ best and rapid management for specific anxiety?


A. Benzodiazipines✔
B. Sertraline
C. Imipramine
D. Bupropion

Q/ Best drug for short term Mx of GAD without causing dependence


or addiction;
1/alprazolam✔
2/sertraline
3/flouxetine
4/Bupropion

Q/ A child always kicking his mother, shouting at her, not responding


to her commands at all, he was separated from her after she was
divorced, what is the problem here?
1/depersonalization
2/derealization
3/anxiety from separation✔
(‫ التصرف‬temper tantrums ‫ السبب‬separation anxiety) ✔

Q/ Female, known case of schizophrenia, came to ER with


suspiciousness, upon examination, she was staring at a person and
saying, *you can't kill me* , what does she have ?
I think choices were
*Derealisation
*Concrete thinking
Could be hallucinations but I'm not sure if it was included in the choices.

Q/ An elderly, known case of Alzheimer's disease, developed


hallucinations, bizarre behaviors and became aggressive, what drug
to add?
1/Haloperidol

October 24, 2017 38


2/Resperidone ✔
3/Chloropromazine
4/Amytreptline

Q/ What is the medication given to reduce the side effects of


analgesics
=====> Metoclopromide

Q/ What anesthetic drug can act as analgesic if given in a low dose?


====> Ketamine

Q/ An elderly with B. Asthma, best drug for induction is


===>Ketamine

Q/ Side effect of morphine is


===> nausea and vomiting

Q/ Which anesthetic agent is 100 times stronger than Morphine?


===>Fentanyl

Q/ Investigation to be ordered before giving epidural analgesia ?


===> Platelet count

Q/ Multiparous, 38 wks, in labor, 90% cervical effacement, 4CM


dilatation, membranes ruptured e clear fluid, On CTG, FHR dropped
from 140 to 80, the pt. platelets=50,000, what type of anesthesia is
suitable?
===>GA

Q/ In order to increase hyperbaric in cephalic position


====> trendelenburg position

October 24, 2017 39


Q/ Mother with Rh -ve and a father with Rh +ve, what the propility of
having Rh +ve child?
- 0%
-25%
-50%
-75%
-100%

Q/ Child with SCA pain in his penis. On Physical Exam you saw this
sign. What is the diagnosis?
- Paraphimosis
- Papisim
- Pyroni's disease
‫ الجواب‬Paraphimosis

Q/ Marfan syndrome features of the mother and her boy: what is the
probability of the children to have it?
25%
50%
75%
100%

Q/ CF gene is located at :
Chromosome 7, long q arm

Q/ Regarding distichiasis definition ? ( multiple defintions and u choose


the most accurate)

Q/ In menopausal lady, diagnostic test


( multiple hormones I chose FSH)

Q/ Congential adrenal hyperplasia test?

Q/ Type of postpartum hemorrhage in atony of uterus? ( primary)

October 24, 2017 40


Q/ Obseesive complussive disorder, what is the mechanism of action
of the drug used to treat it ( SSRI) ?

Q/ A mother gave her baby a juice then he started to cry, what type of
intolerance?

Q/ A question about mothers vs cow's milk comparsion? ( iron is more


in cows milk)

Q/ Infant 8 months with mengitis, what is the causative organism?


(s.pneumonia)

Q/ Regarding preeclampsia, first symptom/ sign ?

Q/ Phyenytoin side effect? Gum hyperplasia

Q/ A child with Quadriplasia, found uncouncious, what to do ? give IM


glucagon

Q/ In hypovolemic shock, which one of the parameters will decrease?

Q/ In open angle glaucoma, which drug is contraindicated

Q/ In panic disorder, how to manage?

Q/ Otitis media with effusion in a 2 yrs baby? ( myrigntomy or gamma


tube)

Q/ Antidose of choice in papracetamol toxicity? n-acetylcysteine

October 24, 2017 41


Q/ Whats the most effective way to disseminate health eduction ?
mass media

Q/ A question about meningitis prophylaxis for a child


( rifampicin)

Q/ The method that has the greastest potential to decreased health


problems?

Q/ Greenish vaginal discharge with red cervix?


Trichomatis vaginalis

Q/ Stress incontinence treatment?


(kegel exercise)

Q/ Cells that secret melanin PIGMENT? somatorophs

Q/ Pt in ER, hypotensive, tachycardiac and hypercabnia, ECG showed


arrhythmia, immediate action ?( needle
decomop/pericardiocentesis/US/thoracotomy)

Q/ Most common cause to perform hysterectomy? (uterine fibroid )

Q/ A clear case about firboadenoma,

Q/ A cut wound in Superficial layer in sole of fooT , which structres


are affected? (, abductor hallucios)

October 24, 2017 42


Q/ After long time of running, a pt had pain anterior to heel?(planter
fascitits)

Q/ MERS-CoA , RELATIVE risk? 66:1

Q/ Relative risk RR is defined by ( A/A+B, C/C+D)

Q/ What type of heart failure occurs with high cardiac output ?

Q/ A physiology question ( not mentined) answed anemia !

Q/ Most common cause of hearing loss ?SNHL/conductive ?

Q/ Pt with bilateral knee pain. By exam there is joint crepitus and


muscle
wasting around knee? ( RA, osteoarthritis)

Q/ Long thoracic nerve root number?

Q/ Antiphospholipid syndrome x2 questions !!

Q/ A pregnant in 5th month had previous 2 abortions due to cervix


incompetence presented now with bleeding, by exam, CVX was open,
whats the cause of bleeding?

Q/ A question about primary postpartum hemorrhage

October 24, 2017 43


Q/ Neonate after delivery, had bleeding, what to give?

Q/ Baby born with 1 artery in umblical cord ?

Q/ Most cc of fibroid?

Q/ Best dx tool for female with irregular massive bleeding

Q/ A female want to conceive after 2 yrs, what best ocs to give?

Q/ Most common urthra injury site

Q/ Most common urthra injury site

Q/ Pelvic injury, what to do ? suprapubic cath

Q/ Herpz simplex in pediatrics?

Q/ What is the tt of sebrrotic dermatitis

Q/ Multiple questins regarding vaccines ( route? Missed dose what to


do?)

Q/ A question about minmal change GN

Q/ Histologic findings in post strep GN

Q/ A q about rhadbdomyolisis

Q/ Athlete man with hematuria with calf muscle pain ?

Q/ Pt w Back pain, but he doesn’t have money, what test to order?


Xray?

October 24, 2017 44


Q/ A female pt had been isolating herself, and thinks people are
chasing her,plus she avoids eye contact ?

Q/ Pt visits psychiatrist and the docnoticed depressive symptoms,


while taking
hx, she was not responding, then stopped visiting,,???

Q/ Your friend avoids you, what type of personality ???? avoidant !

Q/ Stab wound in obturator N. what are miscles affected? (Adductor


longus/magnus ?

Q/ Muscle in lesser formen? Obturator interns ?

Q/ A 5 yr old boy who is shy and can only say mama, baba, his older
brother was like him?

Q/ Anterior axillary lymph node x2 questions !!

Q/ A scenario about teenager, best way to deal with him? And whats
the dx?

Q/ Pt in 40s with obs jaundice, las showed direct bilirubin 40, ferritin
high,
Alkaline phosp v high, AST elevated, what dx? Cystic
fibrosis/gallbladder
stone?

Q/ During fights, what system is activated? Sympathetic ! ☺

Q/ Origin of right atrium

Q/ A female with immunity disorder, where is the defect? IL-2/OR 3


OR 6

Q/ A q about x linked immunodefincy

Q/ A case with symptms of typhoid fever, what is the causative


organism? And ttt?

Q/ Mechanism of action of cipro

October 24, 2017 45


Q/ Organism resistance to cephalosporin, what to do? ( add vaco?

Q/ A case about burn fluid management ?

Q/ Calculate anion gap ☺

Q/ Labs VBG and Type of acid base disturbance?

Q/ Nitrofuratonin side effect?

Q/ Overdosed drug caused resp depression?

Q/ Overdosed lead to comatose?

Q/ Overdosed lead to dilated pupils?

Q/ A truma case with facial fractures, his eyes ( something) what type
of reaction?

Q/ Treatment f necrotizing fascititis

Q/ Food allergy with symptoms of anaphlytic shock..

Q/ Pscych pt swallowed safety pin reached dudenm what to do

Q/ Pt stabbed in abdomen, a small part of omentum is exposed, what


to do

Q/ Gout dx test?

Q/ A question about RA

Q/ X linked is? AD, Multifactorial

Q/ Von willbrand dis? ( prolonged pt, prolonged PTT, factor 12 def )

Q/ VT by ECG + hypotensice, management ? (


amidorone/adenosine/Dc SHOCK

Q/ Most common risk factor in schizophrenia ? both parents have it

Q/ Baby with rash ( pic) measles?

October 24, 2017 46


Q/ Muscles responsible for unlocking the knee? Poplitus

Q/ Unilateral nasal obstruction, with foul smelling discharge and


verstible
erusion, his nasal exam was normal ,..what to do ( x ray head and
chest,/
nasal exam UNDER GA/ABX?)

Q/ First line tx in depression ? SSRI

Q/ Man with aortic stenosis develop syncompe, what is the cause?


Systemic hypotension

Q/ Pregnant lady in 40 wks, suddenly have SOB, hypoxemia, chest


pain? (PE. MI.my answer was amniotic fluid emboli)

Q/ Pt with auditory hallucination, and delucsion for 1 m, she was dx


with schizo..why is this dx wrong? ( duration should be 6 months )

Q/ Women found uncouncious with empty bottle of drug next to


her,she was dx with depression previously, on exam, pupils dilated
and un reactive;; whatsthe drug?

Q/ Questions about (Incidence, relative risk and prevelance )

Q/ Pt on warfarin, presented with melena + prolonged PT. what to do?


( vit k iv/FFP, protamine suphate)

Q/ Pt who is unable to extend his knee, muscles affected? (


sartious/quadriceps/biceps femoris?

Q/ At the OPD, a pt was waiting then collapsed, LOC . the nurse


checked
breathing- ( abscent) what to do ? ( chest compression 30/2 – check
carotid pulse- give 2 breaths)

Q/ Painless ulcer ? syphilis

Q/ A case about end stage liver disease with fungal infection?


(capsufungin)

Q/ Apptosis ( p53)

October 24, 2017 47


Q/Tamoxifen for breast cancer pt, she has metrorrhgia, us showed thick
endometrium.. what to do next ? ( endometrial biopsy, CA125

Q/ Most common s/e of IUD ?

Q/ Mechanism of warfarin ?

Q/ Necrotizing fascititis ttt ?


( impenim and metronidazole/ampilcillin
and gentamicin/ pipercillin and tazobactam/pencilin and smth)

Q/ Very poor apgar score with cyanosis, what to do?

Q/ Bone tumor seeding in pelvic bone?

Q/Knee pain in eldery. He is on NSAIDs 3 wks came with bloody


vomiting, cause?

Q/ Bipolar disorder pt, developed ss of hypothyroidisim, what is the drug


causing this ?

Q/ Dizygotic twin, what type of chorion?

Q/ Man with erectile dysfunction, but in morning, normal erectile


fuction, where to refer ? cardio, neuro, psych ( I choose it )

Q/ Man with chest pain and transient loc x4 times, ( v long scenario) cxr
given. Asks about management?

Q/ 2 pics of upper and lower extermeties fx, managments?

Q/ Muschroom positiong case, whats the pathophusiology of reaction

Q/ 40 yrs lady with heavy bleeding, all tests normal even, cause?
Anvoultory cycle

Q/ Firt trimester with hx of GDM in last pregnanc 3 yrs ago, when to do


GTT? 16 WKS

October 24, 2017 48


Q/ Middle aged women trying to get pregnant, last baby 1 yr, all test
normal, what to check? FSH

Q/ Pt in labour 6 cm dilation, fetal CTG accelation and variable next step?

Q/ Werenger's case, asks about basic pathology of the dis

Q/ SCD with body pain, the question was. Will u admit the pt or refer t to
tertiary hosp

Q/ Pic of histopath of tumor with starry sky pattern? EBV

Q/ Pic of ring cell stage of malaria, asks about stange and type of
malalria,

Q/Youg pt with bp 160/110, high creatine, ++ protein , whats the doc for
htn?

Q/ Polycystic kid dis, pathology and interhetince,

Q/ Case about gout, the options were mechanism of the drugs, and u
choose appropriate ?

Q/ Blood supply to post compartment of leg?

Q/ Nerve damage affecting lower jaw and post auricle, which nerve?

Q/ Surgery of lower esoph junction, post vagus trunck damaged, what


organ will be affected ? ( rectum bladder, colon, esophagus

Q/ A scsnario about dm tablets ( acretin )

Q/ Stab wound in post triangle of neck, cant raise arm nerve damaged?

Q/ Lower lip lesion, whats lymph drain

October 24, 2017 49


NEW QUESTIONS
From 18 / 9 / 2017
To 24 / 10 / 2017

October 24, 2017 50


٧١٨٢ ‫ سبتمبر‬٨١

Q/ Pancreatic trauma which space will fill first

Q/ Which population in 2015 hajj postpon in corna out brack

Q/ Most common bad prognostic factor of siquzofrina


‫كلها كانت صح ويبغا االكتر‬

Q/ TTP /ITP / 2metheniagravis Q

Q/ Calculata expectded date of ovulation for 32 cylcle

Q/ Fluconazol mainly for oral - mucosal fungus

Q/ Caspofungin new drug to be used for fungemia


caspoofungin ‫ تحطي‬ICU ‫ والمريض في‬IV line ‫يعني لما يقلك‬

Q/ Scphoid forman contant

Q/ Atrial fibrillation prophylaxis (( warfarin )

Q/ Atropin side effects

Q/ TCA side effects

October 24, 2017 51


Q/ Side effects ( respiratory.. forget rest ) wich chemotherapy cause
this : I think bolymycin

Q/ 3 antibiotics ( TT bacteroid fragilis )

Q/ Site of infection of s.taiphy

Q/ Open fracture TT

Q/ Preth's x-ray DD

Q/ Appendistis with mass how to treat

Q/ ‫ بسيط من برا‬.‫ باالضافه لعدد مو‬٨١ ‫ الي ف مدونه‬All comunity question

20/9/2017
Q/ Difference between delirium and dementia?

Q/ Patient come to ER with symptoms for 3 days ) ‫ ( كاتبين اعراض كتير‬and


doctor diagnosis is schizophrenia and that's not good at all because
of:
A.time
B.symptoms

Q/ ALS affect ? Anterior horn

Q/ Protein enter the peroxisomes?

October 24, 2017 52


Q/ What dizygotic twins means?

Q/ Dideoxynucleotide used for ?

Q/ Neurofibromatosis gene ? Ch 17q11.2

Q/ Gene induce apoptosis ? P53

Q/ Bond between tRNA and mRNA ?

Q/ Which one made in nucleolus ? mRNA

Q/ Pregnant women first day of her last menstrual cycle was


13.March 2013 what's EDD ?

Q/ Most common cause of tinnitus ?

Q/ CML ? t 9:22

One question about acid base balance

Q/ Meckls diverticulitis most common in ?

Q/ Winging scapula ? Long thoracic n injury

October 24, 2017 53


‫‪21/9‬‬

‫‪ /Q‬فانكوني انيميا ايش االنهرتنس حقها؟‬

‫‪ /Q‬لما تعالجي احد عنده استيوبوروزيز بفيتامين د؟‬


‫اسمه برايمري برينڤنشن وال تيرشري وال سكندري وال كوادري؟‬

‫‪ /Q‬مين من هدول عنده السيناريو األسوأ لـ‬


‫‪Steven Johnson syndrome‬‬

‫‪ /Q‬اي سي جي ‪ST elevation MI‬ايش الـ‪most appropriate management‬؟‬


‫‪LMWH‬‬
‫‪Streptokinase‬‬
‫‪Thrombolysis‬‬
‫‪Stenting‬‬

‫‪ /Q‬تحسب ‪abgar score‬؟‬

‫‪ /Q‬واحد مابيتنفس ايش حتسوي ‪initially‬؟‬


‫تبدأ ‪ chest compressions 30:2‬وال تشيك عالكاروتيد وال تعطي ‪breaths 1‬‬

‫‪ /Q‬واحد عنده ‪ tonsillitis‬كم حيكون كورس المضاد؟‬


‫‪07,723‬‬

‫‪ /Q‬مين مسؤول عن السنسيشن حق ‪ ant. 2 third of the tongue‬السنسيشن مو التيست؟‬


‫‪0‬‬
‫‪,‬‬

‫‪October 24, 2017‬‬ ‫‪54‬‬


‫‪9‬‬
‫‪21‬‬

‫‪ /Q‬كوع العب التنس ‪lateral epicondylitis‬؟‬

‫‪ /Q‬نوع الهيباتيتس بي ڤاكسين ؟‬

‫‪ dtp /Q‬يحمي ضد مين؟‬

‫‪ /Q‬ايش االقي في االسكلتيشن حق الـ‪croup‬؟‬


‫انسبايرتوري ويز بسبب السكريشن‬
‫وال انسبايرتوري ويز بسبب الناروويينق تحت االيبي قلوتس‬

‫‪ /Q‬جاني ‪ perths‬مرتين؟‬
‫مرة باشعة ومرة بدون‬

‫‪ /Q‬جاني ‪osgood‬‬

‫‪patellar chondromalasia /Q‬؟‬

‫‪ /Q‬سؤالين ‪medial collateral‬؟‬

‫‪ pots in spine - /Q‬وسار عنده ابسيس في االنتيريور تشست وول اتنقلت االنفكشن عن طريق‬
‫‪Anterior cutaneous‬‬
‫‪Or‬‬
‫‪Posterior cutaneous‬‬

‫‪October 24, 2017‬‬ ‫‪55‬‬


‫‪ /Q‬شالولها البيكتوراليس ميجور ويبغو ياخدو فالب من ‪ inferior rectus sheath‬ايش الشريان‬
‫اللي حينقلوه مع الفالب؟‬
‫‪Superior epigastric‬‬
‫‪Inferior epigastric‬‬
‫‪Superficial epigastric‬‬

‫‪ - /Q‬وحدة عندها ديبرشن وسوسايدل اتيمبت لها ‪ ١‬شهور ستيبل على الباروكزتين ودحين حملت؟‬
‫هل حتكمل الباروكزتين وتعمل مونيتور للدبرشن‬
‫وال توقف الباروكزتين عشان البري ماتيور‬
‫وال توقفه عشان الدامج للبيبي‬
‫وال تخليه وتضيف معاه ‪valium‬‬

‫‪ /Q‬وحدة عندها سمبتومز حقت شيزوفيرينا لمدة ‪ ١‬شهور وبعدين رجعت عادية؟‬
‫سكيزوفورم‬
‫بريف سكيزو‬
‫سكيزو‬

‫‪ /Q‬وحدة عندها اعراض شيزوفرينيا من شهر والدكتور شخصها شيزوفرينيا ايش اللي يخليك تستبعد‬
‫الشيزوفرينيا؟؟‬
‫المدةالنها اقل من ‪ ٦‬شهور مو شيزوفرينيا‬

‫‪ /Q‬بيدياتريك عنده مننجايتس ال‪csf‬؟‬


‫‪Protein highCellular high Glucose normal‬‬
‫هل هوا‬
‫‪Tubercular meningitis‬‬
‫‪Or‬‬
‫‪Viral meningitis‬‬

‫‪October 24, 2017‬‬ ‫‪56‬‬


‫‪ /Q‬ايش اللي لما مايحصل له ‪Heterogeneity‬يسبب جاستريك كانسر او شي زي كدا مني فاكرة‬
‫الخيارات‬
‫‪E cadherine‬‬

‫‪ /Q‬وحدة حامل بتوأم ‪dizygotic‬ايش حيكون وصف البالسنتا؟‬


‫‪ Di di‬بغض النظر عن السكس‬
‫‪Or mono mono‬‬
‫‪ Or di mono‬ونفس السكس‬
‫‪Or‬‬
‫‪ Mono di‬ومو نفس السكس‬

‫‪ /Q‬وحدة جات تتابع عشان عندها ليجن في الليبيا ميجورا نص سم في نص سم اخدو بايوبسي ايش‬
‫حتكون النتيجة حقت الهستوباثولوجي؟‬
‫‪Adenocarcenoma‬‬
‫‪Squamous cell carcinoma‬‬
‫‪Adeno squamous cell carcinoma‬‬

‫‪ /Q‬وحدة عندها ‪ plaque‬في الليبيا ميجورا برضو هل هيا‬


‫‪Basal cell carcinoma‬‬
‫‪Melanoma‬‬
‫‪lupus pernio‬‬

‫‪ /Q‬سؤالين فيهم ‪ cerviacl incompetence‬؟‬

‫‪ /Q‬لو وحدة جات في االسبوع العاشر كان عندها نزيف وهيا متشخصة بسرڤايكل انكومبتنس النه‬
‫حصلها ابورشن مرتيندحين السرڤايكل اوس مقفل ووقف النزيفايش حتسوي؟‬
‫‪Bed rest‬‬

‫‪October 24, 2017‬‬ ‫‪57‬‬


‫‪Cerclage‬‬
‫‪Salicylate‬‬
‫‪Admission‬‬

‫‪ /Q‬وحدة في االسبوع الخامس من الحمل نزيف وكلوت واالوس مفتوح والنزيف وقف‬
‫وعندها هستوري مرتين ابورشن بسبب ‪ cervical incompetence‬دي المرة ايش السبب؟‬
‫‪Chromosomal‬‬
‫‪Or cervical incompetence‬‬

‫‪ - /Q‬لو وحدة حامل في ‪ ١٢‬اسبوع وجات العيادة بـ ‪RUQ pain, headache and blurred‬‬
‫‪vision‬عملتولها ‪emergency CS‬واعطيتوها اي ڤي مغنيسيوم وبعدها بساعة سارت تاكي كارديا‬
‫‪ ٨١١‬والضغط ‪ 001‬على شي وسبعين ‪What to expect‬‬
‫‪Hemorrhage‬‬
‫‪Mg toxicity‬‬
‫‪anasthesia induced hypotension‬‬

‫‪ /Q‬وحدة بعد الوالدة بساعتين جاها نزيف ايش اول شي تسويه؟‬


‫‪Ergo‬‬
‫‪Ringer lactate‬‬

‫‪ /Q‬واحد جا الطواريء فاقد دم وعاطيك الڤايتل ساين الضغط مرررة نازل وتاكي سأل عن ايش اول شي‬
‫تعطيه‬
‫‪Ringer‬‬
‫‪Packed RBC‬‬
‫‪Blood‬‬
‫‪Fresh frozen plasma‬‬

‫‪October 24, 2017‬‬ ‫‪58‬‬


‫ كان بيلعب تنس وجا بالم في بطنه‬01 ‫ واحد عمره‬/Q
‫ وبس بدون تروما ايش اول شي تعمليه؟‬Lt para umbilical
Erect chest xray
Abdominal ct
Kidney us

‫ واحد بياخد بنسلين او شي من عيلته وبعد تاني يوم جاته دياريا ومدري ايش معاها ايش السبب‬/Q
Psudomembranus colitis
Something colitis
Ulcerative colitis

psudomembranus colitis ‫ ايش سبب‬/Q


Bacterial
Idiopathic

Angiodema‫ ايش الدوا اللي من السايد ايفكت حقته‬/Q


CCB
ACEi

‫ وقلوكوما يش حتعطيلو دوا؟‬COPD ‫ واحد عنده‬/Q


Topical temolol
Topical something olol
Oral Acetazolamide

‫ وحدة اتخدشت عينها بظفر ايش تعطيها؟‬/Q


Steroid ointment
Steroid drop

October 24, 2017 59


‫‪Antiviral ointment‬‬
‫‪Lens‬‬

‫‪ /Q‬واحد عنده صداع ومشكلة في النظر والم ويمكن عيونه مدفوفه على برا مني متأكدة ايش‬
‫المانجمنت؟‬
‫‪Head MRI‬‬
‫‪Corticosteroids‬‬

‫‪ /Q‬واحد عنده تشنج من ‪ ١‬ايام عنده هايبوناتريميا واالزموالليتي حقت الدم نازله شوية وحقت اليورين‬
‫طبيعية ايش السبب‬
‫‪Cons syndrome‬‬
‫‪Addison syndrome‬‬
‫‪Cushing‬‬
‫‪Inappropriate ADH secretion‬‬

‫‪ /Q‬واحد عنده كومنيوتيد فراكشر في البروقزيمال تيبيا البلس تمام والنيورو تمام‬
‫‪ABI 0.83‬‬
‫‪What to do next‬‬
‫‪Angiography‬‬
‫‪Ct tibia‬‬
‫‪Doppler‬‬

‫‪Apoptosis p53 /Q‬‬


‫‪Q/ Proteins enter protosome‬‬
‫‪Folded c terminal‬‬

‫‪ /Q‬واحد كان عنده فيڤر ‪ ١‬ايام وسار عنده راش في الجسم وفي صورة نقط بيتيكي في الخد من جوا‬

‫‪October 24, 2017‬‬ ‫‪60‬‬


Measles
Kawazaki

polycistic ovary‫ اكتر كانسر يجي مع ال‬/Q


Lung
Breast
Endometrium
Ovary

‫عشان االكني حتحذريها من ايش؟‬retin A ‫ وحدة وصفتيلها‬/Q


‫زيادة البرست تيشو‬
‫وال التعرض للشمس‬

pansystolic murmur radiated to axila ‫ عندها‬٨١ ‫ وحدة عمرها‬/Q


‫ومدري ايش كمان االي اس ار عالي الخيارات‬
SLE
milary tb
hodjkin

?fictitious fever‫ كيف تعملي اكسكلوجن للـ‬/Q


Blood culture
Urine analysis
Heart rate
Rheumatoid factor

?‫ شي يقلل حدوث بعض انواع الكانسر‬/Q


Vit D

October 24, 2017 61


Fibers
Salt

‫ واحد كبير عنده نقص في الوزن وامساك ووايش اللي له دايقنوستيك ڤاليو اكتر شي؟‬/ Q
Colonoscopy

‫ واحد جا الطواريء كونجستيد نيك ڤين مافي صوت للرئه صعوبة في التنفس ايش اول شي تعمله؟‬/Q
Needle thoracostomy

‫ واحد عنده نيموثوراكس ايش تعمله؟‬/Q


Tube thoracostomy

‫؟‬TOF‫ ايش من دي يجي مع ال‬/Q


Pulmonary stemosis
Lt ventr. Enlargement

Pancystolic murmur /Q
‫ويزرق لما يبكي‬In 3rd Lt parasternal
‫هل‬
Ventricular septal defect
Or

.severe aortic regurg ‫ ايش يجي مع‬/Q


Collapsing pulse
Low raising pulse
Pulsus something
Pulsus something

October 24, 2017 62


‫‪months 03 /Q‬‬
‫‪ On regular auscultation‬لقيت بانسيستوليك مرمر في االيكو ‪ ٧‬مل فتحه في‬
‫ال ‪interventricular septum‬في الماسكوالر ايش حتسوي؟‬
‫‪Watchful waiting‬‬
‫‪Surgicalrepair‬‬

‫‪ /Q‬لو واحد عنده سكر على ميتفورمين وتبغي تزوديله دوا تاني وهوا عنده حساسية من السلفا ايش‬
‫تعطيه‬

‫‪ /Q‬ايش دوا السكر اللي ينفع في الحمل‬


‫‪Metformen‬‬
‫‪Sulfa‬‬
‫‪Sitagliptin‬‬

‫‪ /Q‬لو مريض عالجتيه بعدين جابلك هدية ساعة غالية‬


‫تقبلها وتحذره من انه يعيدها‬
‫ترفضها‬
‫تقبلها‬

‫‪ /Q‬وحدة عندها سكر على فورمين وحملت‬


‫تحوليها على انسولين‬
‫‪ /Q‬واحد عمره ‪ ٨١‬سنة وكان بيلعب مع اصحابه قام دف صاحبه من ورا من جهة الهيب بعدين جا‬
‫المسكين تعبان ايش التندون اللي يمنع االوڤر اكستنشن‬
‫‪Iliofemoral‬‬
‫‪...Ischio‬‬

‫‪ /Q‬واحد على سنترال الين وجاله فنقال انفكشن‬

‫‪October 24, 2017‬‬ ‫‪63‬‬


‫تعطيه كسبوفنجين‬

‫‪ /Q‬جا واحد عمره ‪ ٨١‬اظنقبل سنه اخد الڤاريسيال ڤاكسين وماجا ياخد السكند دوز غير دحين‬
‫فهل‪:‬‬
‫تعطيه السكند دوز‬
‫تشوفي التايتر قبل وبعدين تعطيه‬
‫تعيدي من جديد الدوز االولى وبعدين التانية‬

‫‪ /Q‬وحدة جات العيادة عشان الكاونسلينق قبل الحمل‬


‫هل‬
‫‪Give her MMR‬‬
‫‪Rubella titer‬‬
‫‪Varicella vaccine‬‬

‫‪Q/ honey moon cystitis‬‬

‫‪ /Q‬وحدة معاها قونوريا ايش الزم تعمليله رول اوت؟‬


‫‪Herpes‬‬
‫‪Chlamydia‬‬
‫‪Bacterial vaginosis‬‬

‫‪ /Q‬اشهر مكان للقونوريا عن الفيميل‬


‫‪Urethra‬‬
‫‪Cervix‬‬

‫‪ /Q‬وحدة بعد ماولدت جات بديستارج اظن المهم اعراض زي كدا‬


‫هل عندها ‪fungl‬‬

‫‪October 24, 2017‬‬ ‫‪64‬‬


‫‪Or bacterial vaginosis‬‬

‫‪ /Q‬وحدة عندها ‪greenish to yellowish discharge‬‬


‫‪On vaginal exam there is‬‬
‫اظن نقط حمرا زي البيتيكي‬
‫يعني غالبا ً ‪trichomons‬‬

‫‪ /Q‬متى تبدأي عالج الترايكومونس‬


‫لما تبدأ االعراض‬
‫وال فورا ً‬

‫‪ /Q‬وحدة في التالتين والڤجاينا مدري اشبها المهم اظن ‪ovarian failure‬‬


‫النه كمان زايد عندها ال‪fsh‬‬
‫‪And‬‬
‫‪LH‬‬

‫‪ /Q‬وجا سؤال دستشارج برضو كان ‪protozoa‬‬


‫فهل هيا كالميديا وال ترايكومونس وال وال‬

‫‪ /Q‬وحدة عندها متروريجيا ايش تعطيها‬


‫انا اخترت كومبايند‬

‫‪ /Q‬سؤالين عن ال‪emergency contraceptives‬‬


‫واحد انه كيف تشتغل‬
‫تمنع االمبالنتيشن‬

‫‪ /Q‬ووحدة انه جات بعد بيوم من االنتركورس وخايفه من الحمل‬

‫‪October 24, 2017‬‬ ‫‪65‬‬


‫تعطيها امرجنسي كونتراسبتڤ‬

‫‪ /Q‬سبب االكتوبيك برقننسي في التيوب‬


‫‪Early abscess of zona pellucida‬‬
‫‪Or persistence of zona pellucida‬‬

‫‪ /Q‬وحدة كبيرة عندها كونستبيشن وويت لوس وماس في البلفس ايش حيساعدك توصلي للتشخيص‬
‫‪Pap smear‬‬
‫‪Colposcopy‬‬

‫‪ /Q‬واحد جا من غانا‬
‫‪Starry‬‬
‫الجواب بركت‬

‫‪ /Q‬واحد عنده ‪lymphadenopathy Spleenomegaly‬‬


‫ومدري ايش كمان‬
‫الجواب ايبيشتاين‬

‫‪ /Q‬وحدة عندها هيباتايتس بي وجايبينلك نتيجة الباب سمير ايش السبب‬


‫الهيباتايتس ڤايرس وال الهيومان بابيلوما‬

‫‪ /Q‬واحد عنده ستريك حمرا ومدري ايش اللي هوا لمفانجايتس يبغا العالج‬
‫هل اي في انتيبيوتك وال اورال انتيبيوتك وال انتي فنقل‬

‫‪ /Q‬سؤال واحد بدأتيه على شي زي البنسلين وبعدين طلعت النتيجة انه عنده رزستنس لل ‪cefozlin‬‬
‫ايش حتسوي؟‬
‫تكملي‬

‫‪October 24, 2017‬‬ ‫‪66‬‬


‫وال تحوليه على ڤانكو‬
‫وال وال‬

‫‪ /Q‬العضلة اللي تمر في الـ‪lesser sciatic foramin‬‬

‫‪ /Q‬سؤال عن ‪wernicke's area‬‬

‫‪ /Q‬المايلستون واحد يجلس ‪triploid‬‬


‫يحاول يمسك االشياء وينقلب وكدا‬

‫‪ /Q‬واحد يحكي حكاية ويلعب دور االب ويرسم اخوه‬


‫‪ 4‬وال ‪ 0‬سنين‬

‫‪ /Q‬من يبدأ يبتسم الطفل؟‬

‫‪ /Q‬واحد عمره ‪ ١‬سنين‬


‫امه جابته وبتسأل عنده تأخير والال‬
‫الغرباء يفهمو ‪ ٪٢٧‬من كالمه يستجيب لـ‪parts command 4‬ويسأل فريكونتلي‬
‫هل طبيعي‬
‫وال عنده تأخير في االستجابة‬
‫وال تأخير في الكالم‬
‫‪ /Q‬بنت صغيرة عندها سبتك ارثرايتس‬
‫المانجمنت عملية و اي ڤي‬
‫وال‬

‫‪ /Q‬فين يمشي ال‪csf‬‬


‫‪Subarachnoid‬‬

‫‪October 24, 2017‬‬ ‫‪67‬‬


‫‪ /Q‬يساعد على زيادة امتصاص ‪non heme iron‬‬
‫‪Vit c‬‬

‫‪ /Q‬وحدة تبغي تولديها البليتلت نازلة ايش حيكون التخدير؟؟‬


‫جينرال‬
‫البيودندال وااليبيديورال كونترا انديكيتيد في اللو بليتلت‬

‫‪ /Q‬واحد كان عنده الم في البطن واستفراغ واظن سخونه‬


‫والم في المفاصل‬
‫بعدين سار عنده راش في البوتوكس وناسية فين كمان‬
‫هنوكشالين وال‬

‫‪ /Q‬واحد عنده سكر وبروتين في اليورين ايش تعطيه دوا ضغط؟‬

‫‪ /Q‬جابو سؤالين عالنالوكزون واالوبيويد‬

‫‪ /Q‬جابو سؤال واحد عنده اعراض من ضمنها اللي تجي في العين حقت ولسون‬
‫والجواب بينيسالمين‬

‫‪ /Q‬وحدة اظن ماحملت من ‪ ١‬سنين‬

‫وكان في شي زي عقد اللولو 😅‬


‫ناسية صراحه🤦🤦♀‬

‫‪ /Q‬وحدة عندها اكتئاب وجات اظن فاقدة الوعي وكان جنبها علبة حبوب فاضية‬
‫دايليتد نن رياكتڤ بيوبل ايش الدوا اللي اخدته؟‬
‫‪fluxatin‬‬

‫‪October 24, 2017‬‬ ‫‪68‬‬


‫‪or sertraline‬‬

‫‪ /Q‬وحدة من االعراض باين انها في ثايرويد ستروم‬


‫ايش اول شي تعطيها؟‬
‫‪Propranolol‬‬

‫‪ /Q‬حالة هايبوثايرويد واضحه‬

‫‪ /Q‬حالة هايبرثايرويد‬
‫ايش اكتر شي يفيدك في التشخيص؟‬
‫ثايرويد سكان‬
‫وال نيدل‬
‫وال الترا ساوند‬

‫‪ /Q‬وحدة عندها ماس في البريست‬


‫في االم ار اي كالسفيكيشن‬
‫ايش تسوي‬
‫نيدل‬
‫وال ماستكتومي على طول‬
‫وال‬

‫‪ /Q‬وحدة ‪٨١‬عندها ماس في البرست ايش حيفيدك اكتر شي في التشخيص‬


‫نيدل‬
‫وال الترا ساوند‬
‫وال مامو‬

‫‪ /Q‬واحد عنده فومتينق وهايبر ڤنتليشن‬

‫‪October 24, 2017‬‬ ‫‪69‬‬


‫ايش حيكون عنده‬
Metabolic acidosis and respiratory alkalosis
Or
Metabolic acidosis and respiratory acidosis

‫كيف تمنعي العدوى‬viral conguctivitis ‫ واحد عنده‬/Q


‫غسيل اليد‬
‫العزل‬
‫باتش‬

‫ سنين‬١ ‫ واحد عنده ريكرنت هيموبتسيس من‬/Q


Missing details
Bronchiactasis
Tb

uncomplicated‫ عالج ال‬/Q


Falciparum malaria

‫ متى نقول انه برولونقد بريقننسي‬/Q


43
42
41
40

‫ واحد عنده اعراض ريكتس ايش الفيتامين الناقص‬/Q

October 24, 2017 70


‫‪ /Q‬واحد عنده استيوبوروسيس‬
‫‪0.0-‬‬
‫‪1-‬‬
‫‪0.0‬‬
‫‪1‬‬

‫‪ /Q‬واحد كبير في السن عندخ استيوبوروزيز ايش تعطيه‬


‫‪?Alendronate or vit d‬‬

‫‪ /Q‬ايش االنسيزيا اللي لها انالجيسيك ايفكت لو كان الدوز قليلة‬


‫ميدازوالم وال كيتامين وال‬

‫‪ /Q‬ايش الدوا اللي نعطيه مع االنالجيزيا عشان يقلل تأثيرها؟ ميتوكلوبرمايد؟؟‬

‫‪ /Q‬واحد جا باعراض في االبر لمب زي كأنه ريجيد مني فاكرة وانه قبل كم يوم جا المستشفى اخد‬
‫عالج ايش كان العالج اللي سبب دي االعراض‬
‫ميتكلوبرمايد وال‬

‫‪ /Q‬واحد عنده ليتيك ليجن في الفيمر تحاليل البروستيت كانسؤ طبيعية‬


‫هل عنده باجيت ديزيز وال‬

‫‪ /Q‬كونترا انديكيشن للقاستريك الفاج‬


‫اللي يكون اخد اسبرين وال شي حقتنظيف‬

‫‪ /Q‬تشايلد بلع ‪caustic material‬‬


‫‪drooling‬‬
‫ايش اول شي تسويه؟‬

‫‪October 24, 2017‬‬ ‫‪71‬‬


‫‪airway‬‬
‫‪emergency endoscope‬‬
‫‪charcol‬‬

‫‪ /Q‬ديابيتك وعنده السر في الفود‬


‫ايش اللي يعمله الديابيتيس عشان يقلل االميونتي‬
‫يقلل الفاقوسايتوزيز‬
‫وال يقلل المناعه‬
‫وال يزيد البكتيريا‬

‫‪ /Q‬شي في الخيارات ‪cd4‬‬

‫‪ /Q‬الطاقة من والى النواة‬


‫‪atp in cytoplasm‬‬
‫‪gtp in cytoplasm‬‬
‫‪gtp in nucleus‬‬
‫‪atp in nucleus‬‬

‫‪ /Q‬اطول مرحلة في السل سايكل‬


‫‪interphase‬‬

‫‪ /Q‬مياثينيا قريفيز كيس وايش الميكانزم‬


‫انتي بدي لالستايل كولين ريسيبتور‬

‫‪ /Q‬سؤال عن الفوسفو بويزنينق‬

‫‪ /Q‬سايد ايفكت حقت االتروبين او االكشن حق االتروبين ‪not sure‬‬

‫‪October 24, 2017‬‬ ‫‪72‬‬


‫‪ /Q‬وحدة حامل بتاخد ايرون لعالج االنيميا وجاتباعراض انيميا والهيموقلوبين نازل ايش التشخيص‬
‫ايرون دقشنسي وال بيتا ثالسيميا‬

‫‪-‬كان في كذا سؤال قريبين من بعض‬


‫واحد منهم الفرتين عالي والهيموقلوبين نازل‬

‫‪ /Q‬واحد عنده ساركويدوزيز من سنتين وجا باعراض انيميا وبرضو نتايج تحاليل‬
‫ايش السبب‬
‫انيميا اوف كرونيك ديزيز وال وال‬

‫‪ - /Q‬وحدة حامل وعندها ماكروستيك انيميا‬


‫ايش اعطيها؟ فوليك؟‬

‫‪ - /Q‬تشايلد جا بفراكتشر اكتر من مرا والسكليرا زرقا‬


‫‪osteogenesis imperficta‬‬

‫‪ /Q‬ايش الميكانزم حق االنسولين رزستنس في االوبيز‬

‫‪ /Q‬ايش الميكانزم حق الميتوفرمين على العضل‬


‫يزود القلوكونيوجينيسيس؟‬
‫وال وال‬
‫‪ /Q‬مكونات االنكل جوينت‬
‫‪Distal tibia, fibula, talus‬‬

‫‪ /Q‬الدم اللي جاي للسوبرفيشيال بالمار ارش قليل‬


‫المشكلة في ايت شريان‬

‫‪October 24, 2017‬‬ ‫‪73‬‬


‫‪?ulnar‬‬

‫‪ /Q‬وحدة ماتقدر تحرك يدها هل المشكلة في ال‬


‫‪ant. interossi‬‬
‫‪'or post. interossii‬‬
‫‪or theaner‬‬

‫‪ /Q‬وحدة ديابيتك وعندها الم لما تحاول تعمل سترتش للسكند والثيرد فنقرز واظن ماتقدر تحرك يدها‪,‬‬
‫كانو جايبين صورة‬
‫‪diabetic dactylyl‬‬
‫‪dupturen contracture‬‬

‫‪-‬كذا كيس عن التنميل في الهاند‬

‫‪ /Q‬حصل تروما بعدين ماسار يقدر يثني الدستال فالنكس السبب ربشر في الفليكسور بروفندس تندون‬

‫‪ /Q‬ايش وظيفة العضالت اللي في االنتيريور كومبارتمنت حقت الفورارم‬


‫انها تثني الرست والفنقرز‬
‫وال تثني الكوع‬

‫‪ /Q‬صورة كسر في الهيومرس وويكنس في الرست اكستنشن‬


‫مين حصل له انجري؟ الريديال‬

‫‪ pancreatic laceration and leaking out‬لالنزيمات‬


‫فين حتتجمع؟‬
‫‪para somthing gutter‬‬
‫‪or‬‬

‫‪October 24, 2017‬‬ ‫‪74‬‬


‫‪Q/ acute sudden testicular pain forget the rest‬‬
‫‪answer refer to surgery‬‬

‫‪Q/ breast cancer, dimpling of skin caused by‬‬


‫‪coopers ligamints or‬‬

‫‪ /Q‬لو في ماس في االوتر ابر كوادرنت‪ ,‬ايش اول لمف نور حفحصها يعني اول لمف نود حيروحلها؟‬
‫‪... anterior or lateral or medial or‬‬

‫‪Q/ bilateral breast cancer‬‬


‫‪lobular‬‬

‫‪fibroadenoma /Q‬‬

‫‪ /Q‬وحدة في االربعين وعندها ماس ايش تسويلها؟‬


‫مامو‬

‫‪ /Q‬واحد عنده باد بريث وراجع االسنان كلو طبيعي‬


‫‪and he expectorates seed like particles‬‬
‫ايش عنده؟‬
‫‪crypt tonsilits or‬‬

‫‪ /Q‬تشايلد عنده فريكونت فومتينق‪ ,‬وريفلكس ايش السبب‬


‫انه اللور ايسوفجيال سفنكتر ضعيف‬

‫‪ /Q‬البنت بعد المينارك بكم شهر حيوقف نمو السباين (طوله يعني)‬

‫‪October 24, 2017‬‬ ‫‪75‬‬


‫‪ 6‬شهور وال ‪ 04‬شهر؟‬

‫‪ /Q‬واحد جا بهورسنس ومدري ايش‬


‫ايش حتسوييله اول شي‬
‫‪laryngyoscope or enner ear MRI or‬‬

‫‪ /Q‬واحد جا بماس في الثروت‬


‫ايش المانجمنت‬
‫‪surgery plus or minus radiotherapy‬‬

‫‪ /Q‬هيب دسلوكيشن اور سمثنق‪ ,‬والرجل اكستيرنالي روتيتيد ايش العضلة المسؤولة؟‬
‫‪gluteus maximus‬‬

‫‪ /Q‬واحد ضغطة ‪ 061‬على شي‬


‫ايش السبب؟‬
‫‪change in paroreceptor sensitivity or‬‬
‫‪increase periphral vascular resistance‬‬

‫‪ /Q‬واحد في شوك اور سمثنق‬


‫ايش الميكانزم اللي حيحصل‬
‫يقل الكوروناري بيرفيوجن وال تقل الفينس كباستي‬

‫‪ /Q‬واحد عمره ‪ ٨١‬شهر جابته امه عشان متأخر في شي‬


‫ايش ممكن يكون؟‬
‫‪Crawling‬‬
‫‪Walking using furniture‬‬
‫‪Walking alone‬‬

‫‪October 24, 2017‬‬ ‫‪76‬‬


‫‪ /Q‬واحد كبير ديابيتك جا باعراض هايبرقاليسيميا‬
‫السكر عالي مافي كيتون والرينال فنكشن تمام هل السبب‬
‫‪Hyper osmolar‬‬
‫‪Lactic acidosis‬‬
‫‪Diabetic ketoacidosis‬‬

‫‪ /Q‬طفل عنده ‪!garlic breathing‬‬


‫‪!Forget other details‬‬

‫‪ /Q‬طفل ديابيتك اتصلو عليكي اهله انه فاقد الوعي ايش حتنصحيهم يعملو اول شي؟‬
‫ابرة جلوكاجون‬

‫‪ /Q‬وحدة عنها كلير فلويد من النيبل ريقيوالر سايكل ايش تعمليلها؟ ‪prolactin‬‬

‫‪ /Q‬واحد عنده ‪erectile dysfunction‬‬


‫اعطيتيه عالج‪ ،‬ايش اللي حيكون كونترا انديكيتد انه ياخده معاه؟ ‪nitrate‬‬

‫‪ /Q‬ولد عمره ‪ ٨١‬متأخر في البيوبيرتي‬


‫في النمو كل شي عنده نورمال‬
‫‪Fair hair in groin‬‬
‫هل‬
‫‪wait‬‬
‫‪Free testosterone‬‬
‫‪Testicular us‬‬

‫‪ /Q‬بنت برضو متأخرة في البلوغ ‪wide neck‬‬

‫‪October 24, 2017‬‬ ‫‪77‬‬


No breast buds
Turner syndrome or nonan syndrome or x fragile

‫وصفات داون‬asd ‫ عنده‬١٤ ‫ طفل ألم عمرها‬/Q


.. Trisomy 21 or

‫ واحد عنده هايبرثايرويد جا بعين حمرا ايش الدوا اللي سبب دا الشي؟‬/Q

‫ سؤال عن بهجت ديزيز‬/Q

‫ سؤال يوڤيايتس‬/Q

21 / 9 / 2017

Q/ Lower lip lesion what is the Lymphatic drainage?

Q/ Which of the following made in nucleolus?

Q/ long scenario he wants the equation of relative risk ?

Q/ Blood supply to AV and AS node in the heart ?

Q/ Child , unmplical swelling , yellow discharge + pic?

Q/ Blood supply to AV and AS node in the heart ?

Q/ drug of choice in absence seizures?

October 24, 2017 78


Q/ pt with parotitis and have pine with eating that radiated to the ear
"pain by which nerve?"

Q/ associated with animal bites?

Q/ case of Henoch-Schönlein purpura

Q/ Central line with fungal inf. What is the TTT?

Q/ Pt with central line and get infection what is the site ?

Q/ Case with conjunctivitis and the eyelash turnd inward?

Q/ at daycare centre 10 out of 50 had red eye in the 1st week ,anther
30 developed same condition in the next week .What is the attack
rate?

Q/ Water discharge +red eye?

Q/ Anti psychotic causes weight gain?

Q/ Hinge joint?

Q/ Ioos of sensation of 2/3 of the tounge?

Q/ Postroir vagul trunk supplies?

October 24, 2017 79


Q/ case about bartholin cyst

Q/ Pregnancy with HTN and preaclmsea (3 cases!)

Q/ PCOS and associated malignancy

Q/ vaginal discharge (4 cases!)

Q/ pt with high gred hyperplasia with atypia and total hysterctomy

Q/ precusu puberty

Q/ Moler pregnancy

43/9
Q/ Epileptic pt on carbamazepine =prospective cohort study .

Q/ Study selective every10th family in city =systematic study .

Q/ non response bias =cs

Q/ Lung ca 90% 30% smokers =70% .

Q/ Dadycare another developed same =80% .

Typhoid fever 💯 🤦

Q/ Sensitive =probability that a disease pt have +ve test results

October 24, 2017 80


Q/ specificty = probability thr non disease have -ve test results .

Q/ Balder ca = 2.25 .

Q/ IR 1000/8000Q/ 100=12.5 .

Q/ Cohort study +ve mersa ration of exp on non exp =66:1


Q/ Risk exp to mersa -vir=66

Q/ Lung ca66:0=)011/6(/)011/11( =

Q/ RR =a/(a+b)/ c/(c+d)= 204 .1=)11+292(/292/ )11+41(/

Q/ Insulin =near to treatment 14

Q/ Congenital hearts -GD


OR0.400=)2931/11(/)911/41( =
RR0.40=)0111+11(/)0111/41(=

Q/ GDM -> OR =2

Q/ new case 200 old case 80 total 280 so)1111111/411( Q/ 3.0=011111

Q/ odd ration 75 control rate 0.5


Relative risk is .12

October 24, 2017 81


Q/ group of breast ca mastectomy with radiation and mastectomy
with
Q/ chemo = randomized controlled trial .

Q/ Epidemic investigations wt first = identiving population at risk .


Risk of un exp 2 and risk of exp 3 wt true =1

Q/ Bladder ca cohort study =5

Q/ 4 groups disease and non disease = RR=.5

Q/ Incident of RS infection 117 in 1000. = 1.2

Q/ Study compared and followed till birth type =cohort study

Q/ Research about creatin disease in the beginning were 2000 but


after 3y were 1000 wt incidence in one year =10%

Q/ Most determine study =positive predictive value .

Q/ Study used to comparing 2 groups = cohort study

25-9-2017

Q/ Child unvaccinated developed fever, SOB, stridor. Lateral X-ray


Show thumb sign what’s the treatment :

A)Erythromycin , B) vancomycin C) ceftriaxone✅ D) metconazole

October 24, 2017 82


Q/ Open lung biopsy of old female show hemosiderin laden
macrophage what is the cause:

A)Heart failure✅ , B) Pneumocystis jiroveci

Q/ Middle age man with acute onset of fever malaise anorexia x-ray
show bilateral hilar and left paratracheal lymphadenopathy what is
the diagnosis:

A) HIV B) sarcoidosis C) TB✅ D) rheumatic fever

Q/ Female pregnant with small fibroid what to tell her about the risk:
A)Risk of preterm labor B) Degeneration is common C) Mostly
asymptomatic✅

Q/ female with bilateral menstrual breast pain o/e bilateral breast


nodularity and mobile mass in the right breast axillary lymph free
what to do next:

A)mammograph then US✅ B) aspiration and cytology C) Reevaluate at


the next cycle

Q/ newly diagnosed DM patient came for the appointment and you


came late 1 half hour because you had critical case the patient was
angry what to do:
A)acknowledge the patient anger. B) Explore the patient anger. C) talk to
the pt about his anger D) Empathy with the pt ✅

Q/ case of hyperthyroidism what will be associated:

A)AF ✅B) VT C) SVT D) WPW

October 24, 2017 83


Q/ pt with only palpitation examination normal ecg show heart block
what is the treatment:

A) CCB ✅ B) beta blocker C) digoxin

Q/ Most "effective" treatment for obstructive sleep apnea:

A) decrease BMI less than 30 B) CPAP✅ C) Moindifil

Q/ what is best to describe post traumatic syndrome:

A) disorientation and dissociation B) Flashback and arousal ✅

Q/ out come of 45 y/o pt with recurrent schizophrenia on


maintenance therapy:

A) complete remission B) 70% will have good quality of life. ✅

Q/ case of viral conjunctivitis how to prevent the spread of the


disease:

A) Isolate pt. B) Eyes patch C) Hand wash. ✅

Q/ which one is consider as 1st line treatment of esotropia :

A) medial rectus resection✅ B) Ortho”something)

Q/ pt with tachycardia and SOB after bee sting what is the


"OPTIMAL" treatment:
A) remove stinger and watch the pt B) SC epinephrine and IV
antihistamine✅.

Q/ Blunt tauma in antirior abdomen with pancreatic injury and


enzymes leak where the enzyme will go:

October 24, 2017 84


A) omentum bursa✅. B) sub hepatic space C) sub phrenic space

Q/ proximal Tibial fracture palpable peripheral pulse ABI: .85 what to


do:

A)angiography B) Doppler US. ✅ C) CT.

Q/ Child tripped and twisted his leg now he can’t walk:


A) spiral fracture of proximal tibia. B) spiral fracture of femur. C) chop
fracture of tibia✅

Q/ stap injury to the obturator nerve which muscle will be completely


paralyzed:

A) Biceps femoris B) adductor magnus C) Adductor longus✅ D)


Semitendinosus or membranosus.

Q/ tingling and numbness of the ring and index finger worsen with
elevation of the arm positive stress test:

A) carpal tunnel syndrome. ✅ B) thoracic outlet obstruction C) Ulnar artery


thrombophlebitis.

Q/ pain at the midline of the planter surface when walking and


standing:

A) planter fasciitis. ✅ B) flexor digitorum longus. B) flexor hallucis longus.

Q/ loss of adduction of the fingers which nerve injury:

A) ulner✅ B) Radial. C) Median D) Axillary.

Q/ Child with rapid blinking of the eye communicate with his parents
during it what is the diagnosis all are normal:

October 24, 2017 85


A)TIC ✅B) petit mal seizure C) tourette syndrome.

Q/ 2y/o child doesn’t speak except mama papa response will to verbal
command his older brother had the same problem when he was same
age he is shy what is the diagnosis:

A)Autsim B) development language disorder✅.

Q/ Cancer associated with depression:

A) pancrease✅ B) prostate C) kidney. D) Liver.

Q/ Which cell produce melanocyte releasing hormone??

Q/ old man with abnormal hand and head movement he toke drug 1
day before what is the drug:

A) Chloramphenicol B) Digoxin✅ C) ethambutol

Q/ what is the drug that given with analgesia to decrease side


effect" ‫" بهذا النص‬:

A) cimetidine B) Metoclopramide. ✅

Q/ MVA victim in the ICU developed multiorgan failure lab show high
phosphate what is the organ responsible:

A) Heart B) Liver C) Kidney✅ D) Lung.

Q/ pt with oral and genital ulcer and arthritis: A) Bechet disease✅

Q/ Glomerulonephritis with anti basement membrane antibodies :

A) IgA Glomerulonephritis B) good posture disease✅.

October 24, 2017 86


Q/ glucocorticoids produced from which layer of the adrenal gland:

A) zona glomerulosa B) Zona …….✅C) zona mother fucker

Q/ UTI case with gram negative indole positive:

A) E. coli✅

Q/ recurrence of breast cancer depends on:

A) Number of axillary lymph node involved✅. B) estrogen receptors. C)


progesterone receptors.

Q/ side effect of ribavirin:

A) anemia. ✅ B) renal damage C) Hepatic damage.

Q/ treatment of HSV 2:

A) acyclovir✅ B) ribavirin
Q/ prophylaxis of migraine in asthmatic pt:

A) propanol B) amitriptyline ✅

Q/ 09 y/o female with vitiligo in the face and hand the size is
increasing despite medical treatment now the pt want to get married
and ask you what to do:
A) continue medical treatment. B) stop Medical treatment and wait C) skin
graft. D) melanocyte transfusion. ✅

Q/ what is the cause of ischemic stroke in females:

A) hypercoagulable state✅ B) previous DVT.

October 24, 2017 87


Q/ gun shot in the abdomen with bacteroid fragilis which antibiotic??

Q/ UTI treated with 100mg nitrofurantoin what is the duration:

A) 6h for one month. ✅ B)once daily for 2 month C)once daily for six
month D) once daily for one year.

Q/ how to screen for hemochromatosis:

A) ferritin B) transferrin saturation✅C) liver biopsy. D) Liver us.

Q/ sign for low intestinal obstruction:

A) altered bowel movement. B) absolute constipation✅ C) flatulence D)


diarrhea.

Q/ how to diagnose volvulus:

A) CT B) Barium ✅C) MRI

Q/ site for meckel's diverticulum :

A) ilium✅ B) jejunum. C) duodenum.

Q/ pudendal block which will not be affected:

A) Rectum✅. B) Vulva C) perineal body. D) urogenital diaphragm.

Q/ how to diagnose factitious fever:

A0blood culture. B) urine analysis. C)pulse✅

Q/ tumor arising from the kidney displacing the collecting system :

October 24, 2017 88


A) Wilms tumor.✅ B) Neuroblastoma.

Q/ contracting muscle compared to relaxing muscle:

A) more pyruvate converted to lactate.✅ B )

Q/ which enzyme deficiency will result in converting pyruvate to


lactate :

A) Pyruvate kinase. B) pyruvate carboxylase✅ .

Q/ k/c case of chronic constipation complaining of pain and bleeding


during defecation the pain persist for many hours what is the
diagnosis:

A) thrombosed hemorrhoid.✅ B) anal fissure.

Q/ k/c of chronic constipation complain of pain during defecation on


examination there was fluctuating mass in the perianal area what is
the treatment:

A) sitz bath. ✅B) incision and drainage.

Q/ pt complain of proximal muscle weakness and ptosis there is


improvement after administration of anticholinesterase what is the
cause:

A) autoimmune✅ B) viral induced C) drug induced. D) nutritional.

Q/ close scenario to 52 and ask about treatment.

Q/ the maximum accepted level of LDL in mmol/l??

October 24, 2017 89


Q/ Many Qs about research asking about RR incidence “OD” and
types of studies
Q/ Many Qs about different type of jaundice easy just look for indirect
and direct bilirubin levels and liver enzymes.

25/9/2017
Q/ Female bloody discharge from nipple ?
-fibroadenoma - ductal papillary
-breast abscess

Q/ Patient has fever and headache increased with coughing


-intracranial abscess
-mycobacterium meningitis
-viral meningitis

Q/ Patient depressed found unconscious with empty pills around ,


what could it be ?
-fluxitine
-amitriptylline

Q/ Hip what type of joint?


-Ball and socket (( correct))
-hinge))😭 ‫(( انا حطيته‬

Q/ What we give with analgesia to relieve its SE :


-cimetiden
-metoclopromide

October 24, 2017 90


Q/ Pateient complain of dizziness , he of cardiac disease in family ,
O/E decrease sound on auscultation in carotid artery :
-Doppler US for carotid
-full cardiac work up with ECHO

Q/ Treatment of thyrotoxicosis:
-propranolol

-Q/ what analgesia make the pain of gallstones worse?


-morphine

Q/ Milestones ‫جابو عليها كم سؤال‬

Q/ New born with Hip C ,


-continue breast feeding
-stop breast feeding

8/10/2017

Q/ Most common risk factor for fibroid or (most important) I am not


sure:
A-age
B- African race
C -multipara
D- Smoking

October 24, 2017 91


Note: smoking and multipara is protective against fibroid .

Q/ 28 years old pregnant lady with tubal pregnancy


Q/ Which one of the following most common risk factor of ectopic
pregnancy
A- Induction by ovulation
B- Previous pregnancy
C- IUCD
D- Pelvic inflammatory disease.

Q/ 21 years old patient with known case of depression, has been


found on the floor unconscious with empty pill bottle , patient was
obtunded , dilated pupil and unreactive bilaterally and other
symptom( I can not remember)
What is the medication the patient most likely on?
A-Sertraline
B- Fluoxetine
C- Other SSRI
D- Amitriptyline

Q/ which one of the following SSRI has the greatest risk in


pregnancy?
A- Escitalopram
B- Fluoxetine
C- Paroxetine
D- Sertraline

Q/ 2 year and half kid with long scenario about impulsive behavior
and communication impairment with other and the distinct point of

October 24, 2017 92


the question is echolalia (which is repetition of other words) what is
the diagnosis:
A- ADHD
B-ASPENGER SYNDROME
C- AUTISTIC DISORDERS
D..…………………………… -
My answer was ADHD nut I thinks the correct answer is autistic disorder
because ADHD is rarely diagnosed before age of 7 years but autism is
mostly diagnosed at maximum age of 3 years, other point is echolalia
which is specific for autism .
For the Asperger syndrome it is the same as autism but without echolalia.

Q/ there was question about ADHD diagnostic criteria according to


ICD10 CLASSIFICATION, but I cannot remember the answers:
But it was like this :( number of feature and the description of it) for
example:
A- 3 hyperactive, 3 inattentive, and other
B- 2 hyperactive, 3 inattentive
C- 2 hyperactive, 2 inattentive
According to the last classification it said there should be at least 6
hyperactive and 6 inattentive .
Also there is different type of ADHD: either hyperactive predominant or
inattentive predominate.
Go and read more about it.

Q/ case of septic arthritis patient started on oxacillin, 3 days later the


sensitivity shown up and it was staph rues resistance to cefixiton
what is your management :
A- Stop antibiotic
B-continue oxacillin

October 24, 2017 93


C- Start vancomycin .
D ……………………………-

Q/ Extradural hematoma source of bleeding:


A-middle cerebral artery
B-middle meningeal artery
C-anterior cerebral artery

Q/ adolescent with migraine what is the long term complication?


A-hearing loss
B-depression
C- Schizophrenia
D- Reflux esophagitis

Q/ which one of the following cancers directly related to smoking:


A- Colon
B- Bladder
C- Testicular
D- Small cancer

Q/ first line treatment of tenia pedis:(I cannot recall choices clearly)


A-Systemic ketoconazole.
B- Topical terbinafine.
I think there was other topical choice , so make sure which one is more
important.
Note: principally the treatment should be higher rate of foot hygiene and
topical antifungal.

October 24, 2017 94


Read management from Wikipedia.

Q/ treatment of MS attack in ER:


A- Oral antibiotic
B-IV antibiotic
C- Oral steroid
D- IV steroid

Q/ pregnant lady, we want to give her prostaglandin F2 alpha


Which one of the following diseases should be taken into
consideration?
A- Asthma
B-DM
C-HTN
D- cholelithsis

Q/ Patient developed cutaneous lichen planus lesion on flexor


surfaces of the elbow bilaterally, with no mucosal lesion on the
mouth or scalp, which one of the following statement is correct
regarding this scenario:
A- There is risk for malignancy transformation.
B- Automatic resolution over time .
C-persistent lesion.
D- Relapsing and remitting lesion.
Note: my answer based on this information I got it from Wikipedia
In contrast to cutaneous LP, which is self-limited, lichen planus lesions in
the mouth may persist for many years, and tend to be difficult to treat, with
relapses being common. Atrophic/erosive lichen planus is associated with

October 24, 2017 95


a small risk of cancerous transformation, and so people with oral LP tend
to be monitored closely over time to detect any potential change

early. Sometimes oral LP can become secondarily infected with Candida


organisms.

Q/ Which one of the following patients has worst prognosis to,


develops Steven Jonson:
A-HIV patient receiving sulfa group medication with unknown history
B-gouty patient with known sensitivity to allopurinol
Note: my answer was B BUT I am not sure about it.

Q/ case of young lady with typical history and finding examination of


fibroadenoma
Painless, firm, mobile) )

Q/ case of bloody nipple discharge


Answer is intraductul papilloma

Q/ women with bilateral breast nipple white discharge and vision


disturbance
Prolactin level is high, where is the lesion:
There was no pituitary lesion in choices BUT THERE WAS Sella turcica
lesion
And it is the correct answer

Q/ best investigation to diagnose ECTOPIC pregnancy:


A-Ultrasound
B-Laparoscopy

October 24, 2017 96


C-B HCG

Q/ most common urological congenital male anomaly:


A-Hypospadias
B- Cryptorchidism
C- Phimosis

Q/ which one of the following condition is contraindicated to


circumcision ?
A-Hypospadias
B- Cryptorchidism
C- Phimosis

Q/ child exposed to fingernail injury, what this patient may have


A-Purulent tearing
B-photophobia
C-vision loss
Note: my answer based on the booklet 13.

Q/ same previous scenario but asking about management


My answer is topical antibiotic, also based on booklet 13

Q/ pancreatic trauma case , where is the first place for the


pancreatic enzyme to collect in?
A-omentum bursa
b- Sub hepatic space
C-sub phrenic space

October 24, 2017 97


Note: lesser sac is the same as omentum bursa.

Q/ patient has been bitten by dog, and he received rabbis vaccine 18


months ago when he travelled to some place, what action should be
taken immediately:
A- Do nothing as patient is immunized.
B- Give rabbis vaccine and immunoglobulin .
C- Give immunoglobulin alone.
D- Give two doses of rabbis vaccine.
Note: this q is solved before in one of the questions collection as B, but
what I found that answer D is the correct choice.
There are big details about this topic. So you do not need to waste your
time.

٧١٨٢-١-٧٤
Q/ ‫ وش اكثر شي يصير له انجري‬، ‫سوال البيشنت الي بيركبونله كاثتر‬
Membranous - urethra- post urethra

Q/ 41 y.o presents with hairstusim , acne , menstrual is regular ,


ovaries US shows peerl string appearance , wt dx :
Prolactinoma
PSOS
hypothyrodism.

October 24, 2017 98


Q/ Same case but irregular menstural :
PCOS
Congenutal adrenal hyperplasia

Q/ , cushing ‫بيشنت معاه اعراض‬


‫وين حتشوفون االبنورماليتي‬
Faciculata
Reticularis
? Glumerulosa

Q/ 32 y.o women ,Multiparous , complain of heavy mensturation ,


upon examination : there is intramural fibroid
What is the most common factor for fibroid:
Age ?
African race?
Obesity
Multiparty.

Q/
‫ باالضافه الى‬، ‫ معها سلس البول‬، ‫حرمه ملتيباروس‬pelvic strengthened exercises,
she was given duxbutine , what is the mechanisms of action :
-inhibit muscuranic cholenrgic
-Activate acetylene
... -etc

Q/ 22 y.o man , smoker , overweight, inactive present with heart


problem , What is the most common associated with heart disease :
Smoking

October 24, 2017 99


Obesity
Inactivity

Q/ Parents brought their sun with cystic fibrosis, asking about the
chance of his daughter to be carrier :
2:2
1:4
2:25
‫ كم نسبه ان اخته تصير حمله للمرض‬. ‫ قايلين ان الولد معاه سستك فايبروسيز‬- ‫سوال غبي‬
. ^^ ‫وكاتبين النسب بالشكل هذا‬

Q/ Male present with multiple maculs 5-15 mm , and axilary frickling ,


his wife is pregnant , what is the mood of inheritance ?
Autosoma
X-linked
Mitochondrial
‫قصدهم الـ‬NF

‫ وش الجين؟‬: ‫ كمان سوال ثاني عن النيوروفايبرمتوسيز‬/Q


2,q11.2

:Prophylaxis for travelers diarrhea‫ ايش للـ‬، ‫ واحد بياخذ ولده لساوث اسيا‬/Q
Ciprofluxacine
Ceftrexione
Doxycycline

Q/ ‫واحد بيروح مكان ( السودان او مدري وين) المهم وطلبك نصيحه عن المالريا ف ايش من ذولي‬
‫االفضل كـ‬Prophylaxis for malaria :

October 24, 2017 100


Quinine 1week before traveling, until6 week after returning.
Quinolone 2days before traveling, until returned.

Q/ In viral infection, what will increase:


Lymphocytes
Esionophil
Basophil
Monocyte.

Q/ Patient complain of hand pain after bee sting , on examination:


tenderness + Axillary lymph node enlarged, ( + picture: weal on
forarm) what is dx:
-URTECARIA
-Lypmhangitis ، ?

Q/ Patient on multiple drugs , developed violecious papule and


ezematoid pattern in the trunk , no mucus membrane involving, what
dx?
-Lichnoid erubtion
-Actnic erubtion
-TEN
-eryhthema multiform?

Q/ Patient with red eye + history of cough , ( pic viral conjunctivitis) ,


how to prevent transmission?
-hand washing

Q/ Patient with red eye , cillary flush , photophobia, dentric lesion in


the cornea :

October 24, 2017 101


-HSV of eye

. dentric lesion : ‫ لكن الكي بوينت كان‬.‫ سناريو طويل‬/Q

‫ سناريو طويل عن الجلوكوما‬/Q


pain , congestion of cornea - vomiting, photophobia, what ttt?
-acetazolmide .
- -corticosteroids
-Ab

Q/ How dx corneal ulcer ?


-slit lamb
-Florscine dyeQ/

Q/ What dye is used in corneal ulcer( pic of blue color ):


-florcine
. ‫ انواع صبغات ثانيه‬-

Q/ Which of the following present with positive anti glomelural


basment membrane antibodies
-good pasture syndrome
- -wegner
-Post streptococcus glomerularnephritis

Q/ Cenario , 34 y o female present with fever and decrease


conciousness for 6 hrs .
Investigation :plt : low

October 24, 2017 102


: ‫فقط هذا السناريو‬
Dx?
-ITP
-TTP
-Antiphospholipd syndrome .

Q/ 41 y. O male , with small erythmatous macules non planchable ,


history of viral respiratory infection resolve spontaneously last
week ,
Lab: plt: 15 ( very low )
What TREATMENT:
-ivig
-Splenectomy
-Platelets transfusion

Q/ Cenario : pt develop bloddy diarrhea, abd pain and vomiting, after


eating from restaurant , history if hematuria ,
Lab‫ ناسيه‬:
‫ او يمكن‬- ‫بس كانها هيمولتك يرميك سيندروم‬E.coli
What ttt :
-steroids
-Ab

Q/ Case : patient wake up with edema , under eye + oral dehydrated ,


Lab : hypoalbuminemia , hyperlipidemia ,
What ttt‫ وطالبين العالج‬، ‫? كيس ي نفروتيك او نفريتيك‬
-asprin
-predinsone

October 24, 2017 103


-Cyclophosphamide
-Cyclisprine

Q/ What of the following use scotch tap in diagnosis ?


-Giardia
-Malaria
-Schistosomiasis

Q/ Case: newborn has tetany after delivery calcium: 1 ( normal 2-3)


One year later , he has recurrent upper respiratory infection with : hip
, hsv , pneumocystic jovirci – asprigellus What is underlying his
condition?
-q 22 depletion ( smth like that)
-Depletion of cd4 lymphocytes
-Wiskott dis
-NADPH oxidation .

Q/ What age in month for a Child can say few words:


6-
9-
21 -
14 -

‫ الدكتور عشان يطمن االم وش الشي الي‬، ‫ وامه خايفه انه متاخر بالنمو‬، ‫ شهور‬٧ ‫ واحد عمره‬: /Q
‫يساله عنه ؟‬
-pincer grip
-Reach objectsQ/
-Wave goodbye

October 24, 2017 104


-Sit

Q/ ‫ كم عمره بالشهور ؟‬، ‫ ويضحك‬، ‫طفل يركض اول ماشتف الدكتور‬


6-
9-
21 -
14 -

، ‫ وكل شي نورمال ماعدا عالمه عضه الكلب بيده‬، ‫ وجا الرت وواعي‬، ‫ واحد عضه الكلب‬/Q
. ‫ قبل سنه ونص‬rapies vaccine ‫وقالك هو اخد‬
‫وش تسويله ؛‬
-give rabies vaccine
-Give to dose of rabies vaccine
-Observe for 10 days
-Rabies immuniglublin and vaccine

Q/ Female , came for counsling for pregnancy, she has HSV when she
was child , what to do :
-MMR vacine
-Rubella titerQ/
-Vericella vaccine

Q/ What of the following chrachtrstic for TOF :


-pulmonary stenosis
‫ اشيا ثانيه خطا‬...- -

October 24, 2017 105


Q/ 2 months y.o girl , refuses feeding , she gets cyanosis and crying
whenever she feeds , on auscultation: harsh ejection systolic
murmure : what dx:
-Aurtic stenosis
-Ventricular defect
‫ ناسيه الباقي‬-

Q/ Heel pain , especially in morning , decrease with exercise :


-tendonitis faciitis
-Achillis tendon repture .

Q/ Pregnant , How much folic acid required in milligram:


2-
1-
0-
4-

Q/ What is contraindications of instrumental delivery :


-placenta abruption
-Breach presentation
-Cephalopelvic dispropotion
-Face presentation

Q/ Baby , during delivery , there was stylomastoid formen trauma ,


what features will be during exams:
-loss of eye close
-Loss of facial sensation
-Loss of mastication function

October 24, 2017 106


Q/ Pt present with barky cough , what most likely finding in physical
examination:
-Stridor
-Wheeze
‫ اشيا ثانيه‬-

Q/ Barking cough , what suspected in auscultation :


-Inspiratory sounds due to sub glottic stenosis
-Whezing
-Inspiratory sound due to smth

Q/ Patient trauma to obturator nerve , which muscle will be fully


paralyzed :
-Adductor longus

Q/ Pregnant women ...... fetal heart dropped from 140 to 80 what to


do ?
-GA

Q/ Where to palpate dorsalis pedis artery , lateral to


-halusis longus7
-Digitorm longus

Q/ Which one of these nucleius made in nucleoulus ?


-rRNA
-tRNA
-mRNA

October 24, 2017 107


Q/ Which of the following intergates glocuse and fatty acid
metabolism
-pyruvate
-Citrate
-Carpoxylase

‫ اسئله عن‬١LDL
Q/ elderly patient with dm, HTN , dyslipdemia , what is the target LDL
?level for him
4.6-1.0 -
. ‫ ارقام غريبه بالفواصل جت‬-

Q/ what is the normal level of LDL

Q/ patient with CAD , prescribe for her drug lower lipid . Present with
facial flush , what is the drug ?
-statin
-Nicain
-Fibrate

Q/ Case of ADHA : child present with poor complaince in school , not


doing his homework, doesn't complete his game , not following
teacher command ,
What type of ADHA he has?
-impulsive
-Overactive
-Overactive,impulsive

October 24, 2017 108


‫واحد جاي باعراض دبرشن ‪ ،‬لما ساله الدكتور عن اعراض السايكوسيز‪ ،‬توتر وترك المستشفى ‪Q/ :‬‬
‫وش يسمى هذا ؟‬
‫‪-acting out‬‬
‫‪-Intellectual‬‬
‫‪-Siblimation‬‬
‫‪-Interjection‬‬

‫جا نفس السوال ‪ ،‬الي بيشنت عنده اعراض سكيزوفرنيا من شهر ‪ ،‬ودكتور شخصه بسكيزوفرينا ‪Q/ ،‬‬
‫الخطا في التشخيص ‪ :‬وش‬
‫‪-Duration .‬‬

‫‪ /Q‬طفل مايسمع كالم امه ‪ ،‬وبس يضربها وشقي يعني ‪ ،‬اخر السناريو كاتبين ان االم واالب انفصلو‬
‫وش التشخيص ؟‬

‫‪ , frontal bossing , rosary‬وذاكرين في الهستوري ‪ /bowing legs‬صوره اكس راي ‪Q‬‬


‫‪chest‬‬
‫‪Which of the following vit will be defecient :‬‬
‫‪-Vit D‬‬

‫‪ /radius + ulna ,‬نفس سوال البري تست ‪ ،‬الي عنده كسر في ‪Q‬‬
‫‪what to do‬‬
‫‪-irrigation 7‬‬
‫‪-Close reduction‬‬

‫‪Q/ Dm pt , with chest pain for 3 days , dyspnea with exertion , history‬‬
‫‪of URTI .‬‬
‫‪ECG : t depression‬صوره‬

‫‪October 24, 2017‬‬ ‫‪109‬‬


Lab: troponin : high
What is the dx?
-myocarditis
-Mi
-Constrective pericarditis.

Q/ 21 yo , Non smoker , present with wight loss and dyspnea , he


works in electricity generation
What is the cause :
-Asbestose
-Inhaled coal dust
-Passive smoking ?

Q . ‫ لهم سنه يحاولون‬، ‫ واحد جا العياده النه زوجته ماحملت‬/Work in batteries factory
Semen analysis : oligospermao
Cbc: heavy metals on RBC , what the cause
-Nikle
-Lead

Q/ Young Patient with hematuria , week later he developed


hemoptysis , what is the dx:
-Goodpasture
-HSP
-PSGN

Q/ 2 yo girl , present with general malaise . Bone aches especially in


hand and foot ,
ANA: high

October 24, 2017 110


ESR: high.
What is the cause ?
-latent lupus
-Growing pain

Q/ ‫ اعراض لوكيميا‬:
Lab shows : blast cell ; high
Aur rode in blood film , what is dx:
-AML
-Aplastic anemia
-CML

Q/ child present with malaise , history of meningitis last week treated


with iv ab .
Lab : hb: low
Rbc: low
What antibiotic he used ?
-chloramphinicol
-Erythromycin

‫ يكون معهم رسك الي كانسر؟‬PCOS‫ ال‬: /Q


-ovary
-Endometrial
-Breast
-Cervix

، ‫الثيرويد جاني اسيله كثير‬

October 24, 2017 111


: ‫ سوال البابيالري ثايرويد كانسر‬، ‫ نفس الملزمه‬/Q
‫كان في االجوبه‬hurthle cell.

needle aspiration : ‫ وش تسوي اول شي‬، ‫ عنده ثايرويد نديول‬.‫ واحد يدخن ويشرب كحول‬/Q

‫ ايش كان يستخدم ؟‬.fever ( infections ) ‫ سؤال البيشنت اللي على انتي ثايرويد وصار عنده‬/Q
Methimazol

Q/ pt with hyperthyroidism, with palpitation , what is the most likely


finding in his ECG ?
-SVT
-VT
-A FIB

Q/ ‫ وش عندها ؟‬، ‫ طلبو تحليل الثايرويد وهذي النتيجه‬، .‫ بس ماقدرت‬، ‫وحده تحاول تنقص وزنها‬
TSH: high
T4 normal
-hyperthyroidism
-Subclinical hypothyroidism

Q/ Pt k/c of hypothyrodism , use levothyroxin 175 mg , swith to 200


mg . Labs : Tsh : high t4 : normal What is the cause ?
-ectopic thyroid
-Primary hypo
-Secondary hypo
-Small dose .

October 24, 2017 112


Q/ ، ‫سوالين عن الهايبباراثيرويد‬
. ‫ وستون في اليورن‬، ‫اول سوال كان وحده عندها رنال كوليك‬
، ‫لما طلبو تحاليل‬ca : normal
PTH : high .
What is the dx ?
-hyperparathyrodism .

Q/ Pt known case of asthma , his wife has osteoporosis, he has renal


stone , he eats multivitamin, vit D and calcium .
PTH : high
What is the dx ?

Q/ How hyperglycemia affect wound healing:


-dec imuune system
-Dec phagocytosis
-Stimulates bacteria growth

Q/ Leg ulcer , with rolled out edge :


-SCC
-Basal cell

Q/ Most common cancer mets to stomach:


-lung
-Prostate
-Melanoma

Q/ Smoker developed ulcer lateral to his tongue rolled out edge :

October 24, 2017 113


-SCC
-basal

Q ، ‫ جاني سناريو عن االنروكسيا نرفوزا‬/patient admitted to ER , she denies


eating and says she's not hungry, BMI 11.3
What most likely to find in her labs:
-inc K
-Dec Creatnin
-Other labs

Q/ Patient admitted to er , upon examination: dental smth :


Anorexia
Pulomia nervosa

Q/ Blunt tauma in antirior abdomen with pancreatic injury and


enzymes leak where the enzyme will go:
A) omentum bursa .
B) sub hepatic space
C) sub phrenic space
D) left paracolic

Q/ old man with abnormal involuntary hand and head movement he


take drug 1 day before what is the drug:
A) Chloramphenicol
B) Digoxin
C) ethambutol

October 24, 2017 114


Q/ MVA victim in the ICU developed multiorgan failure lab show high
phosphate what is the organ responsible:
A) Heart
B) Liver
C) Kidney
D) Lung.

Q/ Pt present with unilateral throbbing headache with photophobia (


migraine case ) what is the ttt ?
-triptan ‫ بس من قروبه‬.‫( مو تربتان‬
-Others

Q/ sign for low intestinal obstruction:


A) altered bowel movement .
B) absolute constipation
C) flatulence
D) diarrhea.

Q/ Patient with deoudnal ulcer , what ttt ?


-triple therapy
-Omeprazole

Q/ Most common site for meckle diverticulum:


-terminal ilium
‫ اماكن ثانيه خطا‬-

Q/ Clear case about crohn dis : child with abd cramps , diarhea .... etc
, there is skip lesion , transmural :

October 24, 2017 115


-crohn
-Uc
-Celiac .

Q/ Similar case : child with abd pain and tenderness , bloody diarhea ,
wt loss
:‫ وش التشخيص‬، ‫مو حاطين الب وال شي‬
-crohn
-Celiac
-Uc

Q/ How to manage uncomplicated sigmoid vulvulus ?


-flatu tube
-Nasal tube
-Surgery

Q/ Old man with change bowl habits , LLQ pain , tenderness ,


swelling , wt loss , what is the most likely dx:
-diverticulosis
-Diverticulitis
-Cecum ( or segmoid ) ca

Q/ Patient with dm , which one of the following is contraindicated:


-losartan
-Nifedipine
-Hydrazine

October 24, 2017 116


Q/ Pt with filling defect , us , ecoech shadow ، ‫نفس سؤال الملزمه الي اختلفو فيه‬
: ‫وش نوع الستون‬
-uric acid stone .

Q/ Pt with uric acid stone , what manegmt to prevent recurrence:


-allopurinol
-Erythromycin

Q/ Pt with gout , what should measure :


-uric acid level .

Q/ How heparin in high dose cause hyperthermia


-inc metabolism
- -phosphrylation
-Increase liver metabolism
-Increase energy

Q/ Pt with severe epigastric pain radiates to back , labs : amylase:


high , alp : high , ggt ; high , direct billirubin high , what is the cause
of his condition?
-gallstones

Q/ female with bilateral menstrual breast pain o/e bilateral breast


nodularity and mobile mass in the right breast axillary lymph free
what to do next:
A)mammograph then US
B) aspiration and cytology
C) Reevaluate at the next cycle

October 24, 2017 117


Q/ pt with only palpitation examination normal ecg show heart block
what is the treatment:
A) CCB
B) beta blocker
C) digoxin

27 / 9 / 2017
Q/ Carpopedal spasm > low ca

Q/ Vit.c > iron absorption

Q/ Good healing > epithelium covering

Q/ Ectopic pregnancy scenario > early disappearance of zona pillucida

Q/ Primary dysmenorrhea scenrio

Q/ Clonoscopy after > 5/10 years

Q/ Coil speing > nonsurgical management & bowel rest

Q/ Orchitis schenario > treated by antibiotcs

Q/ Case presentation typical for Appendicitis

Q/ Gerd in child > give antacid

October 24, 2017 118


Q/ Post Mi pt want to drive > should at least after 4 weeks

Q/ OR 5.2 ( high fat diet pegnant ladies comparison )

Q/ HbA1c target for dm2> less than 7

Q/ Targer diastolic Bp in HTN retinopathy > 8090 in 2 days

Q/ Metformin moA

Q/ PTU moa > block release of thyroid hormones

Q/ Trauma to abdomen Omentum out > beed immediate lapratomy

Q/ 4 Qs Naloxone & opiate ( morphin )

Q/ NAC max. Hrs > given after 8 hrs

Q/ Trauma RTA pt got sever hemorrhage


Which decrease > HR

Q/ as a doctor you invited to talk about new drug > refuse

Q/ Achills fat > LDL reseptors

October 24, 2017 119


Q/ Drop foot & loss of sensation in 1st & 2nd digit > Deep pernoneal
nerve

Q/ MG presentation ( faatigue at the end of the day > AB against


receptors Ach

Q/ ESLD yeast > fluconazole

Q/ White vaginal discharge > micoconazole cream

Q/ Pseudotumor of the eye > steroids


+ ve PPD , ve CXR > start INH 6 m

Q/ Exacerbation of asthma > inhaled steroids

Q/ Child with DM1 got multiple hypoglycemic attacks > due to hony
moon period

Q/ Scenario of sever pre eclampsi given MGSO4 .. reflex is +1 >


magnesium sulfate toxicity

Q/ Mcc of honymoon cystitis > E.coli

Q/ Gas gangrene > cl.perfeenges

Q/ Gve bacilli oxidase , urease

Q/ Diarrhea , abd pain , fever moa of the drug > DNA gyrase

October 24, 2017 120


Q/ Protein last destination > ER

Q/ Spleenomegaly , high hg , plt > polycythemia rubra vera ( not sure


)

Q/ Hx of Palpitation & SVT (ECG ) > treated by deltiazim ( my answer


not sure )

Q/ Overlflow incontinance Rx Moa > activate muscarinic receptors (


muscarinic agonist )

Q/ stress incontinance with weak detrosal muscle > kegal exercise


Mickle diverticulm > iluem

Q/ Symmetrical MCP & PIP pain increase at morning > RHeumatoid


arthritis

Q/ Knee pain increase by walking on the stairs releived by walking


on flat floor this pain aggrevated by contracting quadriceps muscle >
Osteoarhtritis

Q/ Thrombocytopenia, high Crt & Bun


With purpra ( adult age ) > TTP

Q/ Scenario of hepatorenal syndrome

Q/ Foley cathetrization > mostly injured membranous urethra

October 24, 2017 121


Q/ Chance of Hemophili A to the next baby > 50%

Q/ Fluride supplmenets > aganst dental carries

Q/ 4 m > smile

Q/ Parkinson case > low dopamin

Q/ Another case of parkinson > problem in substantia nigra

Q/ draw a line > 18 month ‫كانت اقرب اجابه‬


‫ما كان فيه سنتين وهو اعلى رقم‬

Q/ Myopia with pathology > pathological myopia

Q/ Methemazol > they mention long scenario with low readings

Q/ Cold intolerance, wt gain > check thyroid Us

Q/ Renal mass ( loin pain & hematuria & wt loss ) > best to do ct

Q/ HRs > uremia , hemarutis , Qued. Abd pain

Q/ HSP > abx

Q/ Overlflow incontinance Rx moa > activate muscarinic receptros

October 24, 2017 122


Q/ Lung ca > 70%
66:1
191.0

Q/ Post.hip dislocation

Q/ Compl. Of mastoiditis

Q/ Minimal change NS in which age

Q/ Intussception age group

Q/ Pvalue

Q/ B.vagonisis screening in pregnant with hx of pre term delivery

Q/ T.diarrhea > shegella

Q/ The only organism from human source

Q/ Most common site for central line infection > insertion

Q/ Compination of antipsychotic cause constipation

Q/ Dementia rx

Q/ Delerium

October 24, 2017 123


Q/ Osgood schaltter presentation with prepatellar hematoma ?

Q/ 9month vaccines ? Measles & menincoccal

Q/ When baby smile ? 2 m

Q/ Cryptococcal meningitis my answer by exclusion ( CSF readings


were with high both lymphcytes & neutrophils high protien normal
glusose )

Q/ Toxoplasmosis encephalitis in hiv scenario ( scenario of HIV pt


with neurological symptomes )

Q/ Retinal detatchment ( pt got trauma presented with present with


inferior v. feild affected , floaters )

Q/ Nevus creptus something like this ( child wigh scalp swelling with hair
loss above it )

Q/ Vaccine contraindicated in eczema , psoriasis , cobtact dematitis >


rubella ( german measles )

Q/ rubella presentation case

Q/ burkitt ( pt from ghana )

Q/ Repetitve idea scenario pt Knows this is wrong > obsession

October 24, 2017 124


Q/ Scz in 1 month , against the dx ? > duration of the disease

Q/ PTU MoA

Q/ Do periphral Blood smear ( pt from sudan )

Q/ Hydrop fetalis presentation ( Abnormal 4 alpha & 2 normal beta )

Q/ Stroke presentation ( unilateral body weakness & slurred speach


in 1 hr ) what's next step ? CT of the brain

Q/ RCC ( as presentation of hematuria, wt loss , loin pain ) > VHL

Q/ Breast ca > BRACA2

Q/ Ocp ? Stop spermatozoa and i increase thickness of cervical


mucosa
Becareful they don't prevent ovulation , actually they preven implantation
but this word not mentioned and all options with ovulation prevention so all
wrong

Q/ Hormone replacement therapy > increase risk of endometrial ca

Q/ Orchitis scenario

Q/ Facture of the tibia with wound oozing > surgical debridement & intra
medullary nail

October 24, 2017 125


Q/ Deep foot ulcer > surgical debridement & admittion

Q/ Pt got numbness and electrical like pain of the left foot up to knee
, so at which level is the disc ? L3L4

Q/ Bond tRNA & mRNR > hydrogen

Q/ Made in the nucleus > rRNA

Q/ Protein synthesis end > ER

Q/ Lactic acidosis occur with > defeciency of pyruvate dehydrogenase

Q/ Longest cellular phase > interphase

Q/ NF1 > ch.17q ....

Q/ NF case presenationn> autosoma

29/9/2017
Q/ Most common malpresentation:
-braw
-breech*
-face
-transverse

October 24, 2017 126


Q/ case scenario about postmenouposal atrophic vaginitis and ask
about treatment:
-topical steroid
-topical estrogen*

Q/ case scenario about female with past hx of PROM and now she is
pregnant asymptomatic and ask about indication of screening for
bacterial vaginosis:
-no indication
-screen at first trimester
-screen at second trimester
-screen at third trimester.

Q/ MAFP measure in which week:


20 -weeks
20 -weeks
2, -weeks*

Q/ female patient pregnant present with abdominal pain and vomiting.


Vital sign normal except blood pressure 160/100.
What is the diagnosis:
-renal failure
-fetal distress
) no preeclampsia in the choices (

Q/ scenario about female with hypothyroidism and take levothyroxine


for something duration, she is asymptomatic now.

October 24, 2017 127


The lab result : T4 within range , TSH still high 1 above normal range ,
what will you do:
-continue with the same dose
-increase dose by.…
-Increase dose by…
-Increase dose by…

Q/ Pediatric case scenario indicate respiratory distress syndrome ,


ask about the deficiency:
-sphingomylin
-dipalmitoyl phosphotidylcholine*

Q/ case scenario about mickles diverticulum the surgean will look for
diverticulum in which part:
-ileum
-cecum
-jejunum

Q/ picture of man with wound on the leg and x ray show fracture in
the tibia asking about management:
-open reduction
-close reduction
-depridement , antibiotic and intramedullary nailing.

Q/ Female with diffuse thyroid swelling and dominant single nodule ,


lab test result was increase in T4 and decrease TSH what will you do:
-radionuclide scan*
-FNA

October 24, 2017 128


-US
-thyroidectomy

Q/ contraindicated hypoglycemic drug in diabetic patient with heart


disease:
-pioglitazone*
-sitagliptin

Q/ case scenario about patient after dental extraction c/o of loss of


sensation below the eye and upper lip , I think also cheek which
nerve was injured?
-infraorbital*

Q/ aneasthesia provide sedation with subaneasthetic dose:


-midazolam
-ketamine

Q/ loss of sensation of anterior 2/3 of toungue:


-facial
-trigeminal*
-glossopharyngeal
-hypoglossal

Q/ sparing and dissecting which artery during flap of rectus muscle?


-inferior epigastric artery*

Q/ which of the following is made in the nucleolus?


-rRNA*

October 24, 2017 129


Q/ athletic came for regular check up . there was xanthelesma in his
achillis tendon , he had high total cholesterol , the defect in which
gene:
-LDL receptor*

Q/ Case scenario about sign and symptoms of hydrops fetalis and


asking about gene defect?
1 -normal B chain , 4 abnormal alpha chain*

Q/ which defuiciency of following result in lactic acidosis?


-pyruvate kinase
-pyruvate dehydrogenase
-pyruvate carboxylase^

Q/ which medication decrease the effect of OCPs?


-anti epileptic*

Q/ patient presented to ER with hx of drug overdose for last 8 hours


on the examination the gag reflex was absent . the best management
is?
-Iv naloxone
-Immediate endortcheal intubation*
-Gastric lavage

Q/ Patient present to ER with typical symptoms of Ischemic heart , he


had hx of erectile dysfunction he is on PDE-5 inhibitor (sildenafil),
which of the following drug should not be given:
-Nitroglycerine*

October 24, 2017 130


-Metopralol
-Dilitazem

Q/ Patient with controlled DM and HTN he is on ACEI , furoseamide


and spironolactone , electrolyte was normal what should you do?
-stop spironolactone*
-stop fruseamide

Q/ case scenario about DM patient with necrotizing fasciitis , what is


antibiotic should take?
-penicillin/gentamycin
-piperacillin/ tazobactam*

Q/ Diabetic patient with deep ulcer in the sole of foot ask about
management :- admission and debridement*

Q/ Case scenario child with sign and symptoms of meningitis gram


stain and culture result was gram +ve cocci what is the treatment:
-ceftriaxone and vancomycin*

Q/ female patient with hX of multiple abortion and D&C want to get


pregnant what is the diagnosis:
-asherman syndrome*

Q/ patient with two time ASCUS pap smear what do next:


-colposcopy

Q/ which of the following factor can decrease risk of some cancer?

October 24, 2017 131


-fiber
-vitamin D

Q/ case scenario about patient with basal skull fracture what


expected nerve injury?
-Olfactory
-optic
-oculomotor

Q/ case about patient with osteoarthritis had enlargement on distal


interphalangeal joint what is it?
-heberdens node*
-bouchard node

Q/ type of hip joint:


-ball and socket joint

Q/ case scenario about female patient with swelling in labia majora


drainage was done 2 time and she ask the doctor about treatment to
prevent the recurrence ? what will he do?
-aspiration
-marsuplization*
-drainage.

Q/ 24 years old female with hx of hot flushing and night sweating and
amenorrhea what the diagnosis?
-hypothyrodism
-congenital adrenal hyperplasia*

October 24, 2017 132


Q/ how ectopic pregnancy occurs at the cellular level?
-disappearance of zona pellucida*

Q/ pregnant women during labor , Iv oxytocin was given the CTG


show variable and acceleration what will you do?
-stop oxytocin*
-expectant delivery
-change mother position

Q/ giving birth , cervix 6 cm dilated , fetus is left occiptoposterior ,


sign of molding can be felt what the stage?
-first*

Q/ patient diagnosed with scoliosis according to the cobb angle ,


when the orthopedic refer should be done?
23 > -
*13> -

Q/ injury to the surgical neck of humors lead to injury to which nerve


?
-brachial
-axillary

Q/ 31 something old patient present with back pain and tenderness


when palpating the paraspinal muscle , neurovascular was normal
and ask about treatment?
-physiotherapy*
-bifedback

October 24, 2017 133


-surgery

Q/ pain in snuff box most likely indicate fracture in which of the


following?
-scaphoid*

Q/ case scenario about patient with bailetral shoulder and hip


stiffness and ask about the diagnosis:
-polymyalagia rheumatic*

Q/ which tendon medial to dorsalis pedis artery?


-extensor halluces longus tendon*.

Q/ what is the presentation of posterior hip dislocation?


-adduction and internal rotation*

Q/ sign in Duchene muscular dystrophy?


-growers sign*

Q/ picture and scenario of patient with hx of URTI complain of eye


tearing and redness , also what will you find:
-papilla
-follicle*
-papilloma

Q/ patient with scenario of close angle glucoma whats the treatment:


-IV acetazolamide + pilocarpine*

October 24, 2017 134


Q/ patient asymptomatic for regular check up fundoscope show disc
cupping ask about diagnosis?
-DM retinopathy
-HTN
-Open angle glucoma

Q/ patient presenting to ER complaining of eye pain , swelling and


sudden onset of proptosis, eye movement was limitied but vision was
intact whats the diagnosis?
-orbital cellulitis*.

Q/ pregnanat women during labor , spontenous rupture of membrane


, suddenly baby is stress from 140 to 80 what type of aneasthesia
whould you use?
-GA*
-Pedundal

Q/ tonsilits case which AB will you use?


-amoxicillin/ clavilonic*
-vanco
-cipro

Q/ organism associated with animal bites?


-pasteurella multocida*

Q/ IV drug abuser complain of oral thrush , signs and symptoms of


pneumonia , broncho alveolar lavage was done pneumocysticitis
jiroveci was found then HIV test was done and it was +ve , what is the
most likely predictor of her HIV infection?

October 24, 2017 135


-pneumocysticitis jiroveci*
-IV drug use
-Candida

Q/ case scenario of patient with Hx of URTI lab result : low Hb , high


WBc I think reticulocyte was within range or decrease , what is the
investigation:
-electrophoresis
-bone marrow biopsy

Q/ also another Qs with the same scenario ask about diagnosis?


-aplastic anemia
-hypoplastic
-immune hemolytic

Q/ treatment of travelers diarrhea?


-cipro*

Q/ patient with HIV and ask about something I forget ^_^ but the
choices was:
-memory B cell
-CD 8 T cell
-Macrophage

Q/ picture of patient with rash involve buttocks associated with


bilateral knee pain ( HSP ) and ask about treatment?
-IVIG
-Corticosteroid

October 24, 2017 136


Q/ pediatric QS and ask about most common congenital
something?*_*
-cryptorchisim
-hypospadias
-micropenis

Q/ pediatric case with vomiting after feeds , with good appetite and
want to eat despite vomiting during examination found epigastric
swelling whats the next?(pyloric stenosis)
-abdomen US*
-x ray
-barium enema

Q/ which muscle will completely paralyzed if obturator nerve was


injured?
-adducror longus*
-adductor magnus
-membranous

Q/ case scenario pediatric patient with abdominal pain and vomating ,


scenario of intussuception and ask about best initial treatment?
-Iv fluid and bowel rest*
-Surgery
-Barium enema

Q/ patient with irregular menses BMI was 16 she think that she is fat
and want to lose weight what the diagnosis?
-bulimia nervosa

October 24, 2017 137


-anorexia nervosa*

Q/ what move molecule in and out the cell nucleous?


-cytosolic ATP
-nuclear ATP
-cytosolic GTP
-nuclear GTP*

Q/ boy wiith hip pain x ray shows degenerative and collapse of


femoral neck what the diagnosis?
-perthe disease*

Q/ common cardiac anomaly associated with infective endocarditis?


-ASD
-VSD
-TOF

Q/ patient with hallucination and delustion for 1 month and the dr


diagnose him as schizophrenia what is the wrong?
-duration of symptoms*
-age of the patient

Q/ equation of relative risk:


-(a/a+b)/( c/c+D)*

Q/ case scenario about female patient with greenish discharge , the


microcroscope show motile flagellated organism?
-Trichomonas*

October 24, 2017 138


Q/ another case scenario of female with cheesy white discharge , she
had hx of taking AB for something , what is the diagnosis?
-candidiasis*

Q/ case scenario regarding patient known case of epilepsy , he take


anti-epileptic drug for long time and now he complain of increase hair
in his body , what do you think of the following medication he is on?
-phenytoin*

Q/ You performed a pudendal nerve block on a woman in labor, which


of the following structures will be fully sensitive and not blocked by
the anesthesias
?

A. Perineal body

B. Urogenital diaphragm
C. Rectum*

Q/ female pregnant with small fibroid what to tell her about the risk?
-mostly asymptomatic
-risk of preterm labor
-degeneration is common

Q/ 41-something female patient with bilateral cyclic breast pain and


small mobile bilateral nodule , the axillary lymph node was free what
will you do?
-mammograph then US
-aspiration and cytology
-reevaluate at the next cycle

October 24, 2017 139


Q/ case scenario I think regarding breast mass and asking about
what thing indicate is cystic or no need for re-evluation:
-FNA- clear fluid on aspiration and disappear of the mass after*.
-Bloody fluid and disappear of mass
-Clear fluid and reappear of mass.

Q/ case scenario regarding breast mass and asking about which


lymph node must be assist first?
-anterior axillary lymph node
-posterior
-medial
-lateral

Q/ newly diagnosed patient with DM came for appointment and you


came late 1 half hour because you had a critical case the patient was
angry what to do?
-aknowladge the patient anger
-explore the patient anger
-talk to the patient about his anger
-empathy with the patient

Q/ glomerulonephritis with ani-basement membrane antibodies?


-good pasture syndrome*.

Q/ prognosis of breast cancer dpend on?


-number of axillary lymph node
-estrogen receptor
-age of the patient

October 24, 2017 140


Q/ picture of patient with wheal in fore arm , and complain of hand
pain after bee sting associated with enlarged axillary lymph node
what is the diagnosis?
-urticaris
-lymphangitis

Q/ case scenario regarding HUS and sking about treatment?

Q/ case scenario regarding nephrotic syndrome and asking about


treatment?

Q/ about steven jonson syndrome is most likely occur in which


group?
-HIV patient take penicillin
-Patient with something take anti epileptic

Q/ about neurofibrosis which gene?


2,-q11.2*

Q/ which part of urethra commonly injured by insertion of foley


catheter?
-membranous*

Q/ case scenario about patient with hypertention , labs result


hypernatremia hypokalemia and asking about cell responsible for
that?
-fasiculata
-glumerulosa

October 24, 2017 141


-reticularis

Q/ pediatric case , asymptomatic patient ECG , I just noticed irregular


rythem bradycardia and PR interval was unequal some lead was 3
little square and another 5 little square.( I think case of 1st degree
heart block )Then they asked about what future sequence?
-syncope
-need for pacemaker

Q/ daily requirement of iron in pregnancy?

Q/ In a study they are selecting every 10th family in the city, what is
the type of study


- .Systematic 
study
*

- .Stratified study


- .Non randomized study


Q/ In a cohort study on lubricant oil use and urinary bladder CA done


over 20 years 10,000 exposed 10,000 non exposed 750 exposed got
CA 150 non exposed got CA Then they asked about the incidence in
1000 in one year?
- *
1.10
- 40
- 
40.
- 
110.
Q/ Lung cancer affected 80 of 100 smokers,6 of 500
nonsmokers,What is Relative risk reduction
?

00 -:1

00 - :1
66 -:1*

October 24, 2017 142


Q/ 0 qs about type of bias I forget is it for case control or cross
sectional study.

Q/ how to prevent MERSA?


-by hand washing*
-vaccine

Q/ hepatitis diagnosed by?


-blood test ( I choose it )
-imaging
-Hx

Q/ treament of leishmania donovani?

Q/ prophylaxis of malaria for man want to travel to * something?*

2/10
Q/ Patient k/c of HF & schizophrenia on medication ,came with
brownish discoloration in retina & brown vision.. what's the drug?
Thioridazine

Q/ Migratory thrombophlebitis in ?
Pancreatic cancer (Trousseau sign)

Q/ Confidence interval of 95% means ?

October 24, 2017 143


%90 of the results are in the interval or something similar♀🤦🤦

Q/ treatment of leishmania?
There was no amphotrpcin B

Q/ Visceral leishmaniasis?
L.Tropica

Q/ 2Qs about alternative cell reservoir for HIV ?


Is it CD 8 ?! I donno♀🤦🤦

Q/ Hypoparathyrodism ,he will have?


Convulsions

Q/ 21Y/O came with urinary retention ,1st step ?


Urinary catheter

Q/ Latency period in HIV,cell responsible?

Q/ URTI came with conjunctivitis ,to prevent infecting the other eye ?
Hand washing

Q/ Source of infection in venipuncture ?


Site of insertion

Q/ 4direct cases about diagnosis of tension headache

October 24, 2017 144


Q/ case about cluster headache ,what is the treatment?
Ergotamine
Paracetamol
‫ما كان في ادوية اعرفها للكلستر هيديك‬

Q/ prevention of cluster headache ?


Verbamil

Q/ case about DKA ,worst complication?


Hypoglycelia

Q/ testament of thyrotoxic crisis ?


Propranolol

Q/ bond between mRNA & tRNA ?


Hydrogen

Q/ prevention of brucellosis?
Pasteurization of milk

Q/ water with low iodine ,cheer-man wants to treat children suspected


to have cretinism?
Add iodine
Levothyroxine

Q/ osteoporosis most commonly due to ?


Aging

October 24, 2017 145


Q/ rRNA in which cell?
Basophils
Monocyte

Q/ energy through the cell ?


GTB hydrolysis in the cytoplasm

Q/ x-ray and case about perths disease

Q/ bias in meta-analysis?
Response
Recall
Attribution
Publication

Q/ different Qs about type of study needed

Q/ researcher want to start a study but he needs only ppl free of DM


,which will be high in the test ?
Specifity

Q/ most common benign bleeding lesion in postmenopausal?


Cervical polyp
Atrophic vaginitis

Q/ postcoital bleeding ..in?

October 24, 2017 146


Cervix

Q/ most malignant lesion ,needs to be removed ?


Erthyma migrans
Erythyma marginatum
Erythema gyrum
Erythema annulare

Q/ most common type of eczema ?


Cold
Soles

Dermatogeografic ..something like that��♀😂💔

Q/ itching in the face ,cupital and popliteal fossa?


Eczema

Q/ dyphagia ,swollen tonsils,inability to swallow anything ,what will


decrease the swelling in pharynges ?
Predenisolone
Aspirin

Q/ case of streptococcal pharyngitis ,treatment?


Aspirin
Penicillin

Q/ 2 y/o had bone marrow transplantation 18 months ago ,, his


brother developed fever and vesicular rash ,,what will u give the 4
years old as a prophylaxis?

October 24, 2017 147


IV Acyclovir
Varicella immunoglobulin
Varicella vaccine
Both B&C

Q/ injury in the sole ,1st structure injured


Abdactor hallusis longus

Q/ side effect of amitryptaline ?


Dystonia
Diarrhea

Q/ patient with hypertrigelicridemia ,came with abdominal and back


pain ,his symptomes due to ?
Hypertrigylesride

Q/ pt on statin for high cholesterol ,now hight trigylesride ,what to


add?
Fibrate

Q/ case about RA came with bilateral swelling in DIP due to ?


Synovial thickening

Q/ girl on retinol ,must warm her about ?


Sun exposure

Q/ different type of acne ,pustule .. what type of acne ?

October 24, 2017 148


Inflammatory

Q/ contraindications for breast feeding?


HCV
HIV

Q/ disease increases the mortality in pregnant female ?


Pheochromocytoma

Q/ cast about osgood schlatter disease,,came with bilateral knee


swelling ..what is the cause ?
Osteochondritis
Hemorrhage in the bursa
ACL tear

Q/ Tenderness in the snuff box?? Scaphoid fx

Q/ Aspirin toxicity?? Respiratory alkalosis and metabolic acidosis

Q/ The maximum time for giving n.acetylecystin in acetaminophen


toxicity is ??
8h

Q/ What happened in acetaminophen toxicity ??


Depletion of glutathion store

Q/ Final destination of protein is Er membrane

October 24, 2017 149


Q/ Which of the following synthesize in nuecleolus is?? rRna

Q/ Median nerve :ape hand..thenar atrophy

Q/ Surgical neck fx : axillary nerve

Q/ Great auricular nerve supply

Q/ Posterior vagan nerve supply

Q/ Dorsalis pedis is lateral to extensor hallucis longus

Q/ Brunner's gland present in dudenal submucosa

Q/ Numbness in ring and little finger after raising the arm due to
thoracic outlet syndrome

Q/ Murmer of mitral stenosis is mid diastolic

Q/ Feature of tetrallogy of fallot..pulmonary stenosis ..defect in


interventricular septum

Q/ Cholecystitis case treatment

Q/ Spherocytosis ask about the protein defecient which is spectrin


and some thing else

October 24, 2017 150


Q/ Iron defeincy anemia

Q/ Vitamin b12

Q/ Seidroblastic anemia

Q/ Scenario seems to be sarcoidosis

Q/ Bahcet disease ask about pt having erythema nodosum taking


steroid cream but not improving ..i think the answer is colchicin...

Q/ Ligament that prevent uterine prolapse is uterosacral ligament

Q/ Regimn of h. Pylori

Q/ Case of entoameaba histolytica

Q/ Kallmann syndrome which hormone would you like to order is


gonadotropin

Q/ infertility case ask about which factor affected in these cases


..anovulatory factors

Q/ Day of ovulation in menstraul cycle of 34 is 20

Q/ Burkitt lymphoma gene involved

Q/ Neurofibromatosis follows autosomal inheritance

October 24, 2017 151


Q/ Tumor in upper pole of kidney risk factor VHL

Q/ Posterior hip dislocation feature internal rotation and adduction

Q/ Feature of fragile x syndrome is macrorchodism

Q/ Pneumothorax treated by needle decompression

Q/ Site of needle decompression is 2nd intercostal mid clavicular


line
Q/ Virus cause meningoencephalitis is herpes simplex

Q/ Feature of herptic genital lesion is multiple filled vesicles

Q/ Herpes zoster shinglle is band like distribution with burning pain

Q/ Mid stenotic mass in esophagus is SCC

Q/ SX AND SIGN OF PERFORATED DUDENAL ULCER IS epigastric


pain

Q/ Pcos pt is at increased risk of endometrial cancer

Q/ Case of ischemic colitis

Q/ Percentage of multifactorial disease ( cleft palate ) in next


pregnancy is 4%

October 24, 2017 152


Q/ Cause of death in marfan is aortic root aneurysm

Q/ Longest cell cycle is interphase

Q/ Meige syndrome occurs in epithelial ovarian tumor

Q/ Ovarian cyst more than 5 cm is follicular ovarian cyst

Q/ Case of croup what you will find in auscultation

Q/ Case of epiglottitis ask about management ..my answer was admit


to icu and refer to ent specialist

Q/ Croup case treatmen .epinephrin and steroid

Q/ Side effect of betamethasone in children is growth retardation

Q/ Depression ..seratonin defeincy

Q/ Sympathatic neurotransmitter is norepinephrin

Q/ Diabetus affects wound healing by limiting phagocytosis

Q/ Diabetic ulcer dx deep tissue culture and something else

Q/ Pt develops gout due to medication ...thiazide

October 24, 2017 153


Q/ Pt with dx having htn develops protein in urine what to give ACEI

Q/ SAME Q BUT PT CAN' T take ACEI DUE TO SE so give ARB

Q/ cervical incompetence size in ultrasound is 30

Q/ Question about personality disorder ..i think the answer was


antisocial

Q/ Schizophrenia what against it duration of sx

Q/ Schizophrenia sx for 3 m what is called schizoniphorm disorder

Q/ Tx of obsessive compulsive disorder

Q/ Definition of idea of reference

Q/ Example of auditory hallucination

Q/ Feature of major depressive diorder ..the choices was


hypomania...decrease ey contact during conversation

Q/ Bmi calculation

Q/ Barkland formula

October 24, 2017 154


Q/ Maintenanece fluid calculation

Q/ Dehydration calculation

Q/ Colerectal screening recommendation..

Q/ Hba1c level which is contraindicated for pregnancy .choices were


8..9 ..10..11

Q/ Most common cause of 2ry amenorrhea is menopause

Q/ Duration of throat infection tx is 10 days

Q/ What will be elevated in mi


Tropinin I

Q/ Treatment of acute coronary sx

Q/ Pt with t inversion st change what could be the cause ...ischemia

Q/ Pt taking phosphodiestrase inhibitor ..drug contraindicated is


nitrate

Q/ Toxic substance ( i think ethylen or methanol) what is the renal


lesion??..ATN .GLOMERULONEPHRITIS ..PYLONEPHRITIS..ACUTE
interstital nephritis

Q/ Abscence seizure case??

October 24, 2017 155


Q/ Status epileptic case ??..diazepam

Q/ Asthma tx??

Q/ Prolong survival in sever copd?? ..oxygen supplementation (my


answer) or smoking cessation

Q/ Tx of cluster headache??

Q/ Tension Headache case??

Q/ Case of headache treated by analgesic tx and triptan if u stop the


tx .the headache return what is the dx?? ...chronic daily headache
..atypical migraine

Q/ Case of dm type 2 what to give??..metformin

Q/ Mechanism of action of metformin and incretin ..warfrain??

Q/ Cause of prolonged pt and aptt..factor x???

Q/ Hemophilia A treatment??

Q/ TTP treatmen??

Q/ ITP tx??

October 24, 2017 156


Q/ Case seems to be scleroderma??

Q/ Case of infantile congenital pyloric stenosis ??..abdominal US

Q/ SX AND SIGN OF LOW SBO?? IS absolute constipation

Q/ Type of hypersensitivty in AIHA IS?? TYPE 2

Q/ 4 case of preclampsia tx??

Q/ Contraindication of ECV??

Q/ contraindication of breastfeeding?? ..crackled nipple

Q/ Pt taking immunosuppressive drug ..which of the following


vaccine contraindicated .??..oral polio

Q/ Pt having T cell defiency ..can' take which vaccine ??...varricella (


my answer )

Q/ People can' t take BCG VACCINE DUE TO THEY HAVE PROBLEM


IN?? ifn gamma..

Q/ 4 q about milestone??

Q/ Tx of gonnorrhea??

Q/ Tx of group b streptococcus??

October 24, 2017 157


Q/ Scenario about infective endocarditis ..they found gram negative
coccobacilli?? ..organism hemophilus

Q/ Dermatological description of genital lesion due to sulpha drug??

Q/ Stage of labor??

Q/ Women having platelet defeincy ..type of anesthesia ??..GA

Q/ face presentation feature

Q/ Scenario ask about how to prevent postpartum?? ..active


management of 3rd stage

Q/ Scenario ask about what could be the cause that delay the
delivery?? ..suprabupic pressure ..fundal pressure..delivery of posterior
arm...maximum flexion of hip

Q/ Cause of infection in intravenous catheter?? .seeding from remote


site..insertion site...

Q/ How to ask about open ended Q and they give examples??...can u


tell me about the pain ( my answer)

Q/ Alzhiemer dx case??

Q/ Child come with DM sx what the other helpful test?? ..urine


dipstick

October 24, 2017 158


Q/ Case of intestinal obstruction Ask about imaging study?? ...the
choices were all barium study

Q/ The q that its answer is folded with c.terminal

Q/ Abdominal traum where amylase go anteriorly.??.lt paracolic gutter

Q/ Pheochromocytoma case??
Q/ Cushing case??

Q/ Juvenile polyp case??

Q/ Where to find meckles diverticulum??

Q/ Nasal polyp case??

Q/ 3rd CN PALSY??

Q/ uvetis case??

Q/ Reiter syndrome??

Q/ Rheumatoid case??

Q/ Pathophysiology of gout??

October 24, 2017 159


Q/ Osteoarthritis dx??

Q/ Septic arthritis dx??

Q/ Rubella case??

Q/ Antiviral taking by inhalation??...

Q/ Which of the follwing at risk of


PTC??...HASHIMOTO...REIDLE...ADENAMATOID NODULE

Q/ Which liver zones provide most oxygen supply??

A lot of research Q??

Q/ Body dysmorphic disorder case??

Q/ Ehler danlos syndrome inheritance.. ??

Q/ Tx of wilson if associted with liver cirrhosis??

Q/ X ray of adenocarcinoma??

Q/ Ecg seems to be hyperkalemia??

Q/ Case about human bite??

October 24, 2017 160


Q/ Best x ray view for rib fx??

Q/ Case of open fx??

Q/ Lingual nerve is a branch of trigeminal nerve

Q/ Best indicator for uti?


..urine leukocyte..urine nitrate ( my answer)

Q/ Early cervical cancer sx is post coital bleeding...

Q/ Rheumatoid arthritis anemia is NNA

Q/ case of nephritic syndrome??

Q/ Heavy metal cause infertality??

Q/ Treatment of mastoiditis ??

Q/ Noise induced senseneural hearing loss??

Q/ Pt have left hemiplasia with rt medial eye squint ..where is the


lesion.??.pons..medulla ..midbrain ..internal capsule

4/10/2017

October 24, 2017 161


Q/ Posterior vagal trunk affected organ?

Q/ correct definition of recurrent miscarriage?

Q/ amount of bleeding in menorrhagia?

Q/ diaper rash with satellite lesion?

Q/ student e hx of sore throat,headache, mild cough, low grade fever,


chest pain,tired,general malaise? Pneumoccoci,mycoplasma,aspiration
pneumonia,legionella

Q/ child 8years and half with breast bud, sparse pubic hair,mild
pigmentation of labia?
Normal development, precocious puberty

Q/ lower lip sq cell carcinoma which lymph node?

Q/ tender, hard swelling oozing after suturing wound in ankle?


cellulitis, abscess, furuncle, carbuncle

Q/ recall bias in which study?

Q/ UTI ttt in child? Augmentin for 10 days

Q/ pt e dm & htn on medication had tremor with movement ? No


cerebellar lesion on choices and there is essential tremor, psychological,
parkinsonian, physiological

October 24, 2017 162


Q/ male e hemophilia a and female carrier what % their child will have
hemophilia?

Q/ tall & thin boy e joint laxity also his mother has same features what
is mode of inheritance?

Q/ female (forgot her age) nullipara unable to conceive e plethoric


face and purpule striae at abdomen and bp 160/... ,hypokalemia, what
also she has? Acropachy and other unknown features(:

Q/ pregnant e bacterial vaginosis what is ttt?


No clindamycin only metronidazole in choices

Q/ female with hx of gdm gave birth of healthy baby six months ago ,
blood glucose 6.5 ,A1C 5.5 what to give? Insulin, metformin, glitazon 7

Q/ action of metformin on muscles?


Decrease glucose uptake, increase glocuneogenesis, increase release of
glucose, increase acid oxidation

Q/ male e lumbar pain extend to umbilical region no urinary


symptoms, normal urine analysis what is dx?
Epidydimorchitis, pyelonephritis, lumbar muscle spasm

Q/ boutinaire deformity description?

Q/ mastoditis ttt ?

October 24, 2017 163


Q/ study of obesity in children include bmi and gender what to add to
asses risk of obesity?
Hdl/ldl ratio, diatery habits, girth measurment, skin fold thickness

Q/ study of something show reduce the risk of dm from 10% to 5%


what is called?
Efficacy, effectiveness, i forgot the rest

Q/ pt collapsed at clinic, u call cardiac arrest team, pt shows no signs


of breathing what is next step? Start compression 30:2, give 2 rescue
breathing, asses carotid pulse

Q/ child e dka what initial ttt? Bolus of normal saline, sc insulin, iv insulin
infusion

Q/ pt e recurrent renal stone on htn medication to reduced calcium


excretion and cause gout? Hydrochlorithiazide, fursemide

Q/ pt e atrial fibrillation on warfarin 12 mg had hematuria, rectal


bleeding, bruises, INR 6.2 what next?
Reduce warfarin, give whole blood, give vit k, replace warfarin with aspirin

Q/ burkitt lymphoma gene?

Q/ athelet cant planterflex his foot injury of what?

Q/ main lateral rotator muscle of hip?

October 24, 2017 164


Q/ child suck his thumb his mother tell him to stop but he didnt what
u advise her to do?

Q/ pt with tender flank and mutation in chromosome 16 what mode of


inheritance?

Q/ child e barking cough what finding on auscultation?

Q/ pt can't extend wrist and finger , nerve injured?

Q/ female her husband had gonorrhea what best investigation for her
? Gram stain, pcr, ... I forgot others

Q/ child e conistpation since birth which investigation help establish


diagnosis?
Plain x ray, manometry, rectal biopsy, u/s

Q/ child 3 days old normal perinatal hx appear tired, with burned


sugar smell in urine?
Maple syrup sugar disease, phenylketonuria

Q/ paget's disease of bone?


Q/ 2 cases about pneumonia

Q/ apgar score and GCS

Q/ decrease vision & painful eye movement?


I choose optic neuritis

October 24, 2017 165


Q/ what cause color vision loss?
Also i choose optic neuritis

Q/ young healthy pt e red eye circumcirneal congestion, IOP 30,


dilated pupil, what best drug?
Latanoprost, something b blocker, acetazolamide... 7

Q/ old pt had packed RBC transfusion develop fever, chills, irritation


at transfusion site once it started, what to do?

Q/ Pseudomembranous colitis is caused by what? Answer: bacteria.

5/10/2017

Q/ best indicator to have Accurate study :


Specificity
Sensitivity
PPV
NPV

Q/ Mersa:
2:66

Q/ croup treatment :
Epinephrine +steroids

October 24, 2017 166


Q/ corneal abrasion ( pic ):
Topical antibiotics

Q/ most common mets cancer to stomach from:


Kidney
Lung
Heart
Ovary

Q/ pregnant with cancer , which cancer associated with pregnancy:


Breast
Ovary
..etc

Q/ can understand but can't talk all examination is normal :


Functional aphasia?

Q/ temporal arthritis can lead to :


Loos of vision

Q/ lesion in the eye ( petrigus ) what you will tell the patient:
Pre malignant lesion?

Q/ lactating with breast worm and tender treated with BS antibiotics


then develop mass what you will do for her :
FNA

October 24, 2017 167


Aspiration
Mammogram
Antibiotics

Q/ too many cardiac questions depending on the ( pic ) want either


diagnosis or management ( VT , heart block , STMI )

Q/ young had trauma to his right knee and develop non tender non
worm swelling what you will do :
Aspiration
X-ray for both knee
Antibiotics
Cast above the knee

Q/ hip dislocated ( internal rotation addiction ) what type of


dislocation is that :
posterior
Anterior
Lateral

Q/ what muscle responsible for laterally rotate of the hip joint :


Gluteus Maximus

Q/ Child with atopic dermatitis next to steroids : 
Methotrexate


Q/ 23 young female married trying to conceive since 3 months ,


every thing including her husband is normal , what to do for her:

October 24, 2017 168


Try some more

Q/ old man with blood in stool , diagnosis :


Colorectal cancer ?

Q/ pudendal nerve block , which of the following structures will be


not blocked by the anesthesias : 
Rectum

Q/ premenstrual sever pain , received NSAD and improved but incest


to have powerful analgesic, what you will do for her :
Give her hormone therapy or evaluate her psychological

Q/ preeclampsia diagnose by:


Hypertension

Q/ child with itchy scalp infect his classmates , cause is :


Pedicunals capitis

Q/ contraindications for cercumsize is :


Hyposperism

Q/ case of nephrotic syndrome asking for treatment:


Prednisone

Q/ 2 cases of post streptococcus granulmotis :


Key her is low C3

October 24, 2017 169


Q/ case about man can't sleep can't calm always worry about
everything:
Generalized anxiety disorder

Q/ kallman syndrome :
Read about it

Q/ months old child hypotension, dysmorfic feature..

Q/ pneumonia treatment :

Q/ young male went to Asia purulent charge from penis ,


unprotected sex:
Chlamedia or genorrea

Q/ if you have gennora case you should chick for what :


Chlamedia

Q/ depressed male unconscious came to ER found next to him


empty bottles of medication , what is it : no other details

Q/ which is the best indicator to help you diagnose ovarian cancer:


Histopathology
Tumor marker
US
Family history

October 24, 2017 170


Q/ young male sudden polyuria and thirst , same condition with two
of his brother , diagnosis is :
DM
DI
Water toxicity
Normal

Q/ smoking with high calcium what is the diagnosis:


HyperParathyroidism

Q/ 3 years old boy cough when playing , diagnosis :


Asthma

Q/ 2 questions about painless genital ulcer , unprotected sex :


Syphalis

Q/ breast cancer , which lymph node you will examine first ‫ في اختبار‬:
‫واحد جاني مرتين دا السؤال‬
Anterior

Q/ Surgeon take graft from rectus muscle which artery should be


dissected:
Inferior epigastric

Q/ unprotected sex , rash all over the body , diagnosis:


Syphalis

October 24, 2017 171


Rubella
..etc

Q/ research questions ( lung cancer ) :


,3

Q/ research questions ( red eye ) :


03

Q/ research questions ( OD ) :
3.39

Q/ RR is : a/a+b / c/c+d

Q/ see the definition of : efficacy , effectiveness

Q/ epileptic patient , about his vaccinations :


Devote DTP
All living vaccines
Only killed vaccines
Normal vaccination

Q/ his son got influenza after receiving influenza vaccine , what to till
him :
This is not risk in killed vaccines
This is normal

October 24, 2017 172


This might happened with this vaccine

Q/ four group of family eat from Resturant came with poisoning ,


sample show bacilli :
Bacillus something

Q/ Women post intercourse bleeding: Uterine cervix

Q/ Elderly with vasomotor rhinitis :


Antihistamines
Decongested
Corticosteroid

Q/ Neurofibromatosis gene:
2,q11

Q/ blue non tender mass lower lip :


Mucocele

Q/ best way of prevention : 
Genetic counselling


Q/ two first cousins want to get married which test they will do :
Thalassemia

Q/ two carries of thalassemia, chance to get a child with thalassemia


major is :
%10

October 24, 2017 173


Q/ thyroid storm initial therapy :
Propranolol

Q/ painful vesicles in the eyelid , what to do :


Antibiotics and referral to ophthalmologist

Q/ sexually active women with painful vesicles on genitalia :


HSV

Q/ best place to take Pap smear :


Transitional zone

Q/ healthy athletes with high lipid :


Mutation LDL receptors

Q/ tennis player :
Lateral epicondylitis

Baby thirsty , tachycardia , sunken eye , dehydration :


%9-4
%0<
%2


%9>

Q/ RTA patient , what to do :


Respiratory support

October 24, 2017 174


Q/ child , vomiting, bloody stool ,most important step : 

Iv fluid resuscitation

Q/ doctor diagnose patient with Schizophrenia after one month


history , what go against the diagnosis :
Duration

Q/ 15 years old girl with pubic hair , the rest is normal:


Premature adenarce

Q/ Incretin mechanism of action :


Increases the insulin secretion

Q/ Metformin in POS :
Decrease insulin resistance

Q/ Female her hasband known to have HBV , her LFT : high


billirubin mild elevated ALT Investigation :
IgM c AB


Q/ patient jaundice , drowsy , ascitic , tremor , dilated veins ...etc :


Liver failure

Q/ HTN patient came with gout , what is the medication :


Thiazide

October 24, 2017 175


Q/ DM patient will have surgery ( cholecystectomy) :
Shift to insulin

Q/ patient with right knee osteoarthritis, presented with swelling of


the right knee. On examination the right knee is swollen with no
change in temperature. What will you do :
Bilateral Knee X-Ray and Ibuprofen

Q/ Rheumatoid arthritis with the loss of bones in joint , What is the


cause: Substance released by synovial cell

Q/ Patient with 3 reading of high Blood pressure , All investigations


are normal except high Na , What’s the diagnosis : 

Primary hyperaldosteronism

Q/ DM , missed insulin , sweet breathing odor , what you expect to


find in urine :
Acetoactate

Q/ the most important structure control BP is :


Heart
Capillaries
Arterioles
Veins

Q/ in ER received Naloxone , what toxicity :


Morphine

October 24, 2017 176


Q/ patient came with high anion gap metabolic acidosis , cause :
Aspirin

Q/ cell energy :
GTP im cytoplasm

Q/ destination of protein :
ER

Q/ bond between RNA :


Hydrogen

Q/ prevent recurrence of UTI :

Decreased PH ,increased urea increased urine osmolarity


Q/ cervical infection can enter to superior mediastinum through:


Retro-pharyngeal space

Q/ end stage liver disease , anti-fungal:


Caspofungin

Q/ glomerulonephritis + hemoptysis :
Goodpasture syndrome .

Q/ epilepsy, which of the following receptors stimulated :


Glutamate

October 24, 2017 177


Q/ posterior vagus dissected , what structure will be affected
Esophagus

Q/ to diagnose a girl with HTN:


Above 95 percentile

Q/ Ecg shows RBBB , ECHO: right ventricle motion abnormality and


right ventricle hypertrophy : 
ASD

Q/ DM child unconscious in home :


IV glucagon

Q/ 1mm defect in muscular atrial septum What you will do :


Watchful waiting

Q/ baby start to drink juice :


Fructose

Q/ starry appearance :
Burkitt's lymphoma

Q/ cyanosis at end of cough : 
Croup

Q/ painful vision loss , pupil dilated :


Glaucoma

October 24, 2017 178


Q/ flashes of light :
Retinal detachment

Q/ too many questions describing the study and ask about the type :
most of it was cohort

Q/ can get from undercooked beef:


Taenia saginata

Q/ thought agains religion and culture :


Delusion

Q/ dizygotic twins :
dichorionic diamniotic twins

Q/ most common complication of MI in 6 hours :


Arrhythmia or ventricular septal rapture

Q/ Fanconi syndrome :
Autosomal recessive

5/10/2017

Q/ Signs of arterial hemorrhage:


bright red with steady flow
Bright red with profuse flow

October 24, 2017 179


Maroon red with steady flow...

Q/ MVA anterior injury of the pancreas and anterior leakage of


pancreatic fluid will collect:
Subphrenic
Subhepatic
Lf colic gutter
Omentum bollium...

Q/ Otitis media pt resistant to amoxicillin how to treat!?


Penicillin G
Pipericillin
Vancomycin
Azythromycin...

Q/ fetus with only one umbilical artery in his umbilical cord this
indicates what!?
Black race
%03mortality rate
Maternal diabetes...

Q/ her cycle is 35 days with 5 days of menstruation when is the day


of ovulation
24
20
...11

October 24, 2017 180


Q/ community medicine doctor have funds to prevent the most
disease cause mortality in KSA what is the disease!?
DM
Breast cancer
Constructive pulmonary disease
Coronary heart disease or IHD...

Q/ the gold standards of measuring the renal function


Insulin clearance
14h cereatenin clearance...

Q/ dull aching headache increase with straining and coughing in DM


patient
Viral meningitis
Tb meningitis
Brain abscess
Sinusit

Q/ Diabetic pt suddenly suffer from hand pain mainly in the ring and
middle finger increase with passive extension of the the fingers (pic:
swollen hand)
Ttt:
-predinsolon
-nsaid
-antibiotic...is

Q/ Which drug should be stoped before IVP!?


Thiazides

October 24, 2017 181


ACEI
CCB...

Q/ Prognosis of Breast cancer depends on:


A- number of axillary lymph node
B- estrogen receptors

Q/ Deficiency in which enzyme cause depression??


Glutamate
Serotonin...

Q/ Close fracture to humerus type of nerve damage in the radius.n!?


Neuroprexia
Neurotemesis
Axontemesis

Q/ Bacterial vaginosis chose correct answer:


No priorities no erythema
In premenopausal women
Contagious
Nitrate ph become yellow in ph paper‫شي زي كذا‬

Q/ Opthalmology :
Herps in clinical examation what you will see :
-less corneal sensation

October 24, 2017 182


Q/ 2 group asthma drug the other grouo is another drug..
‫ وطلعو من الستدي‬SE ‫نسبة منهم حصل لهم‬
‫ابش حتسوي؟ وخيارات‬

Q/ the cell responsible of erythema nodosum in IBD!?


T cells
B cells
TN cells
Neutrophils

Q/ percentage of prolonged labor!?


%0-2
%23-0
...%20-23

Q/ recommendation A in colonoscopy age!?


60-03
,3-03
.03-,3

Q/ can diagnose breast cancer by imaging before clinical by


2year
1years
0years

Q/ behcet disease with ulcers resistant to steroid how to treat!?

October 24, 2017 183


Alpha interpheron
Cyclo....

Q/ pt complain of vitiligo hoarseness mood swinging on thyroxin


Dx!?
Hypothyroidism
2ry hypoparathyroidism
1ry hypoparathyroidism
Hypopituitarism

6/10

Q/ Nitrofurantoin dose in UTI and for how long.

Q/ Q about lyme disease.

Q/ Q about dizygotic twins.

Q/ Typical case about acute appendicitis.


Q/ Case of paralysis of the upper limbs (and I guess lower limbs) ..
Dx: conversion disorder.

Q/ A case mentioning presence of reed sternburg


—> Dx: Hodgkin lymphoma.

Q/ Nephritic syndrome.

October 24, 2017 184


Q/ Goodpastures

Q/ Girl k/c of depression and found to be semiconscious with empty


bottle next to her, what is the drug?
Answer: most likely Fluxetine (SSRI).

Q/ MOA of metformin in PCOS


—> answer: reduces insulin resistance.

Q/ Extrapyrmidal SE has occurred (dystonia) after administering


some drug. What is the drug?
Answer: Metoclopramide.

Q/ A young woman came to Primary health care for pregnancy


counseling, she had chickenpox when she was a child. What will you
do to her?

⁃ Varicella immunoglobulin. (Most likely)

⁃ Rubella antigen .

⁃ Forget.

Q/ mentioning that there’s EBV. What’s the Dx?


Answer: infectious mononucleosis.

Q/ Memorize the equation of Specificity. Few questions came asking


about it.

Q/ MERS-CoV —> answer: 66:1.

October 24, 2017 185


Q/ Patient with pain in lips and left cheek, with tenderness, what to
give?
Carbamezapine
Amytriptaline
Propanolol

Q/ Patient in labor and got complicated, will go for c/s, when to give
abx?
Preoperative
Intrapartum
Intraoperative
Something else

Q/ Picture of CTG, whats the dx?

Q/ Most common site for crohns?

Q/ Patient with abdominal trauma, what is the first place pancreatic


enzyme will leak to?
Lesser omentum
Subhepatic
Subnephric

Q/ Patient with grade 1 splenic injury, what's the manegment?


Surgical Rx
Observe in ICU

October 24, 2017 186


Observe in surgical ward
Medical Rx

Q/ Most common site for meckel diverticulum

Q/ Child with high lactic acid?

Q/ Pyruvate dehydrogenase def

Q/ Patient with high BP, otherwise normal, when to check BP again?


6month
2year
0years

Q/ Dorsalis pedis artery, lateral to which structure?

Q/ Most common cause of IDA?

Q/ Patient with meningitis, culture showed gram positive bacilli,


whats the organism?
Listeria

Q/ Pt k/c of hypothyrodism , use levothyroxin 175 mg , swith to 200


mg . Labs : Tsh : high t4 : normal What is the cause ?
-ectopic thyroid
-Primary hypo
-Secondary hypo

October 24, 2017 187


-Small dose .

Q/ Pt known case of asthma , his wife has osteoporosis, he has renal


stone , he eats multivitamin, vit D and calcium .
PTH : high What is the dx ?

Q/ Patient has hemorrhage, baroreceptor activated which lead to


increase heart rate, what of those will be decrease?
Ventricular contractility
Venous capacity
Coronary blood flow

Q/ Pregnant women ...... fetal heart dropped from 140 to 80 what to


do ?
-GA

Q/ Case: newborn has tetany after delivery calcium: 1 ( normal 2-3)

Q/ One year later , he has recurrent upper respiratory infection with :


hip , hsv , pneumocystic jovirci - asprigellus
What is underlying his condition?
-q 22 depletion ( smth like that)
-Depletion of cd4 lymphocytes
-Wiskott dis
-NADPH oxidation

Q/ How hyperglycemia affect wound healing:

October 24, 2017 188


-dec imuune system
-Dec phagocytosis
-Stimulates bacteria growth

Q/ Patient with multiple blood transfusion and splenomegaly, what's


the cause?
ankyrin deficiency

Q/ Patient with sings of hyperaldosterone, where is it coming from?


Reticularis
Glamerulosa

Q/ Patient with MS (0.7 mm), with moderate MR, RVH and Afib, whats
the manegment?
Ballon
Valve replacement
valve commissurotomy
Medical rx

Q/ Patient with AS, whats the management?

Q/ Inheritance pattern of NF?

Q/ Case of nephrotic, whats the rx?


Prednisone

Q/ Prophylaxis for travelers diarreha?

October 24, 2017 189


Q/ Patient with bee sting, whats the optimal rx?
Epi, anti histamin and iv fluid
Remove sting and monitor

Q/ Patient with decrease color vision?


Optic atrophy
Iridocyclitis
Forget other options

Q/ Child with 30 degree esotropia, whats your manegment?

Q/ 21 y/o Female preconception counseling, hx of fetal death after


delivery with neural tube defect, whats your best advice?
It could occur because of your age
Cvs at week 16
Forget other options

Q/ Questions about CAGE questionnaire

Q/ Most common sign in lower intestinal obstruction?

Q/ Athlete, Heel pain in the morning, whats the dx?


Plantar fasciitis

Q/ 40 y/o Female with lump in breast, whats the best method for
evaluation?

October 24, 2017 190


Mamogram
US
FNA
Ductography

Q/ Patient with DUB, what to give?


OCP

Q/ Case of ovarian torsion

Q/ Elderly, asymptomatic, with high WBC.


%91lymphocyte, positive CD19, CD23 and CD5, whats the mx?
No treatment
Cyclo
Retuximab
Retuximab + something

41 september

Q/ schizophrenic complaint on medication came with metabolic


acidosis what is the drug?
-Olanzapine
-Clozapine
-haloperido

October 24, 2017 191


Q/ antipsycotic drug causes eye pigmentation?

Q/ 4 cases about panic disorder (ttt and dx)

Q/ q about depressed women ,the doctor asked her about psychosis


she left out the room what do you call her action?
Acting out
Interjection

Q/ definition of delusion?

Q/ adjustment disorder?

Q/ 4q about SE of antidepressant Tca and SSri?

Q/ what is the least fetal dose of Tca


033mg
2g
1g
0g

Q/ what is the most common cause of candida infection?


-candida albicans
there was other choices of candida I cant remember

Q/ dog bites are?


Poly microbial

October 24, 2017 192


Viral
Bacterial
Fungal

Q/ long scenario with osteoporosis and recurrent kidney stones and


increased PTH

Q/ long scenario…biopsy showed starry sky appearance…what is the


gene affected?

Q/ which lung cacer causes SIADH syndrome?

Q/ long scenario about lung cancer on pathology (polygonal cells)


and ?…

Q/ asthsmatic drug that causes candida infection

Q/ tretment of vonwilbrand disease?


FFP
Cryptoprecipitate
.…
Q/ recurrent hemarthrosis what is the dx

Q/ pencil cell in blood smear?


sickle cell anemia
thalassemia
?.…

October 24, 2017 193


Q/ 0yrs old with hand and foot pain?
Sca

Q/ patient with polycystic ovarion syndromeis at risk to develop


which cacer?

Q/ pts had surgery in the posterior triangle now lost sensation the
auricle and lower jaw what nerve is affected?

Q/ after cardiac surgery vagus nerve was injured which organ


affected?

Q/ pregnant with genital warts what to do?


Do pap smear every 3 months
if still there 2 wks b4 delivery do CS

Q/ Retnoblastoma causes?
Strabismus
Squent
???

Q/ how asprin causes hyperthermia?

Q/ q about Wernicke aphasia

Q/ placenta previa and adherent what is the noninvasive test to do?


MRI
Transvaginal US

October 24, 2017 194


Doppler ulsasound
Ct

Q/ q about treatment of iron toxicity

Q/ pts with recurrentangioedema what gene is affected


c1 esterase
..…I don’t remember all choices

Q/ thumb sighn on xray what is the organism ?


hemophelas inf type b

Q/ how does protein enter peroxomes?

Q/ Bond between trna and mrna?

Q/ Treatment of cutaneous lishmania?

Q/ 04 yrs old wake up with ptosis and diplopia at morning then


symptoms resolves?
Botilism
Myasthenia gravis

Q/ what causes cyanotic heart disease?


TOF
PDA
VSD

October 24, 2017 195


Q/ senario of septic arthritis aspiration was done
cloudy…wbc>50,000
what is the manegment?
Surgical debridement and antibiotic
Iv antibiotic for 14 days
Oral antibiotic

Q/ treatment of acute migraine?

Q/ site of meckel diverticulum:

Q/ blunt traumain the anterior abdomen with pancreatic injury and


enzymes leak where the enzyme will go:

Q/ sign for low intestinal obstruction:


altered bowel movement
absolute constipation
diarrhea
flatulence

Q/ significant risk factor for heart disease:


age
smoking
Obesity

Q/ prophylaxis of traverlers diarrhea?

October 24, 2017 196


Q/ pts k/c of hypothyroidism ,use levothyroxine 175 mg switched to
200 mg Labs tsh:high t4 normal
what is the cause?
-ectopic thyroid
-primary hypothyroidism
secondary hypothriodism
small dose

-too many q about:


ligament injury
osteoarthritis
osteoporosis
thyroid
ectopic pregnancy
post partum hemmorage
ovarian tumors

Q/ tt of acute rheumatic fever

Q/ urti in children

Q/ missed vaccination …study the symptom of the disease(


diphtheria,pertussis,mumps,measles.…

Q/ MI treatment

October 24, 2017 197


Q/ Metabolic syndromes

Q/ Diabetic drugs

Q/ Chlorpromazine—Corneal deposits ;

Q/ Thioridazine—reTinal deposits

8/10/2017
Q/ question about Hodgkin's lymphoma and mentioned that there is
reed Sternberg cell and band of collagen or fibrosis. Which one of the
following is the diagnosis?
A-nodular sclerosis
B-mixed cellularity
C-lymphocyte predominance
D- Lymphocyte depletion

Q/ CASE about hematology all description and investigations were


consistent with hereditary spherocytosis, positive osmotic fragility
test and jaundice with splenomegaly:
Which one of the following is correct regarding this scenario?
A-G6PD
B- SPECTRIN –ANKIRIN DEFICINCY
NOTE: Hereditary spherocytosis is caused by a variety of molecular
defects in the genes that code for the red blood cell proteins spectrin
(alpha and beta), Ankyrin, band 3 protein, protein 4.2, and other red blood
cell membrane proteins :

October 24, 2017 198


Q/ CASE scenario about diabetic patient diagnosed recently since 6
months. Which one of the following renal investigation we should do
yearly:
A-serum creatinine
B-microalbuminuria
C-creatinine albumin ratio
D-24h urine protein collection

Q/ which one of the following infection diagnosed by stool analysis


by finding an antigen?
A-Ascaris
B-tenia saginata
C-shistosoma mansoni
D.………………………………-
NOTE: all of the above infections are diagnosed by stool analysis by
finding eggs or trophozoits .
The infections which are diagnosed by finding antigens are H. Pylori and
giardiasis
SO, MAY BE THE MISSING ANSWER HERE IS THE CORRECT
CHOICE

Q/ 40 years old man complaining of vertigo and tinnitus associated


with nausea and vomiting, the attack of vertigo last 30 to 45 minutes
(I am not sure about headache)
Auditory test showed: low frequency sensorineural hearing loss.
What is the most likely diagnosis ?
A-Meniere's disease
B-acoustic neuroma
C-vestibular disease

October 24, 2017 199


D-PAROXYSMAL benign vertigo disorders
Note: most likely the answer here is( A ) because acoustic neuroma
usually is progressive and come with facial weakness + high frequency
sensorineural hearing loss and headache.
Note there is previous q in collection but was with facial weakness and the
q is:
Scenario about hearing loss of gradual onset in one ear and vertigo picture
of bell's palsy diagnosis is acoustic neuroma

Q/ treatment of shigellosis:
A-amoxicillin
B-ceftriaxone

Q/ -CASE about abruption placenta which one of the following sign


and symptoms is the most common or the most serious one (I am not
sure)
A-vaginal bleeding
B-uterine contraction
C- Fetal distress
Note: the choice is not very accurate so, make sure in the exam.

Q/ elderly women with vulvovaginal lichen planus lesion in posterior


vaginal fornix, this patient is at risk to develop which type of the
following vaginal cancer:
A-Squamous cell carcinoma
B-adenocarcinoma
C-squamoadenocarcinoma
D..……………………………………-

October 24, 2017 200


Q/ most common nerve injured in humerus fracture:
A- radial
B-axillary
C-ulnar
D-median

Q/ SCENARIO of ptosis (no other information available) there is


elevator muscle dysfunction, what is the management?
-2mullurectomy
-1levtor resection
-0frontalis suspension
-4levator muscle aponeurosis
Note: To answer this question we have to know three variables which are:
-2Age of the patient (congenital, pediatric or older)
-1unilateral or bilateral
-0severity of the symptoms (mild, severe) and here is the summary:
If it congenital mild unilateral, no severe ptosis or symptoms first option is
Levator resection BUT if the patient is old or the ptosis is bilateral or
severe ptosis go and do frontalis suspension
Please go and read more about it

Q/ Which one of the following GIT cell secret defensins:


-2paratial cell
-1chief cell
-0paneth cell
-4plasma cell

October 24, 2017 201


Q/ case of cardiac patient issues start to develop sign and symptoms
of heart failure which one of the following will prolong patient life?
A- ACE
B-digoxin
C-diltiazem
D-diuretic
Note: This is my answer, no idea about the correct answer.

Q/ case of otitis media, what is the management:


A-Amoxicillin
B- Amoxicillin + clavulanic acid
Note: This is my answer also, no idea about the correct answer.

Q/ PATEINT eat mushroom then developed allergy symptoms


Which one of the following enzymes inhibited:
A-RNA polymerase 1
B- RNA polymerase 2
C-RNA polymerase 3
D-DNA primase
Note: based on booklet 13

Q/ INFECIOUS case of GIT characterized by diarrhea followed by


constipation and give you bacteriology finding which is gram
negative and other features I cannot remember, then asking about
treatment:
A- Ciprofloxacin
B-chloramphenicol
C-penicillin

October 24, 2017 202


D..………………………-
Note: from my reading salmonella is characterized by diarrhea followed by
constipation
So my answer was (A) but not sure about it.

Q/ Obturator nerve injury which muscle is completely paralyzed ?


A-adductor Magnus
B- Adductor longus
Note: part of adductor Magnus innervated by sciatic nerve.

Q/ How to screen for thalassemia in pregnant women:


A- US on 12 weeks
B- Chronic villus sampling at 16 weeks
C- Triple test at 16 weeks
D-quadruple test at 15 weeks
Note: This is my answer not sure about it.

Q/ principle treatment of urge incontinence?


A-medical
B-surgical
C-medical and surgical
D- Bladder training and something else

Q/ type of wrist joint?


A-PIVOT
B-HINGE

October 24, 2017 203


Q43: retrosternal pain and barium swallow showed esophageal
croewkscrow appearance?
A-achalasia
B- Esophageal cancer
C-diffuse esophageal spasm

Q/ BRUNNER's gland location?


Answer: submucosal duodenum ( go and search about it )

Q/ in mackle's diverticulum, the surgeon will look to which part?


A-Terminal duodenum
B-terminal jejunum
C-terminal ilium
D-cecum

Q/ Why ectopic pregnancy occur:


Answer: disappearance pf zona pellucida
Note:
-2Concentrate on the new collection of questions
-1Different questions about incontinence in the exam (so read about them
very well)

Q/ anti diabetic medication working through activation enzyme AMP


protein kinase?
A-glipizide (sulfonylurea) (working through inhibition ATP DEPENDENT K
CHANEL)
B-tolbutamide

October 24, 2017 204


C-pioglitazone (THIS WORKING BY stimulation nuclear PPAR
RECEPTOR)
D-biguanide or metformin.

Q/ describe Boutonniere deformity:


Answer: DIP flexion and DIP extension

Q/ Inferior MI which one of the following arteries blocked?


ANSWER: RCA

Q/ which one of the following bias is associated with meta-analysis


study?
Answer: my answer was publication bias.

Q/ best test to diagnose hepatitis B?


A- US
B-blood test
C-blood culture
D-liver biopsy

️SMLE 9th of October


Q/ Metformin action on muscle

Q/ MOA of metformin

Q/ Drug strongerr 8-10 than morphin

October 24, 2017 205


Q/ term used to call ppl who eat ice
-amylophagia (starch)
-Geophagia(dirt/soil)
-Pagophagia(ice)

Q/ on human milk fasting duration

Q/ 2 y/o every thing is normal Exept she has pubic hair growth , GH
is normal , height & weight normal for the age , P/E normal clit normal
size for the age no breast enlargement or other 2ndary sex
characters, , what to tell the mother / dx ?
-normal
-Turner
-Early puberty

Q/ Teenage Pt. Got a bad grade in an exam and feels guilty, he try to
explain to everyone he meets/ talk with that the exam was poorly
written& its not his fault that he got a bad grade
-intellectualization
- -sublimation

Q/ Feeling hopeless is another form for expressing( or indicates or


assess ) which :
-suicidal ideation
-Depression
-Anxiety

Q/ Side affects of atropine :


-Seizure

October 24, 2017 206


-Vomiting
-Dry mouth

Q/ Paracetamol toxicity what to give .


Acetylcysteine

Q/When to give paracetamol antidote ( maximum or something)


1 -h
4 -h
6 -h
0 -h
Q/ read about paracetamol toxicity treatment

Q/ one year old baby was breast feed till 6 month was developing
normally, then the mother started to give him fruit juices and he
started to delay in development. Which of this substance he should
stop eating
-galactose
-Fructose
-lactose

Q/ Humaral neck/head fracture, what nerve would be affected:


-axillary
-Radial
-Ulnar
-Medial

Q/ Lachman test assess which of the following structures ;

October 24, 2017 207


-anterior cruciate ligament

Q/ loss of sensation of posterior & anterior aspect of the hand ; which


never is affected ;
-radial
-Medial
-Ulnar
-Axillary

Q/ Loss sensation of the little & ring finger which nerve is affected ;
-radial
-Medial
-Ulnar
-Axillary

Q/ 21 yrs old pregnant, history of dead upon delivery baby, is thinking


of getting pregnant; & worries that the same will happen again ,what
will u tell her ;
-Sampling of amionitc fluids Q/ ( X of month)
-Has the same risk as the rest.
-Higher risk because of her age
-US Q/ X of months

Q/ Most serious cause of stroke


-HTN
-Arrhythmias
-Smoking

October 24, 2017 208


Q/ In pudendal nerve block which of the following will not be
affected?
-Rectum
-Vulva
-Perineal body
-Urogenital diaphragm

Q/ Which on of these is a characteristic of tetralogy of fallot ;


-aortic stenosis
-Left ventricle hyperatrophy
-Pulmonary stenosis

Q/ most common fetus malpresentation ;


-breech
-Brow
-Face

Q/ pt came to the clinic & while waiting he collapsed on the floor on


his back ; there is no sign of breathing; what the first thing you
should do ;
-give 2 rescue breath
-Check carotid pulse
-Start chest compressions at 30/2 rate

Q/ pt complains of neck pain and tingling that radiates to his left


shoulder & arm & with loss of sensation of the arm following the pain
; most likely dx ;
-cervical disk prolapse?

October 24, 2017 209


-B
-C
-D

Q/ 40 y female depressed; what drug would you prescribe ?

Q/ active seizure pt what to give ;


-phenotin
-Diazepam
-C
-D

Q/ Most common cause of color blindness :


-cataract
-Glaucoma
-Diabetic retinopathy
-D

Q/ Girl with unilateral knee pain & high fever , no hx of trauma ;


aspiration results showed high wbcs ; what to do ;
-surgical consultation & iv Antibiotic
-Oral antibiotics for 5 days
-Iv antibiotics for 2 weeks
-Antipyretic & wait for lab results

Q/ most reliable marker for diagnosing hypothyroidism ;


-Serum T4

October 24, 2017 210


-Total T4
-TSH
-T3

Q/ what type of anemia is associated whith RA ;


-normocytic normochromic

Q/ adolescence with migraines; what is the long term complications ;


-Hearing loss
-Reflux esophagitis
-Depression

many questions about breast masses mostly asking about the (


)diagnosis
)a lot of research questions ( ) the test was 50-60 % obs&gyn (

9/10
Q/ Thioridazone side effect

Q/ Bubropion side effect

Q/ Headache during intercourse

Q/ Salmonella posioning Diagnosis

Q/ Mushroom mechanism of poisoning : enzyme

October 24, 2017 211


Q/ Premenstral syndrome ttt

Q/ Tuberclous meningitis diagnosis

Q/ Wilson disease diagnosis

Q/ Tibial nerve injury

Q/ Ulnar nerve injury

Q/ Obtrutor nerve injury

Q/ Trochanteric bursitis

Q/ Nitrofurantoin dose for adult

Q/ DM neuropathy ttt: pregabalin

Q/ Acute polio : Dx

Q/ Minimal change NS: age

Q/ Premylocytic leukemia:: lab values

Q/ Commenest presentation of wilm in adult ?

October 24, 2017 212


Q/ Anemia of chronic diseaee dx

Q/ Sideroblastic anemia dx

Q/ Meinere disease dx

Q/ Every women should be screened by DEXA at 65 . ( dont forget )


Q/ What criteria should be considered when establish new screening
test ?
continious screen invasive test short asymptomatic stage?.

Q/ Hazard ratio ??

Q/ PT with macrocytic anemia , received ttt .. what will decrease ?


reticulocyte MCV ??

Q/ Non selecting bias came with retro case CTRL study ( dont forget
)

Q/ Asbestosis x ray finding

Q/ Congenital glucoma : excessive tearing

Q/ Blepharitis ttt

Q/ Bleeding during delivery

Q/ Warfarin related factor

October 24, 2017 213


Q/ Pt on warfarin present with melena : what you will give ?

Q/ HuS ttt ?

Q/ Ankylosing spondiolitis case ?

Q/ Sjorgen sx ttt ?

Q/ RA diagnostic test ?

Q/ Spinal stenosis dx ?

Q/ Drug used for gestational HTN ?

Q/ Drug lower mortality in MI pt ?

Q/ Most accurate to diagnose severity of asthma ?

Q/ Test for hyperaldosteronism?

10/10
Q/ Nephrotic syndrome treatment

Q/ urethra detachment (urthroscopy , cytscopy , cytometric study )

October 24, 2017 214


Q/ bacterial fragils ttt

Q/ p value ,13 whats the meaning?!

Q/ most helpful in precicing in research ) ‫( مافهمته واول مره اشوفه‬

Q/ poor prognosis ( high igg ,low iga , high creatinine)

Q/ osgood ,came with knee swelling ) ‫( تكرر السؤال‬

Q/ somtaization

Q/ GAD ttt

Q/ premenstral syndrome

Q/ 01 girl with irregular period came with left lower q pain ( invest ( b
hcg , abdominal us)

Q/ 22 years old t scan 2.1 with compression fracture diagno(


osteopenia ,osteoporosis)

Q/ morphine vs nalxone

Q/ sulfa drug described the dermatology lesion

Q/ specific test for syphils

October 24, 2017 215


Q/ patient on sulfa and under went to surgery u will change the sulfu
by ( insulin ,metformine, ....)

Q/ definition of distchiasis

Q/ rosoca with belphritis ttt

Q/ ADHA

Q/ injury to posterior to sternomastoid now can not elevate the arm


above head

Q/ general sensory loss ( trigemenal nerve)

Q/ senario of wt loss , joint pain ithink whipples disease

Q/ cause of pseumembrans colitis( bacteria , ischemia )

Q/ can not stand on finger ( where is the injury quadriceps,planter


,calcenial ligament )

Q/ RTA and vomiting on the face first step is ( oral intubation,c spine
my answer)

Q/ seneriao i think gilbert disease ask about gene mutation

October 24, 2017 216


Q/ non cessation granuloma in lung biobsy ( diagnosis , adenocarcima
, sarcoidosis i think sarcodiosis )

Q/ lead poisoning anemia !!

Q/ sourse of energey move molcule in and out ( atp or gtp in


cytoplasm)

Q/ blood vessels in which layer of scalp)

Q/ cause of high output heart failure ( sever anemia , hypertension )

Q/ his brother dead when he is playing type of card myopathy(


hypertrophic,restricted )

Q/ to reduce pluaq (hand washing , insecticide)

Q/ to evaluate preventive interventions whats the study ( case , cohort


, R clinical trial )

Q/ ‫ و‬.. ‫ منهم تعافوا تلقائيا‬٨١ ‫ وبعدين‬.. ‫ المهم جالهم تسمم كلهم‬،، ‫ واحد راحوا تغدوا في مطعم‬٨٧
.. ‫ تعبوا مررة و تنوموا‬٧
‫ وش هو االورقانيزم ؟‬: ‫السوال‬
‫ ساعات‬٤ ‫خالل‬
‫الخيارات‬
Staph
Salmonella
‫اناا اخترت ستاف النها خالل ساعات‬

October 24, 2017 217


Q/ best for nocurnal enurasis)️ ‫( تصحيه قي الليل او بامبرز‬

Q/ ‫ولد يصحى في الليل ويشكتي من الم في رجله وخالل اليوم طبيعي‬


)perthes ,restless legsyndrome

Q/ sypmtom of pancreatitis and high GGT ask about the cause (stone
,.....)

Q/ left hemiplasia with right medial squint where is the lesion ( pons
,medualla , midbrain)

Q/ TTP clear

Q/ pregnant with loss fetal movment and shortness of breath high pt


and aptt (DIC , amoniotic embolism )

Q/ painless testicular swollen normal epdidimis nor urinary sym or


fever ( NOn steroid, us and referral to surgery, biopsy)

Q/ vaccination 9 months

Q/ 01 vaccination but hep A not avalible when to give (today , after


weeks , after3 weeor 7 weeks )

Q/ radial and obturatir nerve injury

October 24, 2017 218


Q/ hypertension urgency and emergency

Q/ syphilys case

Q/ c, diffcle

Q/ case of salmonla ask about mechanizm of drug

Q/ normal no history of cardiac when repeat lipid profile (1,2,5 years )

Q/ ccycstic fibrosis ask about chance to get the dis amng daughter

Q/ sickle cell anemia case

Q/ 3months married no pregnant ( try more , semen analysis )

‫ سووا لها‬، ‫حامل‬


fasting0 ‫ طلع مرتفع وسووو‬hour 50 ٧ ‫ طلع طبيعي وسوو‬hour oggt ‫مرتفع وش تسوي‬
‫تعالجها ب انسولين‬
‫تعالجها اورال‬
‫تسوي فحص ثاني للكربوهيدرات‬

Q/ First line in migraine mang (aspirin ,sumitrptan)

Q/ Prophylaxis (amitryptaline)

Q/ Uric acid stone whats the prophylaxis (allpiranole ,....)

October 24, 2017 219


---
Q/ Female bloody discharge from nipple ?
fibroadenoma
ductal papillary
breast abscess

Q/ Patient depressed found unconscious with empty pills around ,


with pupil dilatation what could it be ?
sertaline‫> االجابه‬
fluxitine
amitriptylline

Q/ What we give with analgesia to relieve its SE :


cimetiden
metocloprmide

Q/ Patient come to ER with symptoms for 1 month ) ‫( كاتبين اعراض كتير‬


and doctor diagnosis is schizophrenia and that's not good at all
because of :
A.time <than 6 months
B.symptoms

Q/ Protein enter the peroxisomes? Folded with help of c terminal

Q/ What dizygotic twins means?

Q/ Dideoxynucleotide used for ? DNA sequence

October 24, 2017 220


Q/ Neurofibromatosis gene ? Ch 17q11.2

Q/ Bond between tRNA and mRNA ? Hydrogen bond

Q/ Which one made in nucleolus ? mRNA

Q/ One question about acid base balance

Q/ Meckls diverticulitis most common in ?


Ilium

Q/ Winging scapula ? Long thoracic n injury

Q/ Blunt tauma in antirior abdomen with pancreatic injury and


enzymes leak where the enzyme will go:
A) omentum bursa. B) sub hepatic space C) sub phrenic space

Q/ sign for low intestinal obstruction:


A) altered bowel movement. B) absolute constipation C) flatulence D)
diarrhea.

Q/ site for meckel's diverticulum :


A) ilium B) jejunum. C) duodenum.

Q/ pudendal block which will not be affected:


A) Rectum. B) Vulva C) perineal body. D) urogenital diaphragm.

October 24, 2017 221


Q/ tumor arising from the kidney displacing the collecting system :
A) Wilms tumor. B) Neuroblastoma.

Q/ pt complain of proximal muscle weakness and ptosis there is


improvement after administration of anticholinesterase what is the
cause:
A) autoimmune B) viral induced C) drug induced. D) nutritional.

Q/ A patient with tuberculosis on medication for 3 months. He


developed pins and nee dles sensation of his lower limbs. Deficiency
of which of the following caused his symp toms?
A. Niacin
B. Folic acid
C. Iron
D. Pyridoxine(B6)

Q/ 16 yr from Africa (Ginia) with painless neck mass for 5 weeks


developed cough, fever , URS, histopathology : starry sky
appearance.
A. Burkitt lymphoma
B. infectious mononucleosis
C. Hodgkin’s lymphoma
D. Lyme disease

Q/ Patient with end stage liver disease, on central venous line, septic,
blood showed ood c/s budding yeast, what antifungal is appropriate
at this stage?

October 24, 2017 222


A. Caspofungin
B. Flucytosine
C. fluconazole
D. Itraconazole

Q/ Best treatment for travelers diarrhea:


ACiprofloxacin

Q/ safe vaccination you can give to immunocompromised patient :


AHBV

Q/ old patient came to ER 4 w of fever cough night sweat... (clear T.B


symptom) immedi ately do??!
A. Isolate patient in negative pressure room
B. give anti T.B drug
C. sputum culture D. chest x ray

Q/ Pt on cloxacillin for staph micro reported it is resistant to one of


the cephalosporins what to do:
A. Continue cloxacillin
B. Start vancomycin
C. Stop antibiotics

Q/ 75. long Scenario about old male bedridden on foley's catheter he


develop Gram ve bacteria with green blue discoloration on agar what
is the organism:
A. E.coli
B. pseudomonas aeruginosa

October 24, 2017 223


C. strep. Pneumonia

Q/ How does hyperglycemia cause infection to occur?


A. allow bacteria to grow.
B. decrease immune response.
C. impairs phagocytosis

Q/ Cervical infection can enter to superiormediastinum through:


A. Retropharyngeal space
B.Parapharyngeal space
C.Carotid sheath

Q/ 40 years old man presents with persistent lesion on the forearm


that started 3 months ago.He came from a deserted area that has
sand fly infestation. Giemsa stain showed Donovan bodies inside and
outside monocytes. What will you use to treat him? ( leishmaniasis)
A. Miltefosine

Q/ case of follicular tonsillitis with pic. Asked about ttt:


A.amoxicillin+ clavulanic acid

Q/ meningitis case: fever, headache, nuchal rigidity, and rash (pic)


what is the late complication?
A.Deafness.

Q/ bacteria sexual like behavior:


A.Conjuction

October 24, 2017 224


Q/ Child with abdominal pain , watery diarrhea since 6 months after
summer camping:
A.Chronic Gardia infection

Q/ patient with asthma , allergic rhinitis , came with red and scaly
plaques over the extensors and fingers:
A. Atopic dermatitis

Q/ Best initial screening test for pt suspected with coarctation of


aorta?
aecho/Doppler (initial test)
bCT cardiac
cMRI cardiac
dcardiacangio

Q/ Patient on warfarin 7 mg presented with melena, INR was very


high. What will you do?
a. Stop warfarin, Give vit. K
b. Lower the dose of warfarin

Q/ (long scenario) case of endocarditis with negative bacterial


culture, there is diastolic murmur radiate to the left axilla. What is the
most likely diagnosis?
a.SLE
b.Rheumatoid arthritis.
c.Acute myocarditis.

Q/ Patient with chest pain and ST changes, you will find elevation in:

October 24, 2017 225


A.ALT
B.AST
C.Troponin

Q/ patient with mitral valve disease and murmur radiating to the


axilla, right ventricle is enlarged. How will you manage?
A.Mitral valve replacement
B.Medical treatment
C.PCI

Q/ Pt HTN came with uric acid 200 you prescribe antihypertensive


drug for him after 1 week uric acid 400 and gout what is the drug?
A.Thiazide diuretics

Q/ Statin lower the lipid through inhibition of:


A.HMGCoA reductase

Q/ Reversible cause of stroke:


A. HTN

Q/ Case of Turner syndrome and asked about associates problems:


A.Cardiac malformation ( coarctation of aorta)

Q/ Patient with uncontrolled diabetes and asked about basal insulin :


A.NPH ‫ البقية مفعولها سريع‬، ‫ هوا اقرب شي‬rapid/fast acting
B.Aspart
C.Lispro

October 24, 2017 226


D. Regular

Q/ Pharmacodynamics of metformin, stimulation or inhibition of


which enzyme? Metformin work in which enzymes? Case about a
diabetic who takes metformin which resulted in correction of ha1c
question metformin lowers glucose by ?
A. decrease Muscle uptake of glucose
B. increase Muscle gluconeogenesis
C. Enhance muscle use of fatty acid oxidation
.‫اخترت سي بس مادري ايش الصح‬

Q/ A scenario of a patient with HTN came with headache and anxiety


and have 3 previous vistas of High blood pressure what is the most
likely diagnosis?
A. Benign medulla adenoma
B. Androgen releasing tumor
C. Cortisol releasing tumor

Q/ Thyroid disease associated with papillary ca:


A. Hashimoto
B. Riedel thyroiditis

Q/
61yfemaleknowncaseofosteoarthritis,cameforregularcheckup,nottaki
ngca supplements nor high ca diet, She is a high risk of osteoporosis
What is the best initial thing before deciding the appropriate mx?
a) DEXA SCAN ( dual energy Xray absorptiometry)‫جا االسم كامل مو االختصار‬
b)Oral ca, vit D, bisphosphonate
c) TSH,Ca,vit D

October 24, 2017 227


Q/ Patient with aspirin toxicity ?
Urine alklinization

Q/ patient with lung cancer and hyponatremia:


A.SIADH

Q/ Case of pneumonia with xray and asked about examination on the


affected site:
A.Bronchial breath sound

Q/ Case of osteoporosis and asked about appropriate treatment:


A.Bisphosphonate

Q/ Young boy with pain in his knee, aspiration of fluid reveals


yellowish and turbid appearance, Diagnosis?
A.Septic arthritis

Q/ Lytic femur lesion & osteoporosis skull? Paget disease

Q/ Patient with recurrent oral and genital ulcers and arthritis when the
pt took some type of IM vaccine, he developed sterile abscess at the
site of injection. What is the most likely dx?
A.Behçet Disease

Q/ patient with Hip septic arthritis ? Treatment?


A.Aspiration and antibiotics

October 24, 2017 228


Q/ Old man with joint pain worsens on movement. Xray of wrist
shows narrow joints in small joints. What's the dx?
A. Osteoarthritis
B. Rheumatoid arthritis

Q/ Elderly female complaining of depression, bilateral shoulder and


hip pain. Normal blood workup.
A. polymyalgia rheumatica
B. fibromyalgia

Q/ Which of the following is the most specific for Rheumatoid


arthritis ?
A. HLADR4.
B.Rheumatoid factor .
C.CRP.
D.Anticyclic citrullinated peptide (Anticcp)

Q/ patient with elevated direct bilirubin?


A. Choledual cyst

Q/ Characteristics of perforated duodenal ulcer?


A.Epigastric pain

Q/ Antacid causes constipation :


A.Aluminum hydroxide

Q/ Aspirin toxicity lead to:


A.Respiratory alkalosis followed by metabolic acidosis.

October 24, 2017 229


Q/ Rota confirmatory test:
A.Stool antigen.

Q/ Epileptic patient with gum hypertrophy and excess hair growth:


A.Phenytoin

Q/ Neurofimbromatosis inheritance:
A. Chromosome 17 q 22.11

Q/ which Alzheimers drug is hepatotoxic :

A.Tacrine😅‫حليته غلط بس جبلتكم الصح‬

Q/ case of carpel tunnel syndrome and asked about nerve


compressed:
A.Median Nerve

Q/ Head trauma and face lacerations , on ex. Eye shifted inward ,


which is the injured muscle:
A.Lateral rectus muscle

Q/ Anemia associated with RA?


A.Normocytic normochromic anemia

Q/ Which of the following can be found on smear in sickle cell


disease?
A. Bite cells
B. HowellJolly bodies

October 24, 2017 230


C. .Acanthocyte
D. Spherocyte

Q/ case pf acute chest syndrome

Q/ case of glumerulonephritis

Q/ patient with anemia and spoon shaped nail:


A.Iron deficiency anemia

Q/ case of sidroblastic anemia with high ferritin

Q/ case of sickler patient with paraphimosis

SMLE OCTOBER 10th


Q/ which one of the following is found in the nucleus?
mRNA

Q/ what is the bond between mRNA and tRNA?


Hydrogen bond

Q/ what is the longest phase in the cellular division?


A-Anaphase(I choose it), B-prophase

Q/ source of energy that transfer molecules in and out nucleus?


GTP in cytoplasm

October 24, 2017 231


Q/ A type 2 DM pt has an appointment in the clinic at 11 o'clock, the
doctor came at 12:30 because there was a serious case in the
hospital,the pt was angry, what will you do as a doctor ?
Apologize and expect that.

Q/ best one considered as open ended question in pt with chest


pain?
A-Tell me about the pain( I choose it)
B-when did the pain start
C-where is the pain.

Q/ definition of epidemiology?
Distribution and determinant of the disease

Q/ which one of the following neonatal problem requires an


immediate action ?
COA (I think)

Q/ definition of distichiasis?
Abnormal growth of eyelash from the orifice of mmeibomian gland.

Q/ cancer associated with polycystic ovarian syndrome ?


Endometrial Carcinoma

Q/ best screening test for colorectal ca?


Fecal occult blood testing ( no sigmoidoscopy in choices )

October 24, 2017 232


Q/ when you'll do Alpha fetoprotein ?
15 weeks

Q/ pt came to ER he was semiconscious you give naltexone he


became conscious what is the drug toxicity?
Opioids

Q/ pt with drug overdose O/E there was pinpoint pupils what is the
cause?
Morphine

Q/ antidote of digoxin?
FAB immune globulin.

Q/ pt with paracetamol overdose you after doing ABC you measure


the drug level you found it toxic what is the proper management ?
Give N-acetylcystiene

Q/ pt with cherry red skin what is he toxic of?


Carbon monoxide
Q/ female with S&S of pregnancy +right lower quadrant pain after
doing US there is ectopic pregnancy and the pt stable what will you
do?
Treat medically

Q/ S&S of pregnancy B-HCG negative,dx?


Ovarian torsion( not sure)

October 24, 2017 233


Q/ pt with periumbilical pain with anorexia and nausea ,labs Increase
WBC,dx?
Acute appendicitis

Q/ on OR the doctor cut vagus nerve by mistake while doing NISSIEN


fundolplication operation for ttt of chronic GERD,what organ will be
affected from cutting the nerve?
A-esophagus,B-fundus,C-urinary bladder( I choose i),D-colon.

Q/ neonate with umbilical swelling and yellow discoloration over it ,


the pt is vitally stable, dx?
There was a picture
A- urchal cyst,B-umbilical grauloma,C-omphalomesentric cyst

Q/ ECG pic shows anterior MI upon Hx he took saldenefil ( viagra )


what is the drug contraindicated to be taken in this case?
A-morphine,B-aspirin , C-nitrate ( I choose it cuz both cause V.D )

Q/ pt think he has cancer went to 6 doctors and examination was


normal ,but still he think he got cancer,Dx?
Hypochondriasis

Q/ pt with neck regidity and abdominal muscle regidity (clear


extraparamidal manifestations) ,which drug can cause these
symptoms?
Metochlopramide

Q/ 2yrs old boy with clear manifestation of ADHD which of the


following drugs might be given?
Atomaxatine

October 24, 2017 234


Q/ female pt with greenish yellowish discharge and itching , dx?
T.vaginitis

Q/ nearly same case (T.vaginitis was the diagnosis) , ask about the
ttt? Ceftriaxone

Q/ neonate smiles at how many weeks? 8

Q/ child rid tricycle at which age? 3 yrs

Q/ which one is considered as delayed milestone in a 13 months yrs


old child? Sit with support

Q/ what is the role used in burns? Role of 9

Q/ pt with burn over the anterior trunk and cercumfrential brun on


upper and lower left limbs ,pt weight is 70 Kg,calculate parklands
formula٤،١ ‫? تقريبا‬

Q/ what is the role of OCP in protection against illegal pregnancy ?


Prevent fertilization

Q/ S&S of schizophrenia,but for 1 month the doctor diagnosed him as


schizophrenia, what is wrong with the diagnosis?
Duration ( schizophrenia more than 6 months)

Q/ finger nail abrasions complain of photophobia , what is the


management?

October 24, 2017 235


Antibiotics and referral?

Q/ old pt with neck stiffness and parasthesia on morning exam there


was paraspinal muscle spasm , ttt?
Physiotherapy

Q/ old pt with neck pain and loss of sensation in the left arm , best
step in Dx? Spine MRI

Q/ female with left upper outer mass in the left breast what is the L.N
to be examined?
Pectoral

Q/ what is the muscle responsible for unlocking the knee?


Popletus

Q/ stab wound in the buttocks there is defect in the lateral rotation of


the leg ? A-gluteus max, B- adductor longus

Q/ commonest cause of olecranon bursitis?


Repetitive truama

Q/ pt compliant of pain in medial epichodyle he is a golf player,


management ? Refine golfer

Q/ pain in the wrest with positive tunnel sign which nerve is affected?
Median nerve

October 24, 2017 236


Q/ pt with thenar muscle atophy what is the nerve affected?
Median N

Q/ case of post partum hemorrhage you gave oxytocin no stoppage


and the diagnosis was uterine atony, next step?
Bimanual compression

Q/ what is the organ affected if you ligate anterior iliac artery?


A-ovary ( I choose it ) ,B- instestine,C- bladder

Q/ OCP cause hyperkalemia ?


Levenogestrol ( not sure )

Q/ clear case of paraneaplastic syndrome, which one you will order


for dx ?
Vinylmadillic acid

Q/ pt complain of chest pain , discribe it as close fist is crushing his


sternum , Dx?
Myocardial ischemia ( not sure ) other choices pulmonary embolism and
pericarditis

Q/ long case ... on auscultation you find mid diastolic murmur in the
heart base ,dx?
Mitral stenosis

Q/ long scenario positive finding were radiofemoral delay and on CXR


there is rib notching ,dx?
COA

October 24, 2017 237


Q/ pt complain of constipation on abdominal examination there is a
mass in the lower abdomen you did US which confirmed the
presence of a mass in which you decided to do colonoscopy which
find 2 polyps with high grade hyperplasia , when you will follow up
the pt by colonoscopy?
A-after 1 year, B-3 yrs ,C-8 yrs

Q/ pt came to PHC complaining of fatigue and polyuria , investigation


: RBS 450 , positive ketone body , what is the predominant type of
kentone body is present in the urine?
Acetoacetate

Q/ S&S of stroke , what is the first step in the management?


Brain CT

Q/ best investigation for detection of retroperitoneal hemorrahge?


Computed tomography ( CT) ‫ماجابوه باالختصار‬

Q/ which one is true about Pap smear ?


Decrease the incidence of cervical ca dramatically

Q/ best site to take Pap smear ?


Transformation zone

Q/ in Pap smear you find hyperplasia in ectocervix , next step?


Colposcopy directed biopsy

October 24, 2017 238


Q/ pt with brown macules on skin with axillary fleckring dx is NF ,
what is the mode of inheritance?
Autosomal linked

Q/ female with developed secondary sex character sparing axillary


and pubic hair in investigation there was high testosterone,dx?
Androgen insensitivity syndrome

Q/ painless rectal bleeding on investigation you diagnosed the case


as meckel's diverticulum ,what is the commonest site for it ?
Lower ilium

Q/ difference between Hb and Hb A1C ?


Glycosylation

Q/ which one of the following glumerulonephritis disease there is anti


basement membrane antibody?
A-IgA neohropathy ,B- post streptococcal glumerulonephritis ,C-SLE (
choose it )

Q/ murmur that change in changing the position?


Innocent murmur

Q/ type of leishmaniasis cause skin manifestation?


Tropica

Q/ a wife of a man diagnosed as having gonoreahl infection she was


wooried about transmission of the infection from her husband what is
the best investigation to exclude gonorreah infection?

October 24, 2017 239


Gram stain

Q/ which type of cells will be high in viral infection?


Lymphocyte

10/10
Cardi:
Q/ MI and U choose the appropriate managment

Q/ History of angina that become more frequent and lasted longer


time without elevation of cardiac enzymes .. answer was Unstable
angina

Q/ TOF and the answer was pulmonary stenosis

Q/ Young with family history of sudden death , answer was


hypertrophic cardiomyopathy

Q/ Patient with MI and i think in the 5th day he deteriorate , l choose


acute mitral regurgitation ( study MI complications and when they occur )

Q/ Patient with atrial fibrillation and U describe for him a medication


.. what is the mechanism of action of that medication , l think they
mean warfarin

Q/ Xray of patient with pleural effusion , what U gonna hear during


auscultation

October 24, 2017 240


Q/ Which type of congenital heart disease least associated with
infective endocarditis

Resp:
Q/ Patient with lung cancer that secrete keratin : SCC or metastatic
melanoma or adnocarcinoma or ....

GI:
Q/ Patient with jaundice , labs shows high indirect bilirubin and total
bilirubin .. question was about the gene or the mutation
Q/ Case of appendicitis

Q/ Case of pancreatitis and the cause i think gall stone

Q/ Case of peptic ulcer and the answer was H.pylori

Endocrine:
Q/ case of thyroid storm and the answer was b blocker

Q/ Case of women with hyperprolactenemia and they found pituitary


adenoma .7 cm what is the management : cabergoline or bromocroptine
or surgery ( l choose cabergoline because it is better tolerated than bromo
, surgical intervention if symptoms progress despite appropriate medical
therapy and the recurrence rate after surgery is high according to step up
medicine book)

Q/ Case of SIADH , they wrote lab results and they want the diagnosis

October 24, 2017 241


Q/ Diabetic patient of NPH and regular insulin twice before breakfast
and dinner , glucose was high at 4pm , how to manage :
Increase NPH before breakfast
Increase regular before breakfast
Increase NPH before dinner
Increase regular before dinner

Q/ read about oral hypoglycemic drugs , mechanism of action and


side effects

Q/ Metformin act on the muscle by Increase glucose release , or


Stimulate fat oxidation , or Stimulate gluconegoenesis , or Increase
glucose uptake

Q/ Diabetic patient with proteinurea , answer was add ACEI

Psychiatry:
Q/ Many questions about medications
‫ندها اعراض شيزوفرينيا من شهر‬
‫والدكتور شخصها شيزوفرينيا‬
‫ايش اللي يخليك تستبعد الشيزوفرينيا؟؟ المدة‬

✅‫ شهور مو شيزوفرينيا‬٤ ‫النها اقل من‬

Gyne/ob:
Q/ definition of spontaneous abortion , answer is 3 consecutive
abortions

Q/ Case of eclampsia , how to stop convulsion , answer was mgso4

October 24, 2017 242


Q/ Case of preclampsia ( weight gain , proteinurea , headache ) with
IUGR , What other clinical finding support IUGR
Oligohydramnios
Polyhydramnios
Liver enzymes

Q/ case of patient with recurrent abortion and she came @ 20 weeks ,


cervix length was 30 .. what to do:
Cerculage
Progesteone
I forget the choices

alot of questions about vaginal discharge

Q/ Pregnant with dizygotic twins , placenta will be :

Di di✅‫بغض النظر عن السكس‬


Or mono mono
Or di mono‫ونفس السكس‬
Or
Mono di‫ومو نفس السكس‬

Q/ question about PCOS and the answer was endometrial cancer

CNS:
Q/ Patient has mutation in NF1 and the answer was neurofibromatosis

October 24, 2017 243


Q/ Patient with gullain barre , what cell is affected ? Bipolar or schwan
or ....
Q/ 4 cases about alzheimer disease

Q/ Patient with left side paralysis (stroke) what to do ? CT or aspirin or


Rheumatology:
Q/ patient with joint tenderness and synovial fluid analysis , answer
was septic arthritis

Q/ Read about synovial fluid analysis

Q/ Patient with osteoarthritis and they ask about the name of nodes
over the PIP joint (PIP is bochard's node , DIP is heberden nodes)

Q/ Case of gout and the answer was indomethacin

Q/ Around 3 cases about back pain and its management

Q/ Typical scenario of measles with oral lesion and they ask about
the diagnosis

Measles✅
Kawazaki

Q/ U prescribe roaccutane for women , what is the serious side effect


, l choose birth defect

Q/ paracetamol antidote is acetylcystine

October 24, 2017 244


Q/ Aspirin overdose cause : metabolic acidosis and respiratory alkalosis

Q/ U give patient naloxone , it is antidote for ? The answer was


morphine

Q/ Injury to posterior vagal trunk , which organ will not be affected

Q/ 4 cases about epistaxis management

Q/ Many questions about nerves : which transmit the pain to the ear ,
which responsible about anterior 2/3 of tongue ...... ,

basic questions

11/10/2017
Q/ Best exercise for CAD is:
__anaerobic __isotonic__ isometric

Q/ Vonwillbrand: increase ptt

Q/ patient came to neuro clinic complaining of prolonged fixed


movement )‫ (يعلق على حركة فترة طويلة‬when you examine the patient had left
hand flexed and right arm flexed and placed on hip, you try to move
but patient is unresponsive: Catatonia (I think)

Q/ Green stick fracture x ray what is the managment

Q/ blue greenish Colony (pseudomonas)

October 24, 2017 245


Q/ patient htn in diuretics on investigation there was increase ha1c
what to add? Acei

Q/ patient thinks that he is not in this world and feels disconnected:


Depersonalizations

Q/ pseudo dilution in sicke cell patient may be due to:


__anemia __hemoglobin c__ hemoglobin d __increased protein

Q/ type of mutation in thalassemia and sickle cell

Q/ Marfan syndrome case ( typical features) what is the type of


inheretence? Autosomal dominant

Q/ temporal arteritis complication: blindness

Q/ patient has lateral headache, tenderness over the area, jaw


claudication what is the diagnosis:
Temporal arteritis

Q/ diabetic patient with diffuse dull headache no neurological


findings what is dx?
__bacterial sinusitis __TB meningitis __ brain abscess (I chose this)
__viral meningitis

Q/ watery eye discharge: viral conjunctivitis

October 24, 2017 246


Q/ 2 cases about myasthenia gravis typical scenario how to diagnose
and the other q what it's mechanism I think it's autoimmune

Q/ which of the following is indication for surgery in crhons disease:


__fistula (I chose this) __intestinal obstruction...

Q/ patient with std using sulfa drugs how would the lesions look?

Q/ htn urgency and emergency

Q/ ecg (I think one about __heart block, sick sinus,they asked about
managment
__st segment elevation, LBBB they asked about diagnosis )

Q/ Cancer associated with pcos:


__lung __breast __ovarian __endometrial

Q/ how to diagnose pcos? Fsh/LH ration

Q/ barvo virus 19 case

Q/ acute sequestration crisis in secular patient how to manage (best


initial management)? (Splenectomy or fluid and analgesia)

Q/ Mersa:
2:66

October 24, 2017 247


Q/ pudendal nerve block , which of the following structures will be
not blocked by the anesthesias : 
Rectum

Q/ contraindications for circumcision is :


Hypospadius

Q/ patient with painful and swelled knee and uretheral pus discharge
when ruined culture done it was negative for 3 organisms: I think
reactive arthritis

Q/ Most common cancer associated with depression: pancreatic

Q/ starry appearance :
Burkitt's lymphoma

Q/ aplastic anemia case ( lab readings) all depressed

Q/ short stature and skin pigmentation, including café au lait spots


diagnosis was Franconia anemia what type of inheritance? Autosomal
dominant

Q/ anti basement membrane antibody: goodpasture syndrome

Q/ patient is severely burned how to manage?


__give the whole amount of fluid in the first 24 hours __other choices like
give first half in fort 7 or 9 hours

Q/ where to insert chest drain? Midaxillary 6th intercostal

October 24, 2017 248


Q/ neck and hand rigidity and stiffness after ingestion of a drug:
metclopramide

Q/ criteria to diagnose ADHD: I chose 6 inattentive, 3 hyper active, 1


......

Q/ erectile dysfunction caused by anti depressant: ssri (it was a name


I think paroxetine

Q/ characteristics of bacterial vaginosis: no itching no burning


sensation

Q/ female G3 p0 known history of incompetent cervix came with


bleeding 3 hours prior to admission then stopped what could be the
cause of bleeding:
-leuteal phase defect or -cervical incompetence -infection

Q/ how to diagnose premature rupture of membrane? Liquor


biochemical something like that😁

Q/ female with gestational diabetes what to tell her?


__she could develop type 2 DM (not sure)

Q/ mastoiditis antibiotic:

Q/ pain over anatomical snuff box: scaphoid fracture

Q/ fracture to the neck of the fibula, patient's feet is falls when he


rises it to walk (looks like a foot drop):

October 24, 2017 249


__common peroneal nerva __deep peroneal nerva

Q/ first lymph node to examine in breast cancer: anterior groupe

Q/ ssri mechanism of action: limits serotonin absorption so increase


seretonin level

Q/ patient received nalaxon in ER what is the toxicity: morphine

Q/ a drug 100 times more potent than morphine: phentanyle

Q/ what to consider before treating palliative patient with opioids:


__nonsteroidal anti inflammatory __antidepressants

Q/ how to treat allergic rhinitis: antihistamine drugs

Q/ bilateral breast cancer:


__medullary __ mucinus
)I think medullary(

Q/ Intrapapillary breast cancer:


bloody niple dicharge

Q/ patient 40 years old with colon ulcers and pseudoplyps


recommendation for screening:
__colonoscopy every 6 months __colonoscopy and Ct every year
__colonoscopy every year

October 24, 2017 250


Q/ 2 positive Pap smear what to do next: Colposcopy

Q/ dehydrated child how to give fluid:


__50ml/kg in the first .. hours then 100ml/kg __100ml/kg in the first then
50 ml/kg

44years old patient with regular menestrual cycle all examination


normal except for clear nipple discharge what to do?
__neurostudy or imaging � __prolactin hormone __mammogram __ACTH

Q/ Hormone that affect melanin: ACTH

Q/ Lung cancer adenocarcinoma responsible cell: Clara cells

Q/ Viral infection what is expected to rise?:


__basophils __neutrophils __macrophages __lymphocytes

Q/ patient developed symptoms and the CBC of hemolytic anemia


after taking quinine antimalarial drug, due to deficiency of which
enzyme?
__g 6 p hydrogenase (i think)

Q/ which rna is made in the neucleulus?


rRNA

Q/ the bond between mRNA and r RNA:


Haydrogen

October 24, 2017 251


Q/ Mechanism of diabetic nephropathy in early stage:
__Sclerosis of the arterioles and increase in filtration rate in the beginning
(I think this is the right answer)
__sclerosis of the arterioles and disruption of protein binding

Q/ young patient came with strider and barking cough, no other upper
respiratory symptoms no prodorme, after 3 hours it subsided patient
is known for atopic dermatitis what is the dx:
__croup __some allergic upper respiratory tract reaction

Q/ a young elementary school girl was diagnosed with measles and is


absent from school for 2 days how many days her sick leave will be?
__8 __10 __4

Q/ patient case suffering from hypopegmented lesions and falling of


skin appendages what is your diagnosis?
__I think leprosy

11/10/2017

Q/ lip swelling for 3 years deficiency in which enzyme?


C1 esterase inhibitor

Q/ massive plural effusion?


thoracocentesis
needle decompression
Inhalator in high dose “ something I don’t remember😝

October 24, 2017 252


Q/ 2 Q about squamous cell carcinoma of lung
First was “ x- ray“
Second I don’t remember💔😝

Q/ HIV pregnant women � something drop from 400 to 200 so how


she will deliver?
Spontaneous vaginal

Q/ proximal Tibial fracture palpable peripheral pulse ABI: .85 what to


do:
A)angiography
B) Doppler US. “ I choose this“
C) CT.

Q/ old man with abnormal hand and head movement he toke drug 1
day before what is the drug:
metoclopramid

Q/ MVA anterior injury of the pancreas and anterior leakage of


pancreatic fluid will collect:
Omentum bursa

Q/ Example of opening question?


Tell me about the pain

October 24, 2017 253


Q/ research Q about OR for smokers and non-smokers what is
something like a relation or something like that?
Strong of association

Q/ What cancer metastatic to stomach?


Lung “ I choose this“

Q/ man doing proctectomy i think and he came later with metastatic


cancer where?
vertebral column “ I choose this“
lung
Kidney
And mmmm I don’t remember the choices

Q/ doctor diagnose the patient with


schizophrenia 1 month what is against his diagnosis?
Duration

Q/ migraine + HTN?
No beta blocker in choices
I choose CCB

Q/ which drug if you stop abruptly cause hypertensive crisis?


No beta blocker in choices
I choose Clonidine

Q/ propylthiouracil action?

October 24, 2017 254


Prevent idonation ... etc

Q/ CP patient all limbs affected but the lower limbs less spastic?
Quadriplegia

Q/ honeymoon cystitis?
E.coli

Q/ male patient with pain , effusion , erythema, swelling in both knee


joints
Ask about treatment?
aspiration and antibiotics “ I am not sure“

Q/ boy he didn’t complete homework, didn’t listen to mom , don’t love


sharing and another things what type of ADHD?
inattentive “ I choose this“

Q/ many Q about thyroid in medicine I don’t remember ☹💔🚶� review


what you should do if there’s mass and about hypo &
hyperthyroidism, there’s investigation in the Questions

Q/ Q about scabies

Q/ Most common malignant lesion need to be removed?


Erythema gyratum

Q/ Q melanocytes about cell producers or something like this?


Corticotropes

October 24, 2017 255


Q/ marffan syndrome descriptions long arm and ask about mood on
inheritance?
AD

Q/ blue sclera and another description?


osteogenesis imperfecta

Q/ epiglottis organism?
influenzae

Q/ descriptions like psoriasis but there’s no psoriasis in choices so I


choose
SLE

Q/ patient using sulfa drug there is lesion in glans of penis ask about
descriptions?
I searched for the answer before my exam And I didn’t found the answer
💔😩
But I choose
Scaly , erythematous and ulceration🤦🚶
Another option
Nodules, erythematous and ulceration

12/10/2017
Q/ Anemic pt on management , he came ē black stool , what causing
drug ?
)....-ferrus sulphate - iron dextan (

October 24, 2017 256


Q/ anemic pt ē abdominal pain and fever diagnosed ē gastritis how to
treat ?
) .... - Im iron - iv iron (

Q/3 y girl ingest 20 pills of isonizaied what the ttt ?

Q/ pregnant woman when to take vaginal swap for meningiococcus or


something about infection to protect the fetus from this infection ?
)... - weeks - 40 weeks - 26 weeks 00 (

Q/ when the body exposed to ionized radiation what the effect of


radiation on DNA ?

Q/ bond between r-RNA & m-RNA ?

Q/which is synthesized in neuclouls ?


)... - r-RNA - mRNA (

Q/ women diagnosed with trichomonas and she is asymptotic, when


to start treatment ?
).... -immediate - when symptomatic (

Q/ when birth consider premature ?


)....- 16 - 00 - 43 (

Q/ young pt sickle cell has penis pain and edematous shaft of penis ,
with pic , what the Dx ?

October 24, 2017 257


)....- periapism- byerine Disease - Peronism (

Q/ women diagnosed previously with lichen sclersus in valva , she


has mass in valva and biopsy taken , what suspect to has ?
squamous cell carcinoma- adenocarcinoma - squamoadenocarcinoma - (
) melanoma

Q/ which ligament protect uterus from prolapse ?


)....- broad - round - utersacral (

Q/ young girl come HTN crisis , antiHTNsive not effective , she has
Hx Of hypotension during appendectomy ?
pheochromocytoma - renal artery stenosis - renal vien thrombosis - (
) essentialsHTN

Q/ abdominal truma with pancreatic body injury, pancreatic juice


split anteriorly , where this juice will collected ?
)....- Omental sac - left para colic - sub hepatic (

Q/ what is most common cause of death in flam burn?


)...- hypovolemic shock - inhalation Smoke (

Q/ pt ē DM suddenly develop weakness in half of his body one hour


ago , no dysarthria , no headache, what’s the management ?
)....-aspirin - anti plasminogen (

Q/ pt suddenly feel the environment is strange ?


)..... - depersonalization- derealization (

October 24, 2017 258


Q/ dose of nitrofurntine ?

Q/ child 9 or 12 month asthmatic on corticosteroid which vaccine to


give ?

Q/ -child always alone , dose not have toys , doesn’t play with others
, what is the relation cause or something?
)..... - intelligence- interpersonally (

Q/ ‫طفل بالع بطاريه وش تسوي له اول شي ؟‬


Observeation for 12 h
Im glucagon
Immediate endscopy
Remive it by foly catheter

Q/ ‫عالج دايرتيك يسوي حصوه‬

Q/ ‫باراسايت في لحم البيف‬

Q/ Polymalegia + polymyositis

Q/ hair growth ‫عالج ابليبتك يسوي‬

Tetratology ‫ مكونات‬/Q

Q/ ‫واحد بيدخل عمليات نحوله على انسولين‬

October 24, 2017 259


Q/ ‫ والباقي نيقاتيف‬+HBcantibiy
Chronic , Acute , booster , vaccination

Q/ Bite by his brother and there is 1 cm injury , what you do :


Amoxcillin
Surgical suteur

Q/ ‫واحد جاء الطواري ومقروص بلدغه ثعبان وش اول شي تسوي ؟‬


Incision on site bite
Mechanincal ‫تمص السم‬
‫تثبت اطرافه‬
‫تربط‬proximalً‫لمكان اللدغه‬

Q/ Nissan opreation

Q .‫ مرض باهجت وش‬/HLA‫حقته‬


HLA B51

Q/ Old Pt have recurrent DVT and obstruction SVC , what the cause
?
Lung carcinoma
‫ناسي الخيارات الباقيه‬

bacterial vaginosis ‫ متى تسوي فحص لل‬/Q


All trimester
First trimester
second trimester

October 24, 2017 260


‫ سوال حق التوينس نسخ‬/Q

antibiotic ‫ متى تعطي‬c/s ‫ وحده بتسوي‬/Q


‫قبل العمليه‬

12/10/2017

Q/ A three year-old girl presented to Emergency Department with


fever, vomiting and

Q/ abdominal pain which began 10 hours ago. Radiological


examination confirmed a dilated
intestinal pouch attached to the anterior abdominal wall. Her
diagnosis was the persistence
of a Meckel’s diverticulum.
Which of the following sites will the surgeon look for this
diverticulum?
A. Lower Duodenum
B. Lower Jejunum
C. Lower Ileum
D. Cecum
Answer : C

Q/ Which is the narrowest part of male urethra and therefore most


likely to be damaged

October 24, 2017 261


during catheterization?
A. Penile
B. Prostatic
C. Membranous
D. Middle spongy
Answer : C

Q/ A 45 year-old man, who has diabetes and has been on metformin


and glimepiride for the
last four weeks, presents to the clinic with poor glycemic control. A
history also confirms
poor dietary management.
Which of the following should be prescribed now?
A. Acarbose
B. Repaglinide
C. Tolbutamide
D. Pioglitazone
Answer: A

Q/ One parent has schizophrenia: 12%

Q/ Father case of hemophilia A mother carrier 50 %

Elbow fracture on lateral x_rey


Posterior fat pad visible - ALWAYS ABNORMAL

Q/ HIV negative with hx of sexual when to repeat the test ?

October 24, 2017 262


Q/ Embryo of folcular ovary ?

Q/ Cancer association with depression ?

P 53

Q/ Gen associated with tumor of the kidney in the upper lobe ?

Q/ Clear case of NF ask which mood of inheritance ?

Q/ CF ch 7q

Q/ cll leukemia philadelphia t ) , (

Q/ Spiral fracture to child what to do


‫ علشان كذا نحولها لألخصائي االجتماعي‬child abuse ‫? انا أشوفها انها ممكن تكون‬

Q/ High lipid LDL receptor

Q/ High TG and LDL ‫ اخترت‬life style modification

Q/ Post Mi day 5 sudden SOB what the cause ?


) 1)acute aortic regar. 1)acute mitral reg 3

Q/ Clear case of septic arthritis

October 24, 2017 263


Q/ myasthenia gravis management ?

Q/ Valve itching with tree like what RX ?

Q/ End stage liver disease with fungel infection RX ?

Q/ Prephral smear target cells and basophilic stippling ?


Thalassemia 😿
Q/ Bio in and out anergey ?

Q/ Protein EPR

Q/ Longest phase ?

Q/ Nerves injury Median Ulner. And auxiliary

Q/ Muscle injury addactor longes

‫* طفله شالها ابوها من يدها تشكي من الم بالمرفق‬albow flexion and pronation hand
‫ايش مكان االصابه ؟‬

Q/ Forehead LN drainage ?

Q/ Breast cancer ‫ يعتمد على‬number of LN ?

Q/ Antipsychotic ‫ تسبب‬Constpation ‫? طبعا كل خيار عباره عن دوائيين ؟‬

October 24, 2017 264


Q/ ‫يغطي التلفزيون الن احد يأمره بذالك ؟‬

‫* تعريف‬delusions /Q

.‫* ايش يعني‬CI 95% /Q


Q/ The proportion of those
intervals that contain the true value of the parameter will match the
confidence

12/10/2017
A file contain 72 new question written by rawan youssef
https://goo.gl/w2LgGT

12/10/2017
Q/ Child with painless neck mass and cough and sore throat and
generalized pruritis?
A-Hodgkin's lymphoma.
B-Lyme disease.
C-Infectious mononucleosis.

Q/ Child with iron toxicity several hours ago , what will u do?
A- gastric lavage
B- activated charcoal
C- iv deferoxamine.

Q/ Bisphosphonate drugs mechanisms, and then asked which one ?

October 24, 2017 265


Q/ -POS have a risk to cause which type of tumor?
A- endometrium cancer
B- ovarian cancer
C- cervical cancer

Q/ Which one of the choice secrets gulcocorticod hormones?


A- reticularis
B- glomerulosa
C- medulla
D... -

Q/ ADHD child , can't involve for anything for long time , like in
school or even when he playing a video games he can't finish it and
searching for something else to do and so on, which type of ADHD he
has?
A- inattentive
B- impulsive
C- hyperactive impulsive.

Q/ Patient have depression and came to the clinic complaining of


decreased lipido which drugs you will shift your patient to ?
A- paroxtein
B- TCA
C- amytriptaline

Q/ Patient have fracture in his tibial it is opened fracture <1cm , what


you do?
A- close reduction with cast

October 24, 2017 266


B- depridement and open reduction by intermedulary nail.
C- give him AB .

Q/ questions about post traumatic stress disorder, I can't remember


exactly what was about!

Q/ pregnant women already have a history of two miscarriage, after


that the doctors diagnosed her as cervical incompetence, she's now
pregnant and have a bleeding, what is the cause of her condition? "
actually it's a stupid question I can't even know how to answer it !
The answers were confusing "
A- cervical incomptence.
B- chromosomal disorders
C- ectopic pregnancy

Q/ case of epidedmorchitis.

Q/ contraindications for circumcision?


Hypospadius.

Q/ pictures of chancre painless ulcer on the penis how to treat? -->


primary syphlis.
A- penicillin V
B- pencillin G and ..." my answer"
C- steroid

Q/ picture of one eye conjunctivitis how to prevent the infection for


the other eye?
A- eye drop

October 24, 2017 267


B- eye ointment
C- hand washing "my answer"

Q/ Mother have a photophobia and severe eyes pain after she


received accidentally fingernail trauma by her daughter how to
manage?

Q/ child have bilious vomiting?


A- pyloric stenosis
B- deuodenal atresia
C- meckles diverticulum

Q/ site of Mickels diverticulum?


Lower ilium

Q/ most common symptoms of lower intestinal obstruction ?


Absolute constipation.

Q/ wernicke's area injured, which type of aphasia!

Q/ neurofibromatosis 1 gene?

Q/ characteristic sign of retinoblastoma!

Q/ Patient complaining of face flushing and head heaviness in the


morning that's relieved during the day , CXR showes bilateral hilar
mass , he is smoker for 30 years?
A- hodgkins lymphoma

October 24, 2017 268


B- lung cancer
C.. -

Q/ Injury to posterior vagal trunk will affect the?


Bladder
Descending colon
Esophagus

Q/ Old age patient came to the Er toxic and comatose and


hypotensive!
A- septic shock
B- insulin overdose

Q/ cancer in the middle of esophagus, which type?


A- adenocarcinoma
B- Scc

Q/ Unilateral knee swelling and pain, knee tap labs: cloudy yellow
color, mucoid, WBC-pmn 00 (normal less than 200,) lymphocytes
,%11 what is the diagnosis?
A. Gout
B. Septic arthritis
C. Rheumatous Arthritis

Q/ glucocorticoids from which layer ?


A-Reticularis
B-Glomerulosa
C-Medulla

October 24, 2017 269


D-Faciculata

Q/ Patient exposures to asbestos what will present in chest x ray?


A- bilateral lung fibrosis
B- interstitial lung disease

Q/ a man work in the new office and he experinced cough and


wheezing and respiratory symptoms I don't remember exactly..
otherwise he is fine and the condition return once he back to this
office,
A- asthma
B- allergic pneumonitis

Q/ 02yo athlete male gain 7kg lately and he has all characteristic of
normal puberty, he also have a foul smell breathing ?
A- anabolic steroid
B- puberty
C..-

Q/ an athlete guy left a heavy weights as usual, this time he


experienced sudden pain while he lefts the weight , examination
reveled normal range of motion in the shoulder, what to do ?
A- reassurance
B- declofenac
C- physiotherapy

Q/ What’s minimum volume of blood to do a culture:

October 24, 2017 270


13_23ml

Q/ Femal 150k.g. Weight and height 160 according to BMI ?


I Obesity
II obesity
III obesity

Q/ Patient exposures to asbestos what will present in chest x ray:


A.Hyperinflation
B. Bilateral lungs fibrosis (my answer )

Q/ Patient complain of abdominal pain I think with diarrhea , also his


wife noticed he had SOB and tightness. Doctor order 5-
hydroxyindoleacetic acid in urine Which cell responsible ?
A.chromaffin cell (the answer) [carcinoid syndrome ]
B.Enterocell
C.Lympho cell
D.Goblet cell

Q/ which drugs decrease mucus secretions in emphysema ?..


A. Steroid
B. Ipratropium (my answer )

Q/ premature menopause at which age:


A. 30
B. 35
C. 40 the answer

October 24, 2017 271


D. 45

Q/ which markers of HBV is present in window phase:

Q/ Patients have D.M. On meatformi 1 g and another anti dm present


with increasing
blood glucose at morning, what you will give :
A. NPH
B. Lispro my answer
C. Regular

Q/ female on anti dyslipidemia drug present with flushing what is the


drug:
A. Niacin my answer
B. Atorvastatin

Q/ what is the MOA of glipizide ?


A. Increase insulin secretions from pancreas

Q/ patient diagnosed with Barrett's esophagus will have :


A. Adenocarcioma
B. Squamous cell carcinoma my answer

Q/ aphthous ulcer with which disease :


A. Celiac

Q/ Case of MG take neostagmin then present with worse symptoms


to ER what is the Rx? No IVIG in answers

October 24, 2017 272


A. Plasmapheresis my answer

Q/ treatment of sever ptosis no other details ?


A. Frontalis suspension my answer
B. Levator resection

Q/ Female c/o dysuria frequency low steam void, she had recurrent
UTI take multiple antibiotic without benefits, what is the diagnosis?
Traumatic urethritis
Interstitial urethritis
DM
Candida

Q/ Most common cancer cause depression ?


Prostate
gastric
renal
pancreatic

Q/ Female had 2 abortion in second trimester, last one done for her
D&C diagnosed as cervical incompetence now pregnant in 5 week
present with vaginal bleeding and open cervical os what is the cause
of bleeding?
Luteal phase defect
Cervical incompetence
Asherman syndrome
Chromosomal cause

October 24, 2017 273


Q/ Dizygotic twines which is true?
1placenta 2 amointic fluid regardless sex
1placenta 1 amontic same sex
2placenta 1 amontic same sex
2placenta 1 amontic regardless sex

Q/ Old age HTN, DM had painless vision loss examination show


drusen diagnosis?
Central vien
Central artery

Q/ Patient complain of vertigo for 22-46 minutes + tinnitus + hearing


loss, audiogram show low frequency hearing loss, diagnosis?
Meniere's disease
Acoustic neuroma
Vestibule neuritis

Q/ Patient has lung infection taking antibiotic then patient develop


flushing in face and hand then came to doctor culture show
methicillin resistant gram + in cluster
Which antibiotic he took ?
Vanco
Cephalo

Q/ Patient had fibroid 3Q/ 4 cm take tamox after 1 years come with
vaginal bleeding us show fibroid 6cm with endometrial hyperplasia
what is the cause of bleeding?
Fibroid
endometrial hyperplasia

October 24, 2017 274


Q/ Patient complain of chest pain, vertebral pain, a lot of complain
with pain also has past history of complain of pain and nausea and
vomiting with irregular menses weakness in left leg Malingering and
factious ruled out, diagnosis?
Pain disorder
Somatization disorders

Q/ Father had hemophilia A and mother carrier percentage of child


will have it ?
10
03
,0
233

Q/ Patient thin and tall came to doctor because 2 of his brother died,
you afraid of which of the following?
Hypertrophic cardiomyopathy
Aortic root aneurism

Q/ Patient has chest pain on left side examination show plurtic chest
pain friction rip on left side next step?
Chest x-ray
21lead ecg
Echo refer to cardio

Q/ Patient had DM with HTN had renal transplant after 1 month show
rejection biopsy show hla class 1. Which cell responsible ?
Macrophage

October 24, 2017 275


Cd 4
Cd8
Nk

Q/ Patient complain of abdominal pain I think with constipation, also


his wife noticed he had SOB and tightness. Which cell responsible ?
chromaffin cell
Enterocell
Lympho cell
G cell

Q/ Most common type of physical urticarial :


Water
Cold
Dematographisim
Cholinergic

Q/ Most common cancer orginate from which sinus


Ethmiod
Frontal
Sephniod

Q/ Patient has palpitation for 1 months not associated with chest pain
nor tightness last one yesterday lasting for 10 minutes now came to
doctor. Echo show no abnormalities ejection fraction 65%
There is ECG it show irregular rhythm but there is p wave last strip
contain 13 or 11 QRS
Treatment :

October 24, 2017 276


Aspirin
Digoxin
Propranolol
Calcium channel blocker

Q/ Female patient will get married had bilateral vitiligo in hands since
3 years lesion increasing in size, want to get rid of it, most
impropriate step?
Graft
Melanin transfer
Continue medication
Stop medication

Q/ Patient has epiglottis symptoms with strider and distress next


step?
Antibiotic
Admit to icu and refer to ent
.…No intubation in choices

Q/ Patient take a lot of sleeping drugs unresponsive pulse rapid


weak, show reflex gasping breathing, give 2 breathing by mask, next
step ?
Wait for blue team
Perform CPR
Intubate

Q/ If the husband has HIV and he refused to tell his wifehis wife is
pregnant What will you do?
‫تجيب الزوجه و الزوج بالغصب و تخليه يقولها‬-١

October 24, 2017 277


‫تكلم ناس مختصين بذي االشياء‬-٢
ّ ‫تقول للزوجه و ما عليك‬-٣
‫بالزوج‬
‫الخيار الرابع نسيته‬

12/10/2017
Q/ Ttt of pericarditis
-aspirin
-coricosteroid

Q/ Injury to posterior vagal trunk will lead to


Bladder
Descending colon
Esophagus

Q/ Complication of PDA ligation is


-injry to vagus
-phrenic nerve
-recurrent laryngeal nerve

‫ واحد اخر يومين مااخد الثايروكسين‬/Q


٢٧ ‫كان ماشي على‬
‫ عالي‬tsh‫وال‬
‫كم حتعطيه الدوز الجديدة؟‬
١١١ ‫ وال‬٥٧

abs and antigens ‫ شارت للهيباتايتس بي‬/Q

October 24, 2017 278


window‫وبيسأل ايش اللي يكون في الـ‬

Q/ ‫ انو دكتور يقيم محاضرة(توعويه) عن الضغط و ادويته و جاك مندوب شركة دوا ضغط‬: ‫سؤال‬
‫ اش تعمل؟‬: ‫جديد‬

Q/ Most common anorectal disease in elderly


-fissure
-hmorrhoids

Q/ Mechanism of polio vaccine

Q/ Oxybutynin mechanism of action


-antagonist muscrinic
-agonist nicotinic

Q/ Prevention of postherpatic neuralgia

Q/ Stone at L3-L4 on xray:


-ureter
-minor calyx
Major
Pelvic

October 24, 2017 279


‫تجارب األطباء والطبيبات الختبار ‪SMLE 2017-2018‬‬
‫وضعناها كما هي وتذكروا أنه لكل شخص تجربة ووصف معين ربما يختلف عن‬
‫اآلخرين وانطباع مختلف‪ ،‬الهدف من وضعها معرفة الطبيعة العامة لالختبار‬
‫والنصائح المقترحة منهم جزاهم هللا خيراً‪.‬‬
‫ـــــــــــــــــــــــــــــــــــــــــــــــــــــــــــــــــــــــــــــــــــــــــــــــــــــــــــــــــــــــــــــــــــــــــــــــــــــــــــــــــــــــ‬
‫ـــــــــــــــــــــــــــــــــــــــــــــــــــــــــــــــــــــــــــــــــــــــــــــــــــــــــــــــــــــــــــــــــــــــــــــــــــــــــــــــــــــــ‬
‫‪1‬‬

‫الحمد هلل رب العالمين‬


‫اختبرت ‪SMLE‬‬
‫‪ 300‬سؤال فترتين كل فترة ثالث ساعات‬
‫الفترة االولى ‪ 150‬سؤال وكانت سهله الى متوسطة مكرر منها كثير من مذكرات الحرس والمدونة ‪03‬‬
‫كان كثير تقريبا ‪ %01‬اسئلة باطنة والباقي نساء واطفال واحصاء‬
‫الفترة الثانية ‪ 150‬سؤال‬
‫كانت صعبه اسئلة اناتومي وفيسيولوجي ونساء وجلدية وعيون اكثرها‪.‬‬

‫‪2‬‬

‫‪21/8‬‬
‫االختبار عامة معقول‬
‫اللي تكرر من ملزمة ‪ SMLE13‬ماتعدى ‪٪١١‬‬
‫‪ ٪٧١‬جديد بس معقول ومقدور عليه‬
‫‪ ٪٧١‬جديد وصعب‬

‫اول شيء الكوميونتي‬


‫جتني ‪ ٤‬أسئلة مسائل حسابية‬
‫و ‪ ٨٨‬تقريبا ً تعاريف وكالم اذا سويت بحث كذا معناته وش تبغى ؟ ‪specificity , sensitivity..‬‬
‫فيه حاسبه بنفس الكومبيوتر تحسبين بس اعتقد احسن حل تختارين عشؤائي النه معقده مره بتضيع‬
‫وقتك وتركيزك‬
‫اكثر شي جتني اسئله عليه االنيميا بانواعها وامراض الدم حوالي ‪ ٨٧‬سؤال‬
‫‪IDA ,SCA, Thalassemia, ITP, TTp‬‬
‫‪ ١‬اسئله ‪ ECG‬مع الصور‬
‫تكرر علي من االختبار التجريبي سؤالين‬
‫ومن اسئلة اللي اختبر قبلي وانتشرت اسئلته ‪١‬‬
‫ـــــــــــــــــــــــــــــــــــــــــــــــــــــــــــــــــــــــــــــــــــــــــــــــــــــــــــــــــــــــــــــــــــــــــــــــــــــــــــــــــــــــ‬
‫ــــــــــــــــ ـــــــــــــــــــــــــــــــــــــــــــــــــــــــــــــــــــــــــــــــــــــــــــــــــــــــــــــــــــــــــــــــــــــــــــــــــــــــ‬

‫‪October 24, 2017‬‬ ‫‪280‬‬


‫‪3‬‬

‫‪23/8‬‬
‫‪SMLE‬‬
‫أتكلم من واقع أخطائي ولو بعيده بعد شهر وش أسوي ‪.‬‬
‫‪. ٨‬إدفر المدونة ‪ ٨١‬لسببين "أبريل آخر تحديث أنا ذاكرته ‪" ..‬‬
‫أ‪ ٨١١ -‬سؤال تقريبا كما هم ‪ ..‬منها ‪.‬‬
‫ب‪ -‬حل المية الباقية تقريبا منها وأشرحه بعد شوي‪.‬‬
‫بطبيعة الحال ‪ ..‬بعد البريك الدنيا تغيرت وصار االختبار للوهلة األولى جراحة سياكل مدوي شصار ‪ ..‬بعد‬
‫أربع أنفاس يمكن بدت توضح شاشة االختبار ‪.‬‬

‫‪. ٧‬أي كيس سيناريو ‪ ..‬ال تنخدع بالالبات اللي حاطينها ‪ ..‬ركز في السؤال اللي قبل اإلجابات‪ .‬إذا فيه‬
‫إكس راي‪ ،‬فاعلم إنه موجود في المدونة ‪.‬‬
‫بطبيعة الحال الالبات أحيانا كثيرة تفيدك في اإلكسلوجن ‪.‬‬

‫‪ . ١‬سوالف ميتابوليك أسيدوزيز وربعها أسيد بايز باالنس ‪ ..‬سوي جدول صغير ‪ ..‬ينفعك من دون ما‬
‫تدري ‪.‬‬
‫جدول ال ‪ developmental milestones ..‬في تورنتو نوتس ‪ ..‬احفظه زي اسمك ينفع‬
‫بقوووووة ‪.‬‬

‫‪. ١‬أي سؤال يقولك فيه إي آر ‪ ..‬ركز إن المانيجمينت ما يطلع من ثالثة ‪ ..‬مينتين آير وياي ‪ /‬فلويد‬
‫ريسيسوتيشن ‪ /‬أدوية ‪ ..‬غير كذا مو شغلك ك إي آر ‪.‬‬

‫‪. ٧‬الريسيرتش ‪ ..‬تقريبا ‪ ٨٧‬الى عشرين ورقة في المدونة ‪ ..‬افهم وش صاير وكيف تحسب ‪ ..‬فيه‬
‫جدول مصطلحات في البداية ووش صاير ‪ ..‬هذي ‪ ٧١‬سؤال تقريبا لو إني ذاكرتهم كان ضمنتهم ‪.‬‬

‫‪. ٤‬البايسك ساينس ‪ ٨٧١ ..‬ورقة في المدونة ‪ ..‬اصحى تسحب عليهم ‪ ..‬حط في اعتبارك ‪ ٧١‬سؤال‬
‫منهم بالريوس جايينك إن شاء هللا‪ .‬أنا شفتهم وايد سحبت وقريب العشر اللي في مذكرة الحرس ‪..‬‬
‫والحين أطالع أقول ليتني ‪ ..‬أسئلة مباشرة ‪ ..‬يعني المفروض أنا شايف ‪ ٨١١‬الى ‪ ٧١١‬سؤال ومتطمن‬
‫بس مع األسف سحبت على الريسيرتش "قلت غير حسابات! ‪ ..‬وسحبت على البايسك مال المدونة ‪.‬‬

‫‪. ٢‬قسم السايك ‪ ..‬سهاالت ‪ ..‬بس في المدونة مو دائما موجود المانيجمينت ‪ ..‬وال تهون ما تترك سؤال‬
‫إال عارف وش العالج ‪ +‬ميكانيزم أوف آكشن ‪ +‬أعراض جانبية ‪ = ..‬األسئلة مال السايك اللي من‬
‫المدونة! أمورك طيبة ‪ ..‬مو من المدونة‪ ،‬أمورك طيبة‪ .‬ما غيروا شي بس أسئلة السايك الجديدة كذا ‪..‬‬
‫يعني تضمن حتى الجديد ‪.‬‬

‫‪. ١‬األسئلة الجديدة؛ فيه منهم اللي ال حول وال قوة ‪ ..‬أنا ما أدري وش قاعد أقرأ ‪ ..‬وفيه اللي تقول‬
‫"شفت ذاك السؤال؟ هذا اخوه بس قالبينه"‪ .‬يعني وانت تذاكر المدونة ‪ ..‬وال تهون أول شي تقرأ عن‬
‫المرض ‪ ..‬ميدبوليتس يكفي ‪ ..‬وإذا فهمت ليش هذا الجواب خطأ فاعرف إنّ إِ ْن شاء هللا أمورك‬

‫‪October 24, 2017‬‬ ‫‪281‬‬


‫سيمباتيك ‪.‬‬

‫‪. ١‬فيه أسئلة واجد من الجدد تقريبا بالشكل هذا ‪ ..‬ملتي ستيب بروبلم سولڤينج ‪..‬‬
‫يعطيك سيناريو ‪ ..‬اكتشف المرض ‪ ..‬عرفته؟ خل في رأسك ‪ ..‬وش عالجه؟ عرفته؟ خله في رأسك ‪..‬‬
‫وش العرض اللي ممكن يصير مع هذا الدوا‪ .‬أو مثال يعطيك العرض ويقولك ويش هو الدوا ‪..‬‬
‫سالفة لونج آكتينج ستيرويدز ‪ ..‬و ماو إنهيبيتيور ‪ ..‬وسيليكتيڤ سيروتونين انهيبيتور ‪..‬‬
‫وسيفاولوسبورين أنساها ‪ ..‬ليش إيسيتالوبرين وليش أميترالبتالين ‪..‬‬
‫يعني اعرف الدوا نفسه ‪ ..‬القروبات ‪ ..‬بيودوسونايد ‪ ..‬إيبراتروبيوم ‪ ..‬سالبوتامول ‪..‬‬
‫إذا مثبت إن مثال ‪ ..‬هذي نوبة ربو وانت مستوعب إن هذا مو شورت آكتينج ‪ ..‬عطه كروس ‪ ..‬ما فيه‬
‫في الخيارات ديكسا ‪ ..‬ويش تسوي؟ لو عارف وش هم اللونج غير المتعارف عليه هذي أمورك تمشي‬
‫سالم عليكم‪.‬‬
‫يعني وانت تذاكر البروتوكول والجايدالينز‪ ،‬اعرف الجنيريكس مع القروبات ‪.‬‬
‫باختصار ‪ ..‬سؤال جديد؟ ترى مهو موجود لتعجيزك وإنما مجرد اختبار معرفتك ‪ ..‬مع شوية شطة‪.‬‬
‫نفسين إن شاء هللا ويبين الجواب ‪ ..‬أو بي على طول ما يضر ‪.‬‬

‫‪. ٨١‬الجنين عمره كذا "احفظ ‪ ..‬الرئتين جاهزين خالل ‪ ١٤‬أسبوع قبل أعطي ستيرويد" ‪ ..‬الباقي ‪..‬‬
‫اعرف الفايزات مال الوالدة ‪ +‬الستيجات والسكشنات وأنت تقرأ ‪ ..‬فيه أسئلة زحمة زحمة على كذا ‪..‬‬
‫ادفرهم مرة وفِي المدونة بيجيك الخير ‪..‬‬
‫وال تنسى تعرف متى األنستيزيا إبي ديورال ومتى بودندال ومتى جي آيه ‪ ..‬واعرف وش اللي يصير له‬
‫بلوك مع كل نوع ‪ ..‬ها البودندال ويش اللي ما يتأثر فيه؟‬
‫سوف تشكرني شكرا ابن حالل وابن كلب ‪.‬‬

‫‪. ٨٨‬ادفر المدونة زيييييين ‪ ..‬ألن فيه تقريبا ‪ ٨٢١‬سؤال ما بتوقف فيهم ‪ ..‬وبتترك الوقت تتكتكت على‬
‫البقية ‪ ..‬وفالكم أعلى الدرجات يا رب ‪.‬‬

‫‪. ٨٧‬فيه رابط في جوجل يتحدث باستمرار كل ما طلع واحد حط أسئلته ‪ ..‬ذاكرها زين ‪ ..‬نصهم بتكون‬
‫تعرفهم أولريدي بس مال أمس واللي قبله حدثوه ‪ ..‬ينفعك في عشرة أسئلة ويمكن أكثر ما تدري ‪.‬‬

‫االختبار من توصل ثمان يدخلك ‪ ..‬وعندك بعد ال ‪ ٨٧١‬سؤال األولى بريك "تخلص متى ما تخلص بعد‬
‫‪ ٨٧١‬بريك" نص ساعة ‪.‬‬
‫تقدر تاخذه وتطلع تسوي اللي تبي ‪ ..‬وتقدر تسحب عليه ‪.‬‬

‫‪4‬‬
‫‪23/8‬‬
‫‪Most questions were about obs & gyne and medicine ( cardio , endocrine‬‬
‫‪& about two questions about hematology ) , 1 question about ophtha and‬‬
‫‪two about ENT , about 10 questions pediatrics and 15 questions about‬‬
‫‪Statistics‬‬

‫‪October 24, 2017‬‬ ‫‪282‬‬


‫‪5‬‬
‫‪40/1‬‬
‫بسم هللا الرحمن الرحيم‬
‫طيب انا الحمد هلل اختبرت اول اختبار‬
‫حقولكم من البداية للنهاية بالترتيب‬
‫اول شيء تعليمات‬
‫‪ -0‬ناموا بدري واصحوا بدري وافطروا كويس‬
‫‪ -2‬خدوا معاكم اصل الهوية او الجواز‬
‫‪ -3‬حياخدولكم صورة ف الزم يكون شكلكم كويس‬
‫‪ -4‬خدوا سناك ‪ +‬موية وخلوها ف اللوكر يصير تاكلوها وقت البريك‬
‫‪ -5‬ممنوع تدخل معاكم ساعة او ورقة او قلم او موية او حاسبة‬
‫‪ -6‬اول ما حتجلسوا ع الجهاز حيكون في تعليمات اقروها كويس وانتبهوا اول نا تخلص التعليمات‬
‫حيبدأ االختبار ع طول‬
‫‪ -7‬االختبار مقسم ع جزئين كل جزء ‪ 150‬سؤال ‪ 3‬ساعات‬
‫‪ -8‬ترا ما حتبدؤوا مع بعض عادي كل مين حيبدأ في وقت ومو الزم تلتزموا بال‪ 3‬ساعات اول ما‬
‫تخلصوا حتى لو كان باقي وقت كبير تقدروا تخلصوا وعادي‬
‫‪ -9‬حتجلسوا بالعباية ف البسوا اكتر شي مريحكم‬
‫‪ -10‬افضل تاخدوا لو جكيت خفيف اذا انكم من النوع اللي يبرد بسرعة‬

‫طيب بالنسبة لالختبار ذاته‬


‫االختبار فيه كل المستويات سهل ومتوسط وصعب وكتيييييير مكرر اقدر اقول ‪ %21‬مكرر‬
‫اللي مو مكرر يا سهل ويتحل يا صعب او معلومات جديدة حتى ما درسناها وما تتحل‬
‫السورس ميد ايزي و ‪03‬‬
‫برنامج االختبار ترا مرة حلو تقدروا تسووا هاياليت ع المعلومات المهمة ومارك لالسئلة اللي تبغوا‬
‫ترجعولها والحاسبة مع البرنامج وسهل استخدامها مرة‬
‫انا اختباري كان اغلبه بيديا وريسيرش وسايكاتري والباقي موجودين لكن اقل من دول ال‪3‬‬
‫مالحظة‪ :‬حتى لو كنتوا بتختبروا ف نفس اليوم ما حيتكرر االختبار بينكم‬

‫‪6‬‬
‫‪My exam updates.‬‬
‫‪Over all it was not easy.. Very long, time consuming scenarios.‬‬
‫‪Scenarios were familiar but answer option were very confusing. Like‬‬
‫‪instead of giving disease name they asked phatophysiology of diseases in‬‬
‫‪option, mode of inheritance, histopatgology.‬‬
‫‪Same with drugs they‬‬
‫‪gave the clear cut scenario but instead of drugs name they mentioned‬‬
‫‪mode of action; interaction, S/E.‬‬
‫‪There were 4 to 5 question from Anatomy, very confusing options.‬‬
‫‪4 to 5 questions from SMLE Q 13 edition. and only 1 from UQU.‬‬

‫‪October 24, 2017‬‬ ‫‪283‬‬


‫‪7‬‬

‫مذكره ‪ 03‬افضل مصدر في وجهة نظري لالختبار‬


‫اسئلة البايوستات بالنص باالرقام‬
‫الكومن جدا كثير كثير وبالنص لكن ما افتكر نسبه المهم البد من النوم الكافي قبل االختبار‬
‫االختبار طويل و ممل خذ معاك غدا سريع زي ساندوتش و مويه او عصير‬
‫بفضل البروتينات زي البيض او الدجاج عشان تحافظ على تركيزك طول الثالث ساعات‬
‫و بعدو عن الحال زي الشوكالته و غيره اضمنلكم بعد ساعه ونص يجيك خمول بعدها‬
‫االختبار جميل و مكرر حاولو تحفظو المدونه مزبوط وتتاكدو من الحلول الصحيحه‬
‫في بريك في النص نص ساعه هذا اللي تقدر تتغدا فيه بسرعه و تدخل الجمام هللا يكرمكم‬
‫نصيحتي ال تستعجلو في الخروج بدري اذا بقيلكم شوبه من الوقت استفيدو من كامل الوقت‬

‫‪8‬‬
‫شباب انا اختبرت الخميس الماضي وتكرر علي نص االختبار على االقل‪.‬‬
‫فيه كم نقطة ندمت عليها‪.‬‬
‫‪- ٨‬انا ذاكرت اسئلة بنك فقط من دون حتى ما ارجع اقرا عنها او اتاكد من االجابة ‪ ،‬هذا‬
‫غلط الن الحظت ان ((االسئلة الجديدة ماتخرج عن مواضيع البنك)) ‪ .‬فلو اني قريت عن كل سؤال‬
‫شوي بس كان حسمت كثير من االسئلة الجديدة وارتحت بخصوص االسئلة المتكرره‪.‬‬
‫‪- ٧‬اسئلة االحصاء نفس السؤال يتكرر علي لكن غيرو االرقام ‪ ،‬لو كنت حافظ قوانين االحصاء اللي مرت‬
‫في البنك كان حليتها كاملة لكني اكره االحصاء والزلت ‪،‬ووقت ماطلعت من االختبار دريت ان درجة‬
‫اسئلته زي درجة اي تخصص ثاني‪.‬‬
‫‪- ١‬التخصصات الفرعية التهملها ابدا ابدا الن اسالتها متكررة ونفس درجة اسئلة الباطنة ‪ ،‬واسئلة‬
‫الباطنة كانت بديهية‪.‬‬

‫هذي كانت تجربتي انا ‪،‬اختبر معي واحد من الزمالء عنده نفس مالحظاتي خصوصا ان غالب المواضيع‬
‫نفسها اللي في البنك ‪،‬عدا انه ماتكرر عليه من االسئلة ‪ ٪٨١‬بالكثير‪.‬‬

‫هللا يوفقنا و يوفقكم ‪،‬ودعواتكم باقي انتظر درجتي تطلع‬

‫‪9‬‬
‫*اول حاجه االمتحان عموما ما صعب ‪ ..‬يعني نسبة الصعوبه ‪ .. %41‬وانا شايف اللي ذاكر مذكرة ‪03‬‬
‫والحرس ‪ ..‬يضمن يجاوب على ‪ 21‬او ‪ %20‬من االسئله‪..‬‬
‫*الفتره االولى ‪ 001‬سؤال شبه واضحين ‪ ..‬واسئله قصيره‪..‬‬
‫*بالنسبه لنسبة االسئله‪..‬‬
‫تقريبا ‪ ،% medicine 61‬وغالبا ‪ cardio ..‬وممكن ال‪ 21‬الباقيه‪ops & pediatric ..‬‬
‫الجراحه تقريبا ‪ 01‬اسئله بالكتييييير‪..‬‬
‫سايكولوجي تقريبا ‪ 00‬سؤال‪..‬‬
‫اطفال بالهبل ‪ ..‬انا حسبت نفسي بمتحن في تخصص اطفال‬

‫‪October 24, 2017‬‬ ‫‪284‬‬


‫بيزك في الورقه التانيه ‪ ،‬بس ما كتير ‪ ..‬حوالي ‪ 41‬او ‪ 40‬سؤال ‪ ..‬بس صعبين ‪ ..‬اول مره اشوفهم‪..‬‬
‫اندوكراين ‪ ..‬يعني حوالي ‪ 01‬اسئله‪..‬‬
‫المدونه لو قريتها كامله مع الشرح حق االجابه ‪ ..‬تضمن النجااح بإذن هللا‪..‬‬
‫االدويه ‪ ...‬حوالي ‪ 31‬او ‪ 21‬سؤال عن االدويه ‪ ..‬ال ‪action‬او‪side effect‬‬
‫االسئله اللي طلب ‪ investigation‬تقريبا ‪ 01‬اسئله بس‪..‬‬
‫نصيحه ‪ ..‬مذكرة ‪ 03‬وجزء من الحرس ‪ ..‬اسبوعين كاااااافيه تحفظ المذكره ‪ ..‬وبإذن هللا نجاح مؤكد‪..‬‬
‫اول جلسه خلصتها في ساعتين اال سوي ‪ ..‬باقي لي ساعه بس خالص انتهيت وطلعت ‪ ..‬يعني بتمشي‬
‫فيها سريع عشان اسئله قصيره وواضحه‪..‬‬

‫بس الفتره التانيه خلصتها في ساعتين وربع تقريبا ‪ ..‬عشان السيناريوهات طويييله‪..‬‬
‫في موقع الهئية في اختبار تجريبي ‪ ..‬حوالي ‪ 41‬سؤال ‪ ..‬جابو منها ‪ 😅 2‬ممكن تشوفوها ‪ ..‬اسئله‬
‫ثابته‬
‫في مذكرة ‪ 03‬اسئلة وأجوبة وشرح ‪ ..‬يعني تقريبا شبه دراسة نظري ‪ ..‬تحل كتير على قدر ما تقدر‬
‫تغطي‪..‬‬
‫‪75%‬من المدونة ‪ ،‬بس شامل الحلول والشروح حقتها ‪ ..‬يعني جابو اسئله من الشرح حق المدونة ‪..‬‬
‫يعني مش كلها منسوخة بالضبط‪..‬‬

‫*أسئلة الفارما ‪ ..‬اكتر االدوية كانت في حاالت الميدسن ‪ ..‬يعني اي دواء تقراه ‪ ،‬حاول تشوف بسرعه‬
‫‪mode of action and side effect ..‬‬

‫*بالنسبة لالحصاء ‪ ..‬جابو حوالي ‪ 00‬سؤال ‪ ،‬فيها بعضهم يحتاج حساب زي سألوا عن‬
‫ال ‪prevelence & incidence & odd rate & relative risk‬كل مسألة لوحدها‪..‬‬

‫حقيقه انا شايف انها بتضيع زمنك في االمتحان ألنه حتفتح اآلله الحاسبه وتحسب ‪ ..‬لو عندك وقت‬
‫اقراهم بسرعه وحل كم مسألة ‪ ،‬وقت االمتحان ممكن تحلهم بسرعه‪..‬‬

‫‪11‬‬
‫االختبار ‪ ١١١‬سؤال ‪ ٤‬ساعات‬
‫بشكل عام يعتمد على المعلومات من السنوات السابقة‬
‫أكثر األسئلة من النساء والوالدة‬
‫األطفال‬
‫العظام‬
‫والجراحة قليلة جدا ومباشرة‬
‫أسئلة العظام يجب التركيز على األصابات الرياضية‬
‫أسئلة االحصاء صعبة شوي كالمعتاد وبعضها معقد ماينحل‬
‫بس في مالحظة أذا وصلت إلى السؤال المية والخمسين وخلصتى منه الزم تنادين على المراقب عشان‬
‫يشرح لك طريقة أنهاء الجزء األول تالي عشان تطلع بريك‬
‫وال أذا كملت بدون مايشرح المراقب بتجلسين ‪ ٤‬ساعات متواصلة‬

‫‪October 24, 2017‬‬ ‫‪285‬‬


‫التجارب الجديدة من تاريخ ‪ 01/9‬حتى نهاية الفترة االختبارية في ‪ 04‬أكتوبر‬

‫‪11‬‬

‫‪9/41‬‬

‫اليوم اختبرت وأتمنى افيدكم ولو بمعلومة ‪ ...‬االختبار تقريبا ‪ %61‬من مذكره ‪ 03‬بالنص والباقي جديد‬
‫بس في نفس المواضيع واغلبها معلومات أساسيه وبديهه تعتمد ع االنتومي والفيزيو والبايوكمستري ‪..‬‬
‫اإلختبار نوعا ما طويل ومرهق شويه يحتاج النوم كويس قبل االختبار ‪ ..‬و ربي يكتب لنا جميعا التوفيق‬
‫وتيسير األمور‬

‫من أسئله امس برضو كانو طالبين معادله ‪Relative Risk‬‬

‫‪12‬‬

‫اختباري يوم ‪ ٧٨‬سبتمبر ‪٧١٨٢‬‬

‫حسيت اغلبه قايني وبيديا وكوميونتي وريسيرش وسايكتري‬

‫االوفثا واالي ان تي مرة قليل البايو من الملزمة كلو‬

‫‪13‬‬

‫‪9/40‬‬

‫اليوم كان اختباري للمره االولى‪ ،‬وهذي انطباعاتي‪:‬‬

‫‪ -٨‬تقريبا نص االختبار مكرر من المذكرات‪ ،‬او اكثر شوي‪ .‬االسئلة بشكل عام بديهيه‪ ،‬وممكن الواحد‬
‫يستخدم المنطق كثير باستبعاد االجوبه‪ .‬االختيارات اللي باالسئلة تكون سهلة االستبعاد غالبا‪.‬‬

‫‪ -٧‬الوقت كان جدا كافي بالنسبه لي‪ ،‬كنت اخلص القسم (‪ ٨٧١‬سؤال) ويبقى معي ساعه او ساعه و‪،٨١‬‬
‫وأمداني أراجع كل االسئلة‪.‬‬

‫‪ -١‬اسئلة ال ‪ stats and epidemiology‬سهله جدا جدا‪ ،‬بس الواحد الزم يفهم االساس‪ .‬عدد‬
‫االسئلة اكثر من ما توقعت‪ ،‬لكن باذن هللا انهم من صالحي‪.‬‬

‫‪October 24, 2017‬‬ ‫‪286‬‬


‫طبعا ال يخلو االختبار من اسئلة عجيبه‪ ،‬تخليك تستغرب وجود مثل هذي االسئلة باالختبار‪ .‬لكنها قليله‬
‫كانت باختباري‪ ،‬تقريبا ‪ ٨١‬اسئلة‪.‬‬

‫‪14‬‬
‫‪9/40‬‬

‫الحمد هلل اليوم اختبرت‬

‫االختبار بشكل عام معقول جدا ً يحتاج انك تكون نايم وماكل كويس‬

‫النه مجهد بصراحة وإذا بديت باالختبار ما تطلع اال بعد تحل ‪ ٨٧١‬سؤال‬

‫تكررت أسئلة كثير من المدونة واللي ما تكرر فهو بنفس المستوى‬

‫االختبار ما هو تعجيزي أبدا ً بالعكس وهللا يوفقكم جميعا ً يا رب‬

‫‪15‬‬
‫اليوم كان اختباري ‪١/٧٤‬‬

‫بالبداية بعض المعلومات عن االختبار‪:‬‬

‫‪ ١ -‬ساعات ‪ ٨٧١‬سؤال بريك ‪ ١١‬دقيقة‬

‫‪ ١‬ساعات ‪ ٨٧١‬سؤال‬

‫‪ -‬نام كويس اليوم الي قبله وافطر كويس يوم االختبار‬

‫‪ -‬اللبس البس الي يريحك‬

‫‪ -‬تروح مكان االختبار حتعبي اقرار وبعدين تحط كل الي معاك بالخزانة تدخل االختبار ما معاك اال مفتاح‬
‫الخزانة وبطاقة االحوال‬

‫‪ -‬مافيه ورقه وقلم ‪ -‬الحاسبة موجوده بنفس البرنامج االختبار‬

‫‪ -‬اول ما تقعد فيه توتولاير لمده ‪ ٧‬دقايق بس يشرح لك عن البرنامج ومزاياه‬

‫‪ -‬البرنامج سهل التعامل معه ويمديك تسوي‪ :‬هاياليت ‪ -‬تستبعد الخيارات الخاطئة ‪ -‬وتحط مارك على‬
‫السؤال عشان ترجع له بعدين‬

‫‪ -‬الوقت كافي جدا ً ويمديك ترجع تراجع االسئلة الي سويت لها مارك‬

‫‪October 24, 2017‬‬ ‫‪287‬‬


‫‪ -‬السيناريوز ماكانت طويله وفيه اساله كثيره سطر واحد ومباشره‬

‫نجي لالختبار نفسه‬

‫‪ -‬اشوف انه موزع بالتساوي ومافيه قسم اكثر من قسم‪ ،‬واشوف كل توبك بالطب جا منه سؤال واحد‬
‫على االقل‬

‫‪ -‬االختبار ما اشوفه صعب ابدا ً بالعكس اشوف صعوبته متوسطه ومو مستاهل التخوف الي نحس فيه‬
‫بالعكس اشوف اساله الكليه اصعب‬

‫‪ -‬واالسئلة الي تحسها صعبه من الخيارات يمديك تطلع الجواب الصح موب زي اسئله الكلية‬

‫‪ -‬التكرار موجود والصراحة ما دققت كم نسبته من االختبار‬

‫‪16‬‬
‫‪My exam on 27 september 2017‬‬

‫*االختبار ابدًا ما يستدعي القلق‪ .‬و الوقت يكفي و زياده و تقدر تراجع فيه زر بالسيستم ( ‪review‬‬
‫‪ ) all‬لمن تخلص السكشن االول ابو ‪ ٨٧١‬سؤال او تختار رقم سؤال معين تراجعه‬

‫* السيستم مره سهل وواضح و بسيط‪ .‬و فيه آله حاسبه‬

‫صا تقريبا ‪ ١١-١١‬سؤال يعني كنز بجيبك‬


‫* ركزوا على البييييييييسك بدون مبالغه جتني ن ً‬

‫* االختبار ابدا مو ذاك الصعوبه لكن بنفس الوقت ال تضمن النص نفسه ( اضمن فهمك هو نفسه )‬

‫* السيناريوز ممكن تكون طويله لكن مو كلها ومو معناته صعبه ابدًا بالعكس لو ركزت ممكن كلمه او‬
‫كلمتين تعطيك الحل على طول‬

‫* لحد يأجل السؤال نصيحه ( ممكن ينحسب بالسيستم انه غلط ) محد يضمن التقنيه النه الموضوع كله‬
‫تقني حاول تحل السؤال قد ما تقدر‬

‫* االختبار ‪sections 4‬‬

‫* ‪Each section has 150 Qs with timer you see it in the screen won't‬‬
‫‪stop even of you take a break so even your break is counted so you‬‬
‫‪have to tell them that you'll take a break so they check your time‬‬

‫* البريك حقك يعتمد على متى تخلص السكشن االول يعني الباقي من الوقت للثالث ساعات االولى بعد ما‬
‫تخلص السكشن لالول هو البريك حقك فيه ناس ساعه و ناس نص ساعه و ضرووووري ترتاح ال‬
‫تسوي فيها انك سوبر مان ترا مرهق االختبار‬

‫‪October 24, 2017‬‬ ‫‪288‬‬


‫* نااااام كويس و جيب معك أكل و االحوال و بس‬

‫‪Majority research alot & community family medicine and Pediaaaaa‬‬


‫‪huge‬‬

‫‪Little ophtha , ortho , GS , medicine , Ob‬‬

‫‪Qs ENT 0‬‬

‫‪17‬‬
‫اختباري كان يوم الخميس ‪ ٧١‬سبتمبر‪.‬‬

‫رحت بدري دخلتني االختبار من ‪ ١‬اال عشرة بديت تمنية بالتمام‪.‬‬

‫االختبار كان ال باس فيه في اسئلة سهلة مره لدرجة ماتتوقعها وفي اسئلة متوسطة ‪ ١١ ،‬في المية من‬
‫االختبار تقريبا كان من المدونة ‪.٨١‬‬

‫ماحس انه البلوبرينت كان صادق ‪ ،‬ميدسن كان اغلبه كارديو شوي هيما شوي انفكشوس‪.‬‬

‫البيديا واالوبي كان غالب كم سؤال من االوفثا ويمكن سؤالين من اإلي ان تي ‪.‬‬

‫البيسك ينحفظ زي ماهو تقريبا جايبينه نسخ لصق ‪ ،‬الريسيرش والكوميونتي المسائل نفس االرقام بس‬
‫في كم سؤال يمكن ‪ ٧‬او ‪ ٤‬ماكانت من المدونة ومافتكرها‪.‬‬

‫‪18‬‬
‫اختباري كان الخميس‬

‫‪0 / October 2017‬‬

‫♥ ‪: General advices‬‬

‫نومة حلوة قبل االختبار و تريحو عينكم من الجوال ايباد البتوب تلفزيون ‪ ....‬او اي جهاز و قبل االختبار‬
‫اذكار الصباح وفطور خفيف و خدو معاكم عصير و سناك لوقت البريك‬

‫وانتو بتختبرو حاولو كل ربع ساعة تطالعو بعيد عن الشاشة عشان ال ترهقو عيونكم ال تخافو الوقت‬
‫مررا كتير بزيادة فال تشيلو همه ابداً‬

‫‪October 24, 2017‬‬ ‫‪289‬‬


‫احلى شي انو وقت االختبار مافي اي مقاطعة وال ازعاج منتهى الرواقة الحمدهلل هوا كان مررا برد‬
‫بالنسبالي المرة الجيا حاالبس كم طويل ان شاءهللا ونبدأ بسم هللا اللي قدرت اتزكرو‬

‫‪, Most questions were : Obgyne , research‬‬

‫وحرام عليهم هلكووووونييييي كارديو و ‪ECG‬‬

‫وكل جينات الدنيا سألوني عنها حتى بدأت أشك انهم بيطقطقو عليا و بيخرفو من راسهم اي حروف جمب‬
‫بعض و يقولو عنها جين‬

‫هللا يوفقكم كلكم يارب ويكتبلكم الخير في كل شي تسووه‬

‫‪19‬‬

‫اختباري كان يوم ‪ ٧‬أكتوبر‬

‫كان فيه اسئلة سايكاتري كثير اغلبها سهل اللي برا الملزمة كانت اسئلة ال ‪personality‬‬
‫‪ disorder & defense mechanism‬وفي أدوية السيكاتري ركزوا برضو على السايد ايفيكتس‬
‫حقتها وجاني برضو كثير من اسئلة دارين اللي ارسلتها اول شوفوها‬

‫‪21‬‬
‫‪9/10/2017‬‬

‫ذاكرت ‪ ٨١‬و جا اسئله كوبي بيست منها و اذا ما كانت نفسها بال ّظبط‬

‫كانت األسئله تدور حول نفس المواضيع الريسيرتش جاني كوبي بيييست‬

‫و التجميعات ث ّم التّجميعات ث ّم التجميعات وهللا فتحتها قبل االختبار بيومين و ص ّميتها و جاني منّها كثييير‬

‫البيسكز مهمه ترا بالنّسبه لي اناتومي كان كثير مو كثير مره بس هو اللي جاني‬

‫ع فكره ملزمه ‪ ٨٧‬ستيل فيها اخطاء فالزم اللي يذاكر منّها يرجع يصحح‬

‫ماصورينه‬
‫ّ‬ ‫سوء اللي‬
‫االختبار ترا بالمتناول و مو بذيك الصعوبه و ال ّ‬

‫بس يحتاج مذاكره و جهد و بحث و استعداد و هللا ما يضيّع تعب احد باذن هللا‬

‫‪October 24, 2017‬‬ ‫‪290‬‬


‫نسوي اللي علينا و الباقي هللا يتوالّنا فيه‬
‫احنا ّ‬

‫‪21‬‬
‫‪٨١/٨١‬‬

‫االختبار بين السهل والصعب ومقدور ع حله باذن هللا فيه اسالة تكررت من مذكرة ‪ ٨١‬وكمان اساله‬
‫تكررت من االساله الجديده حقت شهر اوقست واكتوبر‬

‫االسالة الجديدة اللي ماتكررت ماكانت صعبه مره يعني ممكن يجيبون لك نفس السؤال من مذكرة ‪٨١‬‬
‫بس بدل مايطلبون منك تشخيص يطلبون مننجمنت‪ ،‬يعني لو مذاكر مذكره ‪ ٨١‬زين تقدر بالراحه تحل‬
‫االساله الجديدة‬

‫البرانشات الصغيرة اسالتها كوبي بيست من المذكرة ( االوفثا ‪ -ENT -‬االورثو _ االناثيزيا ‪)..‬‬

‫الريسيرش تكرر منه اساله المهم احفظوا التعاريف والمعادالت‬

‫السايكاتري كمان كرروا وفيه اساله جديده بس واضحة‬

‫االختبار التجريبي اللي نزلوه تكرر لي منه ‪ ١‬اساله بالظبط‬

‫اهم شي الواحد يكون ريالكس واليستعجل بالحل معاه ‪ ٤‬ساعات‬

‫وموفقين يارب 🌷‬

‫‪22‬‬
‫اختباري كان يوم ‪..‬‬

‫‪4102-01-01‬‬

‫‪ -‬نصيحه ص ّموا االسئله حقت اللي اختبروا قريب تكررر منها كثيييير وتاكدوا من االجوبه بانفسكم‬

‫‪ -‬ركزوا على ادوية ال ‪ psychiatry‬جت اسئله كثيره عليها‬

‫‪ -‬جابوا كثير من الريسيرتش‬

‫‪October 24, 2017‬‬ ‫‪291‬‬


‫‪23‬‬
‫انا امتحاني يوم ‪٧١٨٢-٨١-٨٨‬‬

‫أول حاجه الترتبكون ‪ ،‬كالم البعض لألسف خالنا نعيش بخوف ورهبه ولكن الموضوع مايستدعي كل‬
‫هذا ‪.‬‬

‫األسئله في بعضها تكرر بالضبط وبعضها أل بس ماكانت اسئله غريبه بلعكس مريتوا عليهم ب الملزمه‬
‫بس مثال بالملزمه كانوا يبون عالج وباألمتحان جايبين األعراض ويبوون التشخيص ‪..‬‬

‫ف حاولوا تقروون شوي عن المواضيع اذا مداكم‬

‫وبالنسبه لي تمنيت اني خصصت كم يوم للمراجعه ألن في اشياء جات واذكر مريت عليها بس ناسيه‬
‫اإلجابه ‪.‬‬

‫سهاالت بأذن هللا ‪ ،‬تطمنوا وناموا زين ألن تحتاجون تكونون مصحصحين‬

‫وصح ‪ ،‬اسئله الريسيرتش حاولوا تضمنونها ف التسحبون ع الشابتر بالملزمه‬

‫هللا يوفقنا جميعا ‪..‬‬

‫‪24‬‬
‫كان اختباري ‪ 12‬اكتوبر‬

‫االختبار كان اغلبه أوبي وسايكا وبيسك وطوارئ والباقي متوزع بين باقي االقسام‬

‫ما أزيد على كالم اللي اختبرو قلبي من ناحية أهمية البيسك والريسيرتش لكن الشي اللي استغربته اليوم‬
‫تركيزهم ع األدوية من ناحية ال‪ side effects + mechanism of action‬لدرجة اني حسيته‬
‫اختبار فارما مو طب‬

‫عموما اللي باقي له أيام قليلة ع اختباره يحاول يفتح الفارما بقسم البيسك‬

‫باالضافة الى‬

‫‪side effects of psychiatric drugs‬‬

‫الن في أدوية نفسية جات من برا المدونة‬

‫‪October 24, 2017‬‬ ‫‪292‬‬


‫‪25‬‬

‫اختباري يوم ‪ ٨٧‬اوكتوبر ‪ ،‬االسئلة كانت من سهله الى متوسطة الى صعبه ‪ ،‬واالغلب متوسطة وتكرر‬
‫من المذكرة كثير وكمان من االسئلة اللي كتبوها الزمالء مؤخرا ً هللا يعطيهم العافيه ‪ ،‬هذا اللي اذكره حاليا ً‬
‫‪ ..‬التنسونا من صالح الدعاء‬

‫‪October 24, 2017‬‬ ‫‪293‬‬


Resources Suggested by the ones who did the
Exam: (UPDATED)

1- SMLE 13 2017 ( ‫ أحدث إصدار‬03 ‫ ) المدونة‬-- the link to download Here

2- SMLE - KSAUHS ( ‫ ) مذكرة الحرس‬-- the link to download Here

3- QuickSMLE+ (Smleworkup ‫ ) تصحيح الحرس و‬-- the link to download Here

4- Master the board step 2 CK 2017 -- the link to download Here

5- First Aid Step 2 CK 8th ed -- the link to download Here

6- Toronto notes 2017 -- the link to download Here

7- ‫ أسئلة الهيئة العشرين التجريبية‬-- the link to download Here

8- %011 ‫ ملف‬-- the link to download Here

‫تمت النسخة الثانية‬


‫إذا فادتك ولو بشيء بسيط ال تنسانا من دعائك‬

October 24, 2017 294

You might also like